├── src
├── .gitignore
└── Tables.hs
├── .gitignore
├── chapter 4
├── section4-1.pdf
├── prog.hs
├── section4-1.tex
├── section4-2.tex
├── section4-3.tex
└── section4-5.tex
├── chapter 3
├── factor.hs
├── section3.2.tex
└── section3.4.tex
├── README.md
├── chapter 2
├── section2-3.md
├── section2-2.md
├── section2-1.md
└── section2-3.tex
├── introduction
└── chapter0.md
├── chapter 6
├── section6-4.tex
├── section6-1.tex
└── section6-2.tex
├── chapter 1
├── section1-3.md
├── chapter1-1.md
├── section1-4.md
├── section1-5.md
└── chapter1-2.md
└── chapter 5
└── section5-3.tex
/src/.gitignore:
--------------------------------------------------------------------------------
1 | *hi
2 | *o
3 | Table.txt
4 | Tables
5 |
6 |
--------------------------------------------------------------------------------
/.gitignore:
--------------------------------------------------------------------------------
1 | *aux
2 | *dvi
3 | *log
4 | *pdf
5 | *fmt
6 | *prv
7 | _region_.tex
8 |
--------------------------------------------------------------------------------
/chapter 4/section4-1.pdf:
--------------------------------------------------------------------------------
https://raw.githubusercontent.com/psibi/how-to-prove/HEAD/chapter 4/section4-1.pdf
--------------------------------------------------------------------------------
/chapter 4/prog.hs:
--------------------------------------------------------------------------------
1 | import Data.List (nub)
2 |
3 | atLeastOne :: String -> String -> Bool
4 | atLeastOne x y = length (x' ++ y') /= length (nub (x' ++ y'))
5 | where x' = nub x
6 | y' = nub y
7 |
8 | allCom :: [a] -> [(a,a)]
9 | allCom xs = [(x,y) | x <- xs, y <- xs]
10 |
11 | problem2 :: [(String, String)]
12 | problem2 = filter (\(x,y) -> atLeastOne x y) r
13 | where r = allCom ["cat", "dog", "bird", "rat"]
14 |
--------------------------------------------------------------------------------
/chapter 3/factor.hs:
--------------------------------------------------------------------------------
1 | findFactors :: Integral a => a -> [a]
2 | findFactors num = take 1 $ filter (\x -> num `mod` x == 0 && x /= num) [2..num]
3 |
4 | hasFactor :: Integral a => a -> Bool
5 | hasFactor x = if (null (findFactors x))
6 | then False
7 | else True
8 |
9 | isPrime :: Integral a => a -> Bool
10 | isPrime x = not $ hasFactor x
11 |
12 | problem3 :: Num a => a -> a
13 | problem3 n = 2*n + 13
14 |
15 | primeProblem3 :: [Integer]
16 | primeProblem3 = take 1 $ filter isPrime $ map problem3 [3..]
17 |
--------------------------------------------------------------------------------
/src/Tables.hs:
--------------------------------------------------------------------------------
1 | import Data.Logic.Propositional
2 | import System.IO
3 |
4 | p = Variable (Var 'p')
5 | q = Variable (Var 'q')
6 |
7 | exp1 = (Negation p) `Disjunction` q
8 | exp1T = truthTable exp1
9 |
10 | --(S ∨ G) ∧ (¬S ∨ ¬G).
11 |
12 | s = Variable (Var 's')
13 | g = Variable (Var 'g')
14 |
15 | exp2 = (s `Disjunction` g ) `Conjunction` (Negation s `Disjunction` Negation g)
16 | exp2T = truthTable exp2
17 |
18 | -- ¬[P ∧ (Q ∨ ¬P)]
19 |
20 | exp3 = Negation $ p `Conjunction` (q `Disjunction` (Negation p))
21 | exp3T = truthTable exp3
22 |
23 | -- (P ∨ Q) ∧ (¬P ∨ R)
24 | r = Variable (Var 'r')
25 | exp4 = (p `Disjunction` q) `Conjunction` (Negation p `Disjunction` r)
26 | exp4T = truthTable exp4
27 |
28 | main = writeFile "/home/sibi/github/prove/src/Table.txt" exp4T
29 |
30 |
--------------------------------------------------------------------------------
/README.md:
--------------------------------------------------------------------------------
1 | How to Prove It: A Structured Approach
2 | ------------
3 |
4 | Contains solution for the Velleman's book.
5 |
6 | The reason I have started studying this is to ultimately study
7 | [type theory](http://en.wikipedia.org/wiki/Type_theory).
8 |
9 | Feel free to raise
10 | [issue](https://github.com/psibi/how-to-prove/issues) if you think a
11 | proof is wrong or if it needs some clarification. Pull requests and
12 | contributions are welcome.
13 |
14 | Credits
15 | --------
16 |
17 | * [David Cantrell](https://github.com/davecan) for Section 3.3 and
18 | various other improvements.
19 |
20 | Notes:
21 | -------
22 |
23 | From Chapter 2, I have moved from markdown format to Latex as it helps
24 | in much easier rendering of mathematical symbols. It can be compiled
25 | using `pdflatex`:
26 |
27 | `pdflatex tex_filename`
28 |
29 | Although I did solve Chapter 3 problems, I haven't uploaded all of
30 | them yet because of my laziness.
31 |
--------------------------------------------------------------------------------
/chapter 2/section2-3.md:
--------------------------------------------------------------------------------
1 | More operations on Set
2 | -----------------------
3 |
4 | Exercise 1
5 | -----------
6 |
7 | Analyze the logical forms of the following statements. You may use the
8 | symbols ∈, ∈,
9 | / =, =, ∧, ∨, →, ↔, ∀, and ∃ in your answers, but not
10 | ⊆, ⊆, P , ∩, ∪, \, {, }, or ¬. (Thus, you must write out the definitions
11 | of some set theory notation, and you must use equivalences to get rid of
12 | any occurrences of ¬.)
13 | (a) F ⊆ P (A).
14 | (b) A ⊆ {2n + 1 | n ∈ N}.
15 | (c) {n 2 + n + 1 | n ∈ N} ⊆ {2n + 1 | n ∈ N}.
16 | (d) P (∪ i∈I A i ) ⊆ ∪ i∈I P (A i ).
17 |
18 | (a)
19 |
20 | * `F ⊆ P (A)`
21 | * `∀x (x ∈ F -> x ∈ P(A))`
22 | * `∀x (x ∈ F -> x ∈ P(A))`
23 | * `∀x (x ∈ F -> x ∈ P(A))`
24 | * `∀x (x ∈ F -> x ⊆ A)`
25 | * `∀x (x ∈ F -> ∀y(y ∈ x -> y ∈ A))`
26 |
27 | (b)
28 |
29 | * `A ⊆ {2n + 1 | n ∈ N}`
30 | * `∀x (x ∈ A -> x ∈ {2n + 1 | n ∈ N})`
31 | * `∀x (x ∈ A -> ∃n ∈ N(x = 2n + 1))`
32 |
33 | (c)
34 |
35 | * `{n^2 + n + 1 | n ∈ N} ⊆ {2n + 1 | n ∈ N}`
36 | * `∀x (x ∈ {n^2 + n + 1 | n ∈ N} -> x ∈ {2n + 1 | n ∈ N})`
37 | * `∀x (∃n ∈ N(x = n^2 + n + 1) -> ∃n ∈ N(x = 2n + 1))`
38 |
39 | (d)
40 |
41 | * `¬(P(∪i∈IAi) ⊆ ∪i∈IP(Ai))`
42 |
43 |
--------------------------------------------------------------------------------
/introduction/chapter0.md:
--------------------------------------------------------------------------------
1 | Introduction
2 | ------------
3 | ------------
4 |
5 | Exercise 1) a)
6 | --------------
7 |
8 | If n is not prime, then 2^n - 1 is also not a prime.
9 |
10 | 2^15 - 1 = 32,767
11 |
12 | Factors of 15 = 5 * 3
13 | In the proof of Conjecture 2, they use two numbers x & y.
14 | x = 2 ^ b - 1
15 |
16 | So, one of the factors of 32,767 is 2 ^ 5 - 1 = 31
17 | And the other factor is 32767 / 31 = 1057
18 |
19 | Exercise 1) b)
20 | --------------
21 |
22 | Find an integer x such that 1 < x < 2^32767 - 1 and 2^32767 - 1 is
23 | divisible by x.
24 |
25 | So, 2^32767 - 1 should be divisible by x.
26 |
27 | We know that 32767 is not a prime (from Ex 1a), so 2^32767 - 1 is also
28 | not a prime and hence has factors.
29 |
30 | Factors of 32767: 1057, 31
31 |
32 | So, from the proof of Conjecture 2, one of the factors of 2^32767 - 1
33 | should be 2^b - 1
34 |
35 | So, one factor of 2^32767 - 1 is 2^1057 - 1 = a big number.
36 | Another factor of 2^32767 - 1 is 2^31 - 1 = 2147483647
37 |
38 | Exercise 2)
39 | ------------
40 |
41 | Since they are asking for conjecture, I can frame any bullshit
42 | arguments here. But I will try to be reasonable. :P
43 |
44 | Ok, now I have written a
45 | [program](https://github.com/psibi/rwh/blob/bb424ba002c5aef0b5edd105f98d1825446d066c/misc/Factor.hs)
46 | for generating data from 1 to 10, and check whether they are prime for
47 | each of our test case.
48 |
49 | The first column indicates number, the second one indicates 3^n -1 and
50 | the third one indicates 3^n - 2^n.
51 |
52 | * (2,False,True)
53 | * (3,False,True)
54 | * (4,False,False)
55 | * (5,False,True)
56 | * (6,False,False)
57 | * (7,False,False)
58 | * (8,False,False)
59 | * (9,False,False)
60 | * (10,False,False)
61 |
62 | Well, so looking at the second column it seems I can come up with some
63 | sensible conjecture:
64 |
65 | Sibi's (Too much, huh?) conjecture: 3^n - 1 is never a prime number for
66 | any positive integer greater than 1.
67 |
68 | For third column, I can't see any pattern there. Still I can produce
69 | some idiotic conjecture: For all numbers n greater than 5, 3^n - 2^n is
70 | not a prime number.
71 |
72 | Exercise 3)
73 | -----------
74 |
75 | The proof of Theorem 3 gives a method for finding a prime number different
76 | from any in a given list of prime numbers.
77 | (a) Use this method to find a prime different from 2, 3, 5, and 7.
78 | (b) Use this method to find a prime different from 2, 5, and 11.
79 |
80 | The way to solve this problem is to understand Theorem 3 (Euler's
81 | Theorem ?).
82 |
83 | (a) 2.3.5.7 + 1 = 211
84 |
85 | (b) 2.3.5.7.11 + 1 = 2311
86 |
87 | Or 2 + 1 = 3
88 |
89 | 2.3 + 1 = 7
90 |
91 | They are giving redundant information (5,11) in the question to
92 | confuse you!
93 |
94 | Exercise 4
95 | -----------
96 |
97 | Find five consecutive integers that are not prime.
98 |
99 | We need to use Theorem 4 for solving this problem.
100 |
101 | Let n = 5. By Theorem 4, 5 numbers from (n+1)! + 2 will not be prime.
102 |
103 | They are: 722, 723, 724, 72, 726
104 |
105 | Exercise 5
106 | -----------
107 |
108 | Use the table in Figure 1 and the discussion on p. 5 to find two more perfect
109 | numbers.
110 |
111 | Two perfect numbers are already given in the book: 6,28.
112 |
113 | Using Euclid's proof on p. 5 that if 2 ^ n - 1 is prime, then (2 ^ (n - 1)) (2 ^ n - 1) is perfect, we can see that n = 2 and n = 3 gives us the 6 and 28.
114 |
115 | Using the other 2 values of n for which 2 ^ n - 1 is prime gives us two more perfect numbers.
116 |
117 | For n = 5 and n = 7 :
118 |
119 | (2 ^ (5 - 1)) (2 ^ 5 - 1) = 496
120 |
121 | (2 ^ (7 - 1)) (2 ^ 7 - 1) = 8128
122 |
123 |
124 | Exercise 6
125 | -----------
126 |
127 | The sequence 3, 5, 7 is a list of three prime numbers such that each pair of
128 | adjacent numbers in the list differ by two. Are there any more such “triplet
129 | primes”?
130 |
131 | I don't know, it seems unlikely though. Once again I will write a
132 | [program](https://github.com/psibi/rwh/blob/78a5676662b7ecc3b2a01bdeb326986bb4d496cb/misc/Factor.hs#L35)
133 | to check this stuff. I checked it from the first 2000 numbers and I
134 | wans't able to find any triplet primes, Sorry!
135 |
--------------------------------------------------------------------------------
/chapter 6/section6-4.tex:
--------------------------------------------------------------------------------
1 | %%%%%%%%%%%%%%%%%%%%%%%%%%%%%%%%%%%%%%%%%
2 | % Author: Sibi
3 | %%%%%%%%%%%%%%%%%%%%%%%%%%%%%%%%%%%%%%%%%
4 | \documentclass{article}
5 | \usepackage{graphicx}
6 | \usepackage{verbatim}
7 | \usepackage{amsmath}
8 | \usepackage{amsfonts}
9 | \usepackage{amssymb}
10 | \usepackage{tabularx}
11 | \usepackage{mathtools}
12 | \newcommand{\BigO}[1]{\ensuremath{\operatorname{O}\bigl(#1\bigr)}}
13 | \setlength\parskip{\baselineskip}
14 | \begin{document}
15 | \title{Chapter 6 (Section 6.2)}
16 | \author{Sibi}
17 | \date{\today}
18 | \maketitle
19 |
20 | % See here: http://tex.stackexchange.com/a/43009/69223
21 | \DeclarePairedDelimiter\abs{\lvert}{\rvert}%
22 | \DeclarePairedDelimiter\norm{\lVert}{\rVert}%
23 |
24 | % Swap the definition of \abs* and \norm*, so that \abs
25 | % and \norm resizes the size of the brackets, and the
26 | % starred version does not.
27 | \makeatletter
28 | \let\oldabs\abs
29 | \def\abs{\@ifstar{\oldabs}{\oldabs*}}
30 | %
31 | \let\oldnorm\norm
32 | \def\norm{\@ifstar{\oldnorm}{\oldnorm*}}
33 | \makeatother
34 | \newpage
35 |
36 | \section{Solution 1}
37 |
38 | \subsection{Solution (a)}
39 |
40 | ($\Rightarrow$) Suppose $\forall n Q(n)$. Let $n$ be an arbitrary element. Since $Q(n+1)$ is true, it follows $\forall m < n + 1(P(m))$. Let $m = n$. Since $n < n + 1$, it follows that $P(n)$. Since $n$ was arbitrary it follows that $\forall n P(n)$.
41 |
42 | ($\Leftarrow$) Suppose $\forall n P(n)$. Then it follows that $\forall k < n P(k)$. So, $Q(n)$.
43 |
44 | \subsection{Solution (b)}
45 |
46 | By mathematical induction on $n$,
47 |
48 | Base case. $n = 0$. $\forall k < n P(k)$ is vacously true.
49 | Induction step. Suppose $Q(n)$. So, $\forall k < n P(k)$. Since $k < n + 1$, it follows that $\forall k < n + 1 P(k)$. Now $\forall k < n + 1 P(k)$ is equivalent to $Q(n + 1)$.
50 |
51 | \section{Solution 2}
52 | $P(q) = \forall q \in N (q > 0 \implies \neg \exists p \in N(p/q = \sqrt{2}))$
53 |
54 | Let $q$ be an arbitrary element in $N$. Suppose $\forall k < q(P(k))$. Suppose $q > 0$. Let us prove by contradiction. There exists some $p \in N$ such that $p/q = \sqrt{2}$. So, $p^2 = 2q^2$. It follows that $p$ and $q$ are even. So, $p = 2p'$ and $q = 2q'$. Now, $p'/q' = \sqrt{2}$. Since $q' < q$, it contradicts with the Inductive hypothesis. So, $\sqrt{2}$ is irrational.
55 |
56 | \section{Solution 3}
57 | \subsection{Solution (a)}
58 | Suppose $\sqrt{6}$ is rational. Then, $\exists p \in Z^{+} \exists q \in Z^{+}(p/q = \sqrt{6})$. So the set $S = \{q \in Z^{+} \mid \exists p \in Z^{+}(p/q = \sqrt{6})\}$ is non-empty. By well ordering principle, let $q$ be the smallest element in $S$. Since $q \in S$, we can choose some $p \in Z^{+}$ such that $p/q = \sqrt{6}$. So, $p^2 = 6q^2$. We know that both $p$ and $q$ are even. So, let $p' = 2p$ and $q' = 2q$ such that $p'/q' = \sqrt{6}$. Since $q' < q$ it contradicts that $q$ is the smallest element of the set. So, $\sqrt{6}$ is irrational.
59 |
60 | \subsection{Solution (b)}
61 | Suppose $\sqrt{2} + \sqrt{3} = p/q$ where $p \in Z^{+}$ and $q \in Z^{+}$. So, $p^2 / q^2 = 5 + 2\sqrt{6}$. So, $\sqrt{6} = (p^2 - 5q^2)/q^2$. This contradicts the part (a).
62 |
63 | \section{Solution 4}
64 | To prove: $\forall n >= 12 \exists a \in \mathbb{N} \exists b \in \mathbb{N}(3a + 7b = n)$.
65 |
66 | We have to prove it like this:
67 |
68 | \begin{itemize}
69 |
70 | \item
71 | Prove for n=12,13,14
72 |
73 | \item
74 | $\forall n >= 15 \exists a \exists b. (n = 3a + 7b)$
75 | \end{itemize}
76 |
77 | I will just prove the second item here. Let $n >= 15$ be an arbitrary element.Suppose $\forall k( 15 <= k < n)$, there exists $a$ and $b$ such that $k = 3a + 7b$. Since $n - 3 < n$, it follows that $n - 3 = 3a + 7b$ from inductive hypothesis. So, $n = 3(a + 1) + 7b$. Let $a + 1 = a'$, So, $\exists a' \exists b (n = 3a' + 7b)$. Since $n$ was arbitrary, it follows that $\forall n >= 15 \exists a \exists b. (n = 3a + 7b)$.
78 |
79 | Also see this: http://math.stackexchange.com/a/1745857/124772
80 |
81 | \section{Solution 5}
82 |
83 | Let $n$ be an arbitrary number such that $n >= 1$. Suppose $\forall k < n$, $x^k + 1/x^k$ is an integer. Let $k = n-1$, then,
84 | \begin{gather*}
85 | x^{n-1} + \frac{1}{x^{n-1}} \text{ is an integer.}
86 | \end{gather*}
87 |
88 | Then,
89 | \begin{gather*}
90 | (x^{n-1} + \frac{1}{x^{n-1}})(x + \frac{1}{x}) \\
91 | = x^{n-1}.x + \frac{x}{x^{n-1}} + \frac{x^{n-1}}{x} + \frac{1}{x^{n-1}.x} \\
92 | = x^n + \frac{1}{x^n} + x^{n-1} + \frac{1}{x^{n-2}} \\
93 | \end{gather*}
94 |
95 | We know from inductive hypothesis that $x^{n-2} + \frac{1}{x^{n-2}}$ is integer. Also $x^n + \frac{1}{x^n} + x^{n-1} + \frac{1}{x^{n-2}}$ is integer. So, $x^n + \frac{1}{x^n}$ is integer.
96 |
97 | \section{Solution 6}
98 | \subsection{Solution (a)}
99 | By mathematical induction on $n$,
100 |
101 | Base case. When $n = 0$, both sides of the equation becomes $0$.
102 | Induction step. Suppose $\sum_{i=0}^{n} F_i = F_{n+2} - 1$. Then,
103 | \begin{gather*}
104 | \begin{align}
105 | \sum_{i=0}^{n+1} F_i = \sum_{i=0}^{n} F_i + F_{n+1} \\
106 | = F_{n+2} + F_{n+1} - 1 \\
107 | = F_{n+3} - 1
108 | \end{align}
109 | \end{gather*}
110 | So, $\sum_{i=0}^{n} F_i \implies \sum_{i=0}^{n+1} F_i$.
111 |
112 |
113 |
114 |
115 | \end{document}
116 |
--------------------------------------------------------------------------------
/chapter 1/section1-3.md:
--------------------------------------------------------------------------------
1 | Variables and Sets
2 | ------------------
3 | ------------------
4 |
5 | Exercise 1
6 | -----------
7 |
8 | Analyze the logical forms of the following statements:
9 | (a) 3 is a common divisor of 6, 9, and 15. (Note: You did this in exercise
10 | 2 of Section 1.1, but you should be able to give a better answer now.)
11 | (b) x is divisible by both 2 and 3 but not 4.
12 | (c) x and y are natural numbers, and exactly one of them is prime.
13 |
14 | (a)
15 |
16 | D(x,y) = x is a divisor of y
17 |
18 | D(3,6) ∧ D(3,9) ∧ D(3,15)
19 |
20 | (b)
21 |
22 | D(x,y) = x is divisible by y
23 |
24 | D(x,2) ∧ D(x,3) ∧ ¬(D(x,4))
25 |
26 | (c)
27 |
28 | N(x) = x is a natural number.
29 | P(x) = x is a prime number
30 |
31 | N(x) ∧ N(y) ∧ (P(x) ⊕ P(y))
32 |
33 | ⊕ is symbol for exclusive or. It will true when "only one" of its operand is true.
34 |
35 | Exercise 2
36 | ----------
37 |
38 | Analyze the logical forms of the following statements:
39 | (a) x and y are men, and either x is taller than y or y is taller than x.
40 | (b) Either x or y has brown eyes, and either x or y has red hair.
41 | (c) Either x or y has both brown eyes and red hair.
42 |
43 | (a)
44 |
45 | M(x) = x is men.
46 | T(x,y) = x is taller than y
47 |
48 | M(x) ∧ M(y) ∧ (T(x,y) ∨ (T(y,x)))
49 |
50 | (b)
51 |
52 | B(x) = x has brown eyes.
53 | R(x) = x has red hair.
54 |
55 | (B(x) ∨ B(y)) ∧ (R(x) ∨ R(y))
56 |
57 | (c)
58 |
59 | (B(x) ∧ R(x)) ∨ (B(y) ∧ R(y))
60 |
61 | Exercise 3
62 | ----------
63 |
64 | Write definitions using elementhood tests for the following sets:
65 | (a) {Mercury, Venus, Earth, Mars, Jupiter, Saturn, Uranus, Neptune,
66 | Pluto}.
67 | (b) {Brown, Columbia, Cornell, Dartmouth, Harvard, Princeton, Univer-
68 | sity of Pennsylvania, Yale}.
69 | (c) {Alabama, Alaska, Arizona, . . . , Wisconsin, Wyoming}.
70 | (d) {Alberta, British Columbia, Manitoba, New Brunswick, Newfound-
71 | land and Labrador, Northwest Territories, Nova Scotia, Nunavut, On-
72 | tario, Prince Edward Island, Quebec, Saskatchewan, Yukon}.
73 |
74 | (a)
75 |
76 | {x | x is planet in solar system}
77 |
78 | (b)
79 |
80 | {x | x is an ivy-league university }
81 |
82 | (c)
83 |
84 | {x | x is a state of US}
85 |
86 | (d)
87 |
88 | {x | x is a province in Canada}
89 |
90 | Exercise 4
91 | ----------
92 |
93 | Write definitions using elementhood tests for the following sets:
94 | (a) {1, 4, 9, 16, 25, 36, 49, . . .}.
95 | (b) {1, 2, 4, 8, 16, 32, 64, . . .}.
96 | (c) {10, 11, 12, 13, 14, 15, 16, 17, 18, 19}
97 |
98 | (a)
99 |
100 | {x^2 | x ∈ Z+}
101 |
102 | (b)
103 |
104 | {2^x | x ∈ N}
105 |
106 | (c)
107 |
108 | {x ∈ N | 9 < x < 20}
109 |
110 | Exercise 5
111 | -----------
112 |
113 | Simplify the following statements. Which variables are free and which are
114 | bound? If the statement has no free variables, say whether it is true or
115 | false.
116 | (a) −3 ∈ {x ∈ R | 13 − 2x > 1}.
117 | (b) 4 ∈ {x ∈ R − | 13 − 2x > 1}.
118 | (c) 5 ∈/ {x ∈ R | 13 − 2x > c}.
119 |
120 | (a)
121 |
122 | Free variables: None
123 | Bound variables: x
124 | Statement is true since 19 > 1.
125 |
126 | (b)
127 |
128 | Free variables: None
129 | Bound variables: x
130 | Statement is false since 5 doesn't belong to R^-
131 |
132 | (c)
133 |
134 | Free variables: c
135 | Bound variables: x
136 | Statement is neither true nor false since it depends upon c. (3 >
137 | c)
138 |
139 | Exercise 6
140 | -----------
141 |
142 | Simplify the following statements. Which variables are free and which are
143 | bound? If the statement has no free variables, say whether it is true or
144 | false.
145 | (a) w ∈ {x ∈ R | 13 − 2x > c}.
146 | (b) 4 ∈ {x ∈ R | 13 − 2x ∈ {y | y is a prime number}}. (It might make
147 | this statement easier to read if we let P = {y | y is a prime number};
148 | using this notation, we could rewrite the statement as 4 ∈ {x ∈ R |
149 | 13 − 2x ∈ P}.)
150 | (c) 4 ∈ {x ∈{y | y is a prime number} | 13 − 2x > 1}. (Using the same no-
151 | tation as in part (b), we could write this as 4 ∈ {x ∈ P | 13 − 2x > 1}.)
152 |
153 | (a)
154 |
155 | Free variables: w, c
156 | Bound variables: x
157 |
158 | (b)
159 |
160 | Thanks for the hints!
161 |
162 | let P = {y | y is a prime number}
163 | 4 ∈ {x ∈ R | 13 − 2x ∈ P}
164 |
165 | Bound variable: x, y
166 | Statement is true since 5 is a prime number.
167 |
168 | (c)
169 |
170 | let P = {y | y is a prime number}
171 | 4 ∈ {x ∈ P | 13 − 2x > 1}
172 |
173 | Bound variable: x
174 | Statement is false since 4 is not a prime number.
175 |
176 | Exercise 7
177 | ----------
178 |
179 | What are the truth sets of the following statements? List a few elements of
180 | the truth set if you can.
181 | (a) Elizabeth Taylor was once married to x.
182 | (b) x is a logical connective studied in Section 1.1.
183 | (c) x is the author of this book.
184 |
185 | (a)
186 |
187 | {x | Elizabeth Taylor was once married to x} = {Conrad Hilton Jr.,
188 | Michael Wilding, Michael Todd, Eddie Fisher, Richard Burton, John
189 | Warner, Larry Fortensky}
190 |
191 | Copied the husband names from the answers section since I didn't know them.
192 |
193 | (b)
194 |
195 | {x | x is a logical connective studied in section 1.1} = { ∧, ∨, ¬ }
196 |
197 | (c)
198 |
199 | {x | x is the author of this book} = {Daniel J Velleman}
200 |
201 | Exercise 8
202 | -----------
203 |
204 | What are the truth sets of the following statements? List a few elements of
205 | the truth set if you can.
206 | (a) x is a real number and x 2 − 4x + 3 = 0.
207 | (b) x is a real number and x 2 − 2x + 3 = 0.
208 | (c) x is a real number and 5 ∈ {y ∈ R | x 2 + y 2 < 50}.
209 |
210 | (a)
211 |
212 | {x ∈ R | x^2 − 4x + 3 = 0}
213 |
214 | (b)
215 |
216 | {x ∈ R | x^2 − 2x + 3 = 0}
217 |
218 | (c)
219 |
220 | {x ∈ R | 5 ∈ {y ∈ R | x^2 + y^2 < 50}}
221 | {x ∈ R | x^2 < 25}
222 |
--------------------------------------------------------------------------------
/chapter 1/chapter1-1.md:
--------------------------------------------------------------------------------
1 | Deductive Reasoning and Logical connectives
2 | ---------------------------------------------
3 | ---------------------------------------------
4 |
5 | Exercise 1
6 | -----------
7 |
8 | 1. Analyze the logical forms of the following statements:
9 |
10 | (a) We’ll have either a reading assignment or homework problems, but we
11 | won’t have both homework problems and a test.
12 |
13 | (b) You won’t go skiing, or you will and there won’t be any snow.
14 |
15 | (c) √7 ≤ 2.
16 |
17 | (a)
18 |
19 | P = We'll have reading assignment.
20 |
21 | Q = We'll have homework problems.
22 |
23 | T = We'll have test.
24 |
25 | (P ∨ Q) ∧ ¬(Q ∧ T)
26 |
27 | (b)
28 |
29 | P = You go skiing
30 |
31 | Q = There will be snow
32 |
33 | ¬P ∨ (P ∧ ¬Q)
34 |
35 | (c)
36 |
37 | √7 ≤ 2 (The question is not equal to.)
38 |
39 | P = √7 < 2
40 |
41 | Q = √7 = 2
42 |
43 | ¬(P ∨ Q)
44 |
45 | Exercise 2
46 | -----------
47 |
48 | 2. Analyze the logical forms of the following statements:
49 | (a) Either John and Bill are both telling the truth, or neither of them is.
50 | (b) I’ll have either fish or chicken, but I won’t have both fish and mashed
51 | potatoes.
52 | (c) 3 is a common divisor of 6, 9, and 15.
53 |
54 | (a)
55 |
56 | J = John is telling the truth
57 | B = Bill is telling the truth
58 |
59 | (J ∧ B) ∨ (¬J ∧ ¬B)
60 |
61 | (b)
62 |
63 | F = I'll have fish.
64 | C = I'll have chicken
65 | P = I'll have mashed potatoes
66 |
67 | (F ∨ C) ^ ¬(F ∧ P)
68 |
69 | (c)
70 |
71 | X = 3 is a common divisor of 6
72 | Y = 3 is a common divisor of 9
73 | Z = 3 is a common divisor of 15
74 |
75 | X ∧ Y ∧ Z
76 |
77 | Exercise 3
78 | -----------
79 |
80 | Analyze the logical forms of the following statements:
81 | (a) Alice and Bob are not both in the room.
82 | (b) Alice and Bob are both not in the room.
83 | (c) Either Alice or Bob is not in the room.
84 | (d) Neither Alice nor Bob is in the room.
85 |
86 | A = Alice is in the room.
87 | B = Bob is in the room.
88 |
89 | (a)
90 |
91 | A ∧ B = Alice and Bob are both in the room
92 |
93 | ¬(A ∧ B)
94 |
95 | (b)
96 |
97 | (¬A ∧ ¬B)
98 |
99 | (c)
100 |
101 | ¬A ∨ ¬B
102 |
103 | (d)
104 |
105 | (¬A ∧ ¬B)
106 |
107 | Exercise 4
108 | ----------
109 |
110 | Ofcourse these two doesn't make sense:
111 |
112 | (b) ¬(P, Q, ∧R).
113 | (d) (P ∧ Q)(P ∨ R).
114 |
115 | Exercise 5
116 | ----------
117 |
118 | Let P stand for the statement “I will buy the pants” and S for the statement
119 | “I will buy the shirt.” What English sentences are represented by the fol-
120 | lowing expressions?
121 | (a) ¬(P ∧ ¬S).
122 | (b) ¬P ∧ ¬S.
123 | (c) ¬P ∨ ¬S.
124 |
125 | (a)
126 |
127 | P ∧ ¬S = I will buy the Pant but not the shirt.
128 | = I will buy the Pant without the shirt.
129 |
130 | ¬(P ∧ ¬S) = I won't buy the Pant without the shirt.
131 |
132 | (b)
133 |
134 | ¬P ∧ ¬S = I will neither buy the pant nor the shirt.
135 |
136 | (c)
137 |
138 | ¬P ∨ ¬S = Either I won't buy the pant or won't buy the shirt.
139 |
140 | Exercise 6
141 | -----------
142 |
143 | Let S stand for the statement “Steve is happy” and G for “George is happy.”
144 | What English sentences are represented by the following expressions?
145 | (a) (S ∨ G) ∧ (¬S ∨ ¬G).
146 | (b) [S ∨ (G ∧ ¬S)] ∨ ¬G.
147 | (c) S ∨ [G ∧ (¬S ∨ ¬G)].
148 |
149 | The best way to solve this problem are by reducing the
150 | [expressions](http://math.stackexchange.com/questions/885946/simplifying-ambiguous-statements)
151 | as stated in the link.
152 |
153 | Exercise 7
154 | -----------
155 |
156 | Identify the premises and conclusions of the following deductive argu-
157 | ments and analyze their logical forms. Do you think the reasoning is valid?
158 | (Although you will have only your intuition to guide you in answering
159 | this last question, in the next section we will develop some techniques for
160 | determining the validity of arguments.)
161 | (a) Jane and Pete won’t both win the math prize. Pete will win either
162 | the math prize or the chemistry prize. Jane will win the math prize.
163 | Therefore, Pete will win the chemistry prize.
164 | (b) The main course will be either beef or fish. The vegetable will be either
165 | peas or corn. We will not have both fish as a main course and corn as a
166 | vegetable. Therefore, we will not have both beef as a main course and
167 | peas as a vegetable.
168 | (c) Either John or Bill is telling the truth. Either Sam or Bill is lying.
169 | Therefore, either John is telling the truth or Sam is lying.
170 | (d) Either sales will go up and the boss will be happy, or expenses will go
171 | up and the boss won’t be happy. Therefore, sales and expenses will not
172 | both go up.
173 |
174 | (a)
175 |
176 | J = Jane will win the math prize
177 | P = Pete will win the math prize
178 | C = Pete will win the chemistry prize.
179 |
180 | Premises:
181 |
182 | ¬ (J ∧ P)
183 | P ∨ C
184 | J
185 |
186 | Conclusion:
187 | C
188 |
189 | Reasoning is valid: Jane has won the math prize so Pete can't win the
190 | math prize. So he can only will the chemistry prize according to the
191 | premises.
192 |
193 | (b)
194 |
195 | B = Beef will be the main course.
196 | F = Fish will be the main course.
197 | P = Peas will be the vegetable.
198 | C = Corn will be the vegetable.
199 |
200 | Premises:
201 | B ∨ F
202 | P ∨ C
203 | ¬(F ∧ C)
204 |
205 | Conclusion:
206 | ¬(B ∧ P)
207 |
208 | Reasoning seems invalid: In a case where you cannot have both fish and
209 | corn, you will end up with Beef and Peas.
210 |
211 | (c)
212 |
213 | J = John is telling the truth.
214 | B = Bill is telling the truth.
215 | S = Sam is telling the truth.
216 |
217 | Premises:
218 | J ∨ B
219 | ¬S ∨ ¬B
220 |
221 | Conclusion:
222 | J ∨ ¬S
223 |
224 | Reasoning seems valid: J ∨ B ∨ ¬S ∨ ¬B gives J ∨ ¬S
225 |
226 | (d) Either sales will go up and the boss will be happy, or expenses will go
227 | up and the boss won’t be happy. Therefore, sales and expenses will not
228 | both go up.
229 |
230 | S = Sales will go up.
231 | B = Boss will be happy.
232 | E = Expenses will go up.
233 |
234 | Premises:
235 | (S ∧ B) ∨ (E ∧ ¬B)
236 |
237 | Conclusion:
238 | ¬(S ∧ E)
239 |
240 | Reasoning is invalid: When S = T, E = T, B = T, them premise is True
241 | and Conclusion is false.
242 |
--------------------------------------------------------------------------------
/chapter 1/section1-4.md:
--------------------------------------------------------------------------------
1 | Operations on Sets
2 | -------------------
3 | -------------------
4 |
5 | Exercise 1
6 | -----------
7 |
8 | Let A = {1, 3, 12, 35}, B = {3, 7, 12, 20}, and C = {x | x is a prime
9 | number}. List the elements of the following sets. Are any of the sets
10 | below disjoint from any of the others? Are any of the sets below subsets
11 | of any others?
12 | (a) A ∩ B.
13 | (b) (A ∪ B) \ C.
14 | (c) A ∪ (B \ C).
15 |
16 | (a)
17 |
18 | A ∩ B = {3,12}
19 |
20 | (b)
21 |
22 | (A ∪ B) = {1,3,7,12,20,35}
23 | (A ∪ B) \ C = {1,12,20,35}
24 |
25 | (c)
26 |
27 | (B \ C) = {12,20}
28 | A ∪ (B \ C) = {1,3,12,20,35}
29 |
30 | No, two sets seem to be disjoint.
31 | Two sets are subsets:
32 |
33 | (A ∩ B) ⊆ (A ∪ (B \ C))
34 | ((A ∪ B) \ C) ⊆ A ∪ (B \ C)
35 |
36 | Exercise 2
37 | ----------
38 |
39 | Let A = {United States, Germany, China, Australia}, B = {Germany,
40 | France, India, Brazil}, and C = {x | x is a country in Europe}. List the
41 | elements of the following sets. Are any of the sets below disjoint from
42 | any of the others? Are any of the sets below subsets of any others?
43 | (a) A ∪ B.
44 | (b) (A ∩ B) \ C.
45 | (c) (B ∩ C) \ A.
46 |
47 | (a)
48 |
49 | A ∪ B = {United States, Germany, China, Australia, India, Brazil,
50 | France}
51 |
52 | (b)
53 |
54 | (A ∩ B) = {Germany}
55 | (A ∩ B) \ C = { }
56 |
57 | (c)
58 |
59 | (B ∩ C) = {Germany, France}
60 | (B ∩ C) \ A = {France}
61 |
62 | Any set along with `(A ∩ B) \ C` forms a disjoint set. Also, `(B ∩ C)
63 | \ A ⊆ A ∪ B`
64 |
65 | Exercise 3
66 | -----------
67 |
68 |
69 |
70 | Exercise 3 & 4 & 6
71 | -------------------
72 |
73 | Probably I will learn some graphical package to display this answer.
74 |
75 | Exercise 5
76 | -----------
77 |
78 | (a) A \ (A ∩ B) = A \ B.
79 | (b) A ∪ (B ∩ C) = (A ∪ B) ∩ (A ∪ C).
80 |
81 | (a)
82 |
83 | x ∈ A \ (A ∩ B)
84 | = x ∈ A ∧ ¬(x ∈ (A ∩ B))
85 | = x ∈ A ∧ ¬(x ∈ A ∧ x ∈ B)
86 | = x ∈ A ∧ (x ∈ ¬A ∨ x ∈ ¬B)
87 | = (x ∈ A ∧ x ∈ ¬A) ∨ (x ∈ A ∧ x ∈ ¬B)
88 | = (x ∈ A ∧ x ∈ ¬B)
89 | = A \ B
90 |
91 | (b)
92 |
93 | x ∈ (A ∪ (B ∩ C))
94 | = (x ∈ A) ∨ (x ∈ (B ∩ C))
95 | = (x ∈ A) ∨ (x ∈ B ∧ x ∈ C)
96 | = (x ∈ A ∨ x ∈ B) ∧ (x ∈ A ∨ x ∈ C)
97 | = (A ∪ B) ∩ (A ∪ C)
98 |
99 | Exercise 7
100 | -----------
101 |
102 | (a) (A ∪ B) \ C = (A \ C) ∪ (B \ C).
103 | (b) A ∪ (B \ C) = (A ∪ B) \ (C \ A).
104 |
105 | (a)
106 |
107 | x ∈ (A ∪ B) \ C
108 | = x ∈ (A ∪ B) ∧ x ∈ ¬C
109 | = (x ∈ A ∨ x ∈ B) ∧ x ∈ ¬C
110 | = x ∈ ¬C ∧ (x ∈ A ∨ x ∈ B)
111 | = (x ∈ ¬C ∧ x ∈ A) ∨ (x ∈ ¬C ∧ x ∈ B)
112 | = (A \ C) ∪ (B \ C)
113 |
114 | (b)
115 |
116 | x ∈ A ∪ (B \ C)
117 | = x ∈ A ∨ (x ∈ B \ C)
118 | = x ∈ A ∨ (x ∈ B ∧ x ∈ ¬C)
119 | = (x ∈ A ∨ x ∈ B) ∧ (x ∈ A ∨ x ∈ ¬C)
120 | = (x ∈ A ∨ x ∈ B) ∧ ¬(x ∈ ¬A ∧ x ∈ C)
121 | = (A ∪ B) \ (C \ A)
122 |
123 | Exercise 8
124 | -----------
125 |
126 | For each of the following sets, write out (using logical symbols) what it
127 | means for an object x to be an element of the set. Then determine which
128 | of these sets must be equal to each other by determining which statements
129 | are equivalent.
130 | (a) (A \ B) \ C.
131 | (b) A \ (B \ C).
132 | (c) (A \ B) ∪ (A ∩ C).
133 | (d) (A \ B) ∩ (A \ C).
134 | (e) A \ (B ∪ C)
135 |
136 | (a)
137 |
138 | x ∈ (A \ B) \ C
139 | = x ∈ (A \ B) ∧ ¬(x ∈ C)
140 | = x ∈ A ∧ ¬(x ∈ B) ∧ ¬(x ∈ C)
141 | = x ∈ A ∧ x ∈ ¬B ∧ x ∈ ¬C
142 |
143 | (b)
144 |
145 | x ∈ A \ (B \ C)
146 | = x ∈ A ∧ ¬(x ∈ B \ C)
147 | = x ∈ A ∧ ¬(x ∈ B ∧ x ∈ ¬C)
148 | = x ∈ A ∧ (x ∈ ¬B ∨ x ∈ C)
149 | = (x ∈ A ∧ x ∈ ¬B) ∨ (x ∈ A ∧ x ∈ C)
150 |
151 | (c)
152 |
153 | x ∈ (A \ B) ∪ (A ∩ C)
154 | = x ∈ (A \ B) ∨ x ∈ (A ∩ C)
155 | = (x ∈ A ∧ x ∈ ¬B) ∨ (x ∈ A ∧ x ∈ C)
156 |
157 | (d)
158 |
159 | x ∈ (A \ B) ∩ (A \ C)
160 | = x ∈ (A \ B) ∧ x ∈ (A \ C)
161 | = x ∈ A ∧ x ∈ ¬B ∧ x ∈ A ∧ x ∈ ¬C
162 | = x ∈ A ∧ x ∈ ¬B ∧ x ∈ ¬C
163 |
164 | (e)
165 |
166 | x ∈ A \ (B ∪ C)
167 | = x ∈ A ∧ ¬(x ∈ (B ∪ C))
168 | = x ∈ A ∧ ¬(x ∈ B ∨ x ∈ C)
169 | = x ∈ A ∧ x ∈ ¬B ∧ x ∈ ¬C
170 |
171 | From the above: (a), (d) & (e) are equivalent. Also, (b) & (c) are
172 | equivalent.
173 |
174 | Exercise 9
175 | -----------
176 |
177 | It was shown in this section that for any sets A and B, (A ∪ B) \ B ⊆ A.
178 | Give an example of two sets A and B for which (A ∪ B) \ B =/ A.
179 | (not equal to.)
180 |
181 | First some logical analysis:
182 |
183 | x ∈ (A ∪ B) \ B
184 | x ∈ (A ∪ B) ∧ x ∈ ¬B
185 | (x ∈ A ∨ x ∈ B) ∧ x ∈ ¬B
186 | x ∈ ¬B ∧ (x ∈ A ∨ x ∈ B)
187 | (x ∈ ¬B ∧ x ∈ A ) ∨ (x ∈ ¬B ∧ x ∈ B)
188 | (x ∈ ¬B ∧ x ∈ A ) ∨ (False ∧ True)[ or True ∧ False]
189 | (x ∈ ¬B ∧ x ∈ A ) ∨ (False)
190 | x ∈ ¬B ∧ x ∈ A
191 |
192 | B = {2,4}
193 | A = {1,2}
194 | A ∪ B = {1,2,4}
195 |
196 | (A ∪ B) \ B = {1}
197 |
198 | So, the key thing to figure out here is that if set B contains any
199 | elements which is present in set `A`, then `(A ∪ B) \ B` is not equal
200 | to `A`.
201 |
202 | Exercise 11
203 | -----------
204 |
205 | (b) Give an example of sets A, B, and C for which (A ∪ B) \ C ≠ A ∪ (B \ C).
206 |
207 | A = {1, 2}
208 | B = {1, 2}
209 | C = {1, 2}
210 |
211 | (A ∪ B) \ C = ({1, 2} ∪ {1, 2}) \ {1, 2}
212 | = {1, 2} \ {1, 2}
213 | = {} or ∅
214 |
215 | A ∪ (B \ C) = {1, 2} ∪ ({1, 2} \ {1, 2})
216 | = {1, 2} ∪ {}
217 | = {1, 2}
218 |
219 | Hence, shown that (A ∪ B) \ C ≠ A ∪ (B \ C)
220 |
221 | Exercise 13
222 | -----------
223 |
224 | Use any method you wish to verify the following identities:
225 | (a) (A ▵ B) ∪ C = (A ∪ C) ▵ (B \ C).
226 | (b) (A ▵ B) ∩ C = (A ∩ C) ▵ (B ∩ C).
227 | (c) (A ▵ B) \ C = (A \ C) ▵ (B \ C)
228 |
229 | (a)
230 |
231 | A ▵ B = (A \ B) ∪ (B \ A) = (A ∪ B) \ (A ∩ B)
232 |
233 | (A ▵ B) ∪ C
234 | = x ∈ (A ▵ B) ∪ C
235 | = x ∈ (A \ B) ∪ (B \ A) ∪ C
236 | = (x ∈ A ∧ x ∈ ¬B) ∨ (x ∈ B ∧ x ∈ ¬A) ∨ x ∈ C
237 | = ((x ∈ A ∧ x ∈ ¬B) ∨ (x ∈ B)) ∧ ((x ∈ A ∧ x ∈ ¬B) ∨ (x ∈ ¬A)) ∨ x ∈ C
238 | = ((x ∈ A ∨ x ∈ B) ∧ (x ∈ ¬B ∨ x ∈ ¬A)) ∨ x ∈ C
239 | = (x ∈ C ∨ (x ∈ A ∨ x ∈ B)) ∧ (x ∈ C ∨ (x ∈ ¬B ∨ x ∈ ¬A))
240 | = ((x ∈ A ∨ x ∈ C) ∨ x ∈ B) ∧ (x ∈ ¬B ∨ x ∈ C ∨ x ∈ ¬A)
241 | = (x ∈ (A ∪ C) ∨ x ∈ B) ∧ (¬(x ∈ B ∧ x ∈ ¬C) ∨ x ∈ ¬A)
242 | = (x ∈ (A ∪ C) ∨ x ∈ B) ∧ (x ∈ (B \ C) ∨ x ∈ ¬A)
243 | = (x ∈ (A ∪ C) ∧ (x ∈ (B \ C) ∨ x ∈ ¬A)) ∨ (x ∈ B ∧ (x ∈ (B \ C) ∨ x ∈
244 | ¬A))
245 | = (x ∈ (A ∪ C) ∧ (B \ C)) ∨ (x ∈ (A ∪ C) ∧ x ∈ ¬A))
246 |
--------------------------------------------------------------------------------
/chapter 3/section3.2.tex:
--------------------------------------------------------------------------------
1 | %%%%%%%%%%%%%%%%%%%%%%%%%%%%%%%%%%%%%%%%%
2 | % Author: Sibi
3 | %%%%%%%%%%%%%%%%%%%%%%%%%%%%%%%%%%%%%%%%%
4 | \documentclass{article}
5 | \usepackage{graphicx}
6 | \usepackage{verbatim}
7 | \usepackage{amsmath}
8 | \usepackage{amsfonts}
9 | \usepackage{amssymb}
10 | \usepackage{tabularx}
11 | \setlength\parskip{\baselineskip}
12 | \begin{document}
13 | \title{Chapter 3 (Section 3.2)}
14 | \author{Sibi}
15 | \date{\today}
16 | \maketitle
17 | \newpage
18 |
19 | \section{Problem 1}
20 |
21 | Solution (a)
22 |
23 | %% +--------------+--------------+
24 | %% |Givens |Goals |
25 | %% +--------------+--------------+
26 | %% |P \implies Q |P \implies R |
27 | %% | | |
28 | %% |Q \implies R | |
29 | %% +--------------+--------------+
30 | % This LaTeX table template is generated by emacs 24.3.1
31 | \begin{tabular}{| >{$}l<{$} | >{$}l<{$} |}
32 | \hline
33 | Givens & Goals \\
34 | \hline
35 | P \implies Q & P \implies R \\
36 | & \\
37 | Q \implies R & \\
38 | \hline
39 | \end{tabular}
40 |
41 |
42 | %% +--------------+--------------+
43 | %% |Givens |Goals |
44 | %% +--------------+--------------+
45 | %% |P \implies Q |R |
46 | %% | | |
47 | %% |Q \implies R | |
48 | %% | | |
49 | %% |P | |
50 | %% +--------------+--------------+
51 | % This LaTeX table template is generated by emacs 24.3.1
52 | \begin{tabular}{| >{$}l<{$} | >{$}l<{$} |}
53 | \hline
54 | Givens & Goals \\
55 | \hline
56 | P \implies Q & R \\
57 | & \\
58 | Q \implies R & \\
59 | & \\
60 | P & \\
61 | \hline
62 | \end{tabular}
63 |
64 | Proof Structure
65 |
66 | Suppose $P$.
67 |
68 | [Proof of R goes here]
69 |
70 | Therefore $P \implies R$.
71 |
72 | Proof. Suppose $P$. Since $P \implies Q$, using modus ponens we can
73 | conclude $Q$. Since $Q \implies R$, again from modus ponens we can
74 | conclude R. Therefore, $P \implies R$.
75 |
76 | Solution (b)
77 |
78 | %% +--------------------+--------------+
79 | %% |Givens |Goals |
80 | %% +--------------------+--------------+
81 | %% |\neg R \implies (P |P \implies (Q |
82 | %% |\implies \neg Q) |\implies R) |
83 | %% +--------------------+--------------+
84 | % This LaTeX table template is generated by emacs 24.3.1
85 | \begin{tabular}{| >{$}l<{$} | >{$}l<{$} |}
86 | \hline
87 | Givens & Goals \\
88 | \hline
89 | \neg R \implies (P & P \implies (Q \\
90 | \implies \neg Q) & \implies R) \\
91 | \hline
92 | \end{tabular}
93 |
94 | $ \neg R \implies (P \implies \neg Q)$ is equivalent to $(P \land Q)
95 | \implies R$
96 |
97 | Proof. Suppose P. Suppose Q. Since $ \neg R \implies (P \implies \neg
98 | Q) $ is equivalent to $(P \land Q) \implies R$, we can conclude R.
99 | Therefore, $P \implies (Q \implies R)$.
100 |
101 | \section{Problem 2}
102 |
103 | Solution(a)
104 |
105 | Proof. Suppose P. Since $P \implies Q$, we can conclude Q. Since $R
106 | \implies \neg Q$ is equivalent to $Q \implies \neg R$, we can conclude
107 | $\neg R$ as Q is true. Therefore, $P \implies \neg R$ is true.
108 |
109 | Solution (b)
110 |
111 | Proof. Suppose Q. We will prove by contradiction. Let $Q \implies \neg
112 | P$ be true. But $\neg P$ is true but our given $P$ is also true which
113 | leads us to a contradiction. Therefore, $\neg(Q \implies \neg P)$ is
114 | true.
115 |
116 | Alt. Proof: Suppose $Q$. Then $\neg (Q \implies \neg P) \equiv Q \land P$,
117 | because $\neg (Q \implies \neg P) \equiv \neg ( \neg Q \lor \neg P )$.
118 | Since we are given P and assumed Q, then $Q \land P$ is true,
119 | therefore if $P$ and $Q$ are true then $Q \implies \neg (Q \implies \neg P)$
120 | is also true.
121 |
122 | \section{Problem 3}
123 |
124 | Proof. Suppse $x \in A$. Since $A \subseteq C$, it follows $x \in C$.
125 | But B and C are disjoint, so $x \notin B$.
126 |
127 | Expanded Proof. Suppose $x \in A$. Since $A \subseteq C$ then it holds that
128 | $x \in A \implies x \in C$, so $x \in C$. Since $B \cap C = \emptyset$ then
129 | $x \in C \implies x \notin B$, therefore $x \notin B$.
130 |
131 | \section{Problem 4}
132 |
133 | Proof. Suppose $x \in C$. We can conclude that $x \notin A \setminus
134 | B$. From, $x \notin A \setminus B$, it follows that $x \in B$.
135 | Therefore, if $x \in C$, then $x \in B$.
136 |
137 | \section{Problem 5}
138 |
139 | Proof. (by contradiction) Suppose $a \in A \setminus B$.
140 | It follows that $a \in A$ from our assumption. Also,
141 | $a \in C$ from which we can conclude that $a \in B$ since $A \cap C
142 | \subseteq B$ But this contradicts the fact that $a \notin B$.
143 | Therefore $a \notin A \setminus B$.
144 |
145 | \section{Problem 6}
146 |
147 | Proof. (by contradiction) Suppose $a \notin C$. From $a
148 | \notin B \setminus C$, we can conclude that either $a \notin B$ or $a
149 | \in C$ is true. Since $a \in A$, we can conclude that $a \in B$ from
150 | $A \subseteq B$. So, $a \notin B$ is false. Therefore, $a \in C$
151 | should be true. But this contradicts the fact that $a \notin C$.
152 | Therefore $a \in C$.
153 |
154 | Note: This can be proven by focusing on $a \in B$ as well. By assuming
155 | $a \notin C$ we know that $a \notin B$ by $a \notin B \setminus C$, since this
156 | means logically that $\neg (a \in B \land a \notin C)$ which simplifies
157 | to $a \notin B \lor a \in C$. By assuming $a \notin C$ we automatically
158 | must have $a \notin B$. But since we have $a \in A$ then by the subset
159 | $A \subseteq B$ we have $\forall x ( x \in A \implies x \in B)$ and
160 | therefore $a \in A$ is sufficient to force $a \in B$. But this
161 | contradicts that $a$ cannot be in $B$ since we assumed $a \notin C$.
162 | Since $a \notin C$ causes an impossible situation, $a$ must be in $C$.
163 |
164 | \section{Problem 7}
165 |
166 | Proof. (by contradiction) Suppose $y = 0$. Solving $y +
167 | x = 2y - x$ by substituting $y=0$ we get that $x=0$. But it
168 | contradicts the fact that both $x$ and $y$ cannot be $0$. Therefore,
169 | $y \neq 0$.
170 |
171 | \section{Problem 8}
172 |
173 | Proof. Suppose $a < 1/a < b < 1/b$. Multiplying the inequality by
174 | $1/a < 1/b$ by $ab$ we should get $a < b$. From this, we can conclude
175 | that $ab$ is a negative number. Since $a < b$, we can conclude that
176 | $a$ is the negative number. Multiplying inequality $a < 1/a$ by
177 | negative number we get $a^2 < 1$. From that we can conclude that $a <
178 | -1$. Therefore, if $a < 1/a < b < 1/b$, then $a < -1$.
179 |
180 | \section{Problem 9}
181 |
182 | Proof. (by contradiction) Suppose $x^2y = 2x + y$. Suppose $y \neq 0$ Suppose $x = 0$.
183 | Substituting this into the equivalent $x^2y = 2x + y$, we get $y = 0$.
184 | But this contradicts the fact that $y \neq 0$. Therefore $x \neq 0$.
185 | Thus, if $y \neq 0$, then $x \neq 0$.
186 |
187 | \section{Problem 10}
188 |
189 | Proof. Suppose $x \neq 0$. Suppose $y = 3x^2 + 2y / x^2 + 2$. Solving
190 | it we get $x^2(3 - y) = 0$. Since $x \neq 0$, we can conclude that $y
191 | = 3$.
192 |
193 | \section{Problem 11}
194 |
195 | Solution (a)
196 |
197 | If conclusion of the theorem is false, then $\neg( x \neq 3 \land y
198 | \neq 8)$. That is equivalent to $x = 3 \lor y = 8$.
199 |
200 | Solution (b)
201 |
202 | First example: $x = 3$ and $y = 7$.
203 | Second example: $x = 2$ and $y = 8$.
204 |
205 | \section{Problem 12}
206 |
207 | Solution (a)
208 |
209 | This is plain wrong: Since $x \notin B$ and $B \subseteq C$, $x \notin
210 | C$.
211 |
212 | From $x \notin B$ and $B \subseteq C$, we cannot conclude that $x
213 | \notin C$.
214 |
215 | Expansion:
216 |
217 | $A \subseteq C$ means $\forall x ( x \in A \implies x \in B )$. Likewise,
218 | $B \subseteq C$ means $\forall x ( x \in B \implies x \in C )$. We are given
219 | that $x \in A$, therefore we know $x \in C$ by the first given. Now if we
220 | claim that $x \notin C$ because $x \notin B$ then we are committing the
221 | fallacy of denying the hypothesis, or trying to accept the inverse of
222 | $x \in B \implies x \in C$. This is clearly wrong, therefore the proof
223 | is invalid.
224 |
225 | Solution (b)
226 |
227 | $ C = \{1,2,3,4,5\} $
228 | $ A = \{1,2\}$
229 | $ B = \{3,4,5\}$
230 | $ x = 1 $
231 | \end{document}
232 |
--------------------------------------------------------------------------------
/chapter 4/section4-1.tex:
--------------------------------------------------------------------------------
1 | %%%%%%%%%%%%%%%%%%%%%%%%%%%%%%%%%%%%%%%%%
2 | % Author: Sibi
3 | %%%%%%%%%%%%%%%%%%%%%%%%%%%%%%%%%%%%%%%%%
4 | \documentclass{article}
5 | \usepackage{graphicx}
6 | \usepackage{verbatim}
7 | \usepackage{amsmath}
8 | \usepackage{amsfonts}
9 | \usepackage{amssymb}
10 | \usepackage{tabularx}
11 | \setlength\parskip{\baselineskip}
12 | \begin{document}
13 | \title{Chapter 4 (Section 4.1)}
14 | \author{Sibi}
15 | \date{\today}
16 | \maketitle
17 | \newpage
18 |
19 | \section{Problem 1}
20 |
21 | Solution (a)
22 |
23 | $\{(x,y) \in P \times P \mid x \text{ is a parent of } y \} = \{(Prabakaran, Sibi) ..
24 | (Prabakaran, Madhu)\}$
25 |
26 | Solution (b)
27 |
28 | $\{(x,y) \in C \times U \mid \text{Someone lives in } x \text{ and
29 | attends } y \}$
30 |
31 | \section{Problem 2}
32 |
33 | Solution (a)
34 |
35 | $\{(x,y) \in P \times C \mid x \text{ lives in } y \} = \{(Sibi, Chennai) .. \}$
36 |
37 | Solution (b)
38 |
39 | $\{(x,y) \in C \times N \mid \text{Population of } x \text{ is } y \} = \{(Chennai, 3434343)\}$
40 |
41 | \section{Problem 3}
42 |
43 | Solution (a)
44 |
45 | $\{(0,-2), (1,-2), ..\}$
46 |
47 | Solution (b)
48 |
49 | $\{(1,0), (2,0), (3,0), ..\}$
50 |
51 | Solution (c)
52 |
53 | $\{(0,-2), (1,-2), ..\}$
54 |
55 | Solution (d)
56 |
57 | $\{(2,0), (0, -2), ..\}$
58 |
59 | \section{Problem 4}
60 |
61 | \begin{align*}
62 | A = \{1,2,3\} \\
63 | B = \{1,4\} \\
64 | C = \{3,4\} \\
65 | D = \{5\}
66 | \end{align*}
67 |
68 | Solution(1)
69 |
70 | \begin{align*}
71 | B \cap C = \{4\} \\
72 | A \times (B \cap C) = \{(1,4),(2,4),(3,4)\} \\ \\
73 | A \times B = \{(1,1),(1,4),(2,1),(2,4),(3,1),(3,4)\} \\
74 | A \times C = \{(1,3),(1,4),(2,3),(2,4),(3,3),(3,4)\} \\
75 | (A \times B) \cap (A \times C) = \{(1,4),(2,4),(3,4)\}
76 | \end{align*}
77 |
78 | Solution(2)
79 |
80 | \begin{align*}
81 | B \cup C = \{1,3,4\} \\
82 | A \times (B \cup C) =
83 | \{(1,1),(1,4),(1,3),(2,1),(2,4),(2,3),(3,1),(3,4),(3,3)\} \\
84 | (A \times B) \cup (A \times C) =
85 | \{(1,1),(1,4),(2,1),(2,4),(3,1),(3,4),(1,3),(2,3),(3,3)\} \\
86 | \end{align*}
87 |
88 | Solution(3)
89 |
90 | \begin{align*}
91 | A \times B = \{(1,1),(1,4),(2,1),(2,4),(3,1),(3,4)\} \\
92 | C \times D = \{(3,5),(4,5)\} \\ \\
93 | (A \times B) \cap (C \times D) = \emptyset \\
94 | A \cap C = \{3\} \\
95 | B \cap D = \emptyset \\
96 | (A \cap C) \times (B \cap D) = \emptyset
97 | \end{align*}
98 |
99 | Solution(4)
100 |
101 | \begin{align*}
102 | A \times B = \{(1,1),(1,4),(2,1),(2,4),(3,1),(3,4)\} \\
103 | C \times D = \{(3,5),(4,5)\} \\
104 | (A \times B) \cup (C \times D) =
105 | \{(1,1),(1,4),(2,1),(2,4),(3,1),(3,4),(3,5),(4,5)\} \\ \\
106 | A \cup C = \{1,2,3,4\} \\
107 | B \cup D = \{1,4,5\}\\
108 | (A \cup C) \times (B \cup D) = \{(1,1),(1,4),(1,5),(2,1),(2,4),(2,5),(3,1),(3,4),(3,5),(4,1),(4,4),(4,5)\}
109 | \end{align*}
110 |
111 | \section{Problem 5}
112 |
113 | Solution(a)
114 |
115 | ($ \Leftarrow $) Let $p$ be an arbitrary element of $A \times (B \cup C)$. Then by the
116 | definition of cartesian product, $p = (x,y)$ for some $x \in A$ and $y
117 | \in B \cup C$. Let us consider the cases:
118 |
119 | Case 1. $y \in B$. Since $x \in A$ and $y \in B$, it follows that $p
120 | \in A \times B$. So, $p \in (A \times B) \cup (A \times C)$.
121 |
122 | Case 2. $y \in C$. Since $x \in A$, it follows that $p \in A \times
123 | C$. So, $p \in (A \times B) \cup (A \times C)$.
124 |
125 | ($ \Rightarrow $) Let $p$ be an arbitrary element of $(A \times B)
126 | \cup (A \times C)$. Let us consider the cases:
127 |
128 | Case 1. $p \in A \times B$ Then there is some element $x$ and $y$ such
129 | that $p = (x,y)$ and $x \in A$ and $y \in B$. From $y \in B$, it
130 | follows that $y \in B \cup C$. So, $p \in A \times (B \cup C)$.
131 |
132 | Case 2. $p \in A \times C$ Then by definition of cartesian product, $p
133 | = (x,y)$ for some $x \in A$ and $y \in C$. From $y \in C$, it follows
134 | that $y \in B \cup C$. So, $p \in A \times (B \cup C)$.
135 |
136 | Solution(b)
137 |
138 | ($ \Leftarrow $) Let $p$ be an arbitrary element in $(A \times B) \cap
139 | (C \times D)$. Then it follows that $p \in A \times B$ and $p \in C
140 | \times D$. By definition of cartesian product, $p = (x,y)$ for some $x
141 | \in A$ and $y \in B$. Similarly, $x \in C$ and $y \in D$. It follows
142 | that $x \in (A \cap C)$ and $y \in (B \cap D)$. So, $(x,y) \in (A \cap
143 | C) \times (B \cap D)$. Since $p$ is an arbitrary element, $(A \times
144 | B) \cap (C \times D) \subseteq (A \cap C) \times (B \cap D)$.
145 |
146 | ($ \Rightarrow $) Let $p$ be an arbitrary element in $(A \cap C)
147 | \times (B \cap D)$. By the definition of cartesian product it follows
148 | that $p = (x,y)$ for some $x \in A \cap C$ and $y \in (B \cap D)$.
149 | From $x \in A \cap C$, we can conclude that $x \in A$ and $x \in C$.
150 | Similarly, $y \in B$ and $y \in D$. So, $(x,y) \in (A \times B) \cap
151 | (C \times D)$. Since p is arbitrary, $(A \cap C) \times (B \cap D)
152 | \subseteq (A \times B) \cap (C \times D)$.
153 |
154 | \section{Problem 6}
155 |
156 | The cases are not exhaustive. What about when $x \in A$ and $y \in D$,
157 | huh ?
158 |
159 | \section{Problem 7}
160 |
161 | $ m * n$
162 |
163 | \section{Problem 8}
164 |
165 | ($ \Leftarrow $) Let $p$ be an arbitrary element in $A \times (B
166 | \setminus C)$. From the definition of cartesian product it follows
167 | that $p = (x,y)$ for some $x \in A$ and $y \in B \setminus C$. From $y
168 | \in B \setminus C$, it follows that $y \in B$ and $y \notin C$.
169 | Therefore $(x,y) \in (A \times B)$. Similarly, $(x,y) \in A \times C$.
170 | Since $p$ is arbitrary, we can conclude that $A \times (B \setminus C)
171 | \subseteq (A \times B) \setminus (A \times C)$.
172 |
173 | ($ \Rightarrow $) Let $p$ be an arbitrary element in $(A \times B)
174 | \setminus (A \times C)$. It follows that $p \in (A \times B)$ and $p
175 | \notin (A \times C)$. From the definition of cartesian product it
176 | follows that $p = (x,y)$ for some $x \in A$ and $y \in B$. Similarly
177 | $(x,y) \notin (A \times C)$. But we know that $x \in A$, so $y \notin
178 | C$. Therefore, $y \in B \setminus C$. So, $(x,y) \in A \times (B
179 | \setminus C)$. Since $p$ is arbitrary, we can conclude that $A \times
180 | (A \setminus C) = (A \times B) \setminus (A \times C)$.
181 |
182 | \section{Problem 9}
183 |
184 | ($ \Leftarrow $) Let $p$ be an arbitrary element in $(A \times B)
185 | \setminus (C \times D)$. It follows that $p \in A \times B$ and $p
186 | \notin (C \times D)$. From the definition of cartesian product, it
187 | follows that $p = (x,y)$ for some $x \in A$ and $y \in B$. Similarly,
188 | $(x,y) \notin (C \times D)$ which means either $x \notin c$ or $y
189 | \notin D$. Let us consider the cases separately:
190 |
191 | Case 1. $y \notin D$. We already know that $x \in A$ and $y \in B$.
192 | So, $y \in B \setminus D$. Therefore $(x,y) \in (A \times (B \setminus
193 | D))$. So, $p \in (A \times (B \setminus D)) \cup ((A \setminus C) \times B)$.
194 |
195 | Case 2. $x \notin C$. From earlier, $x \in A$ and $y\in B$. It follows
196 | that $x \in A \setminus C$. Therefore, $(x,y) \in ((A \setminus c)
197 | \times B)$. So, $p \in (A \times (B \setminus D)) \cup ((A \setminus
198 | C) \times B)$.
199 |
200 | ($ \Rightarrow $) Let $p$ be an arbitrary element in $(A \times (B
201 | \setminus D)) \cup ((A \setminus C) \times B)$. Let us consider the
202 | cases separately:
203 |
204 | Case 1. $p \in A \times (B \setminus D)$. From the definition of
205 | cartesian product, it follows that $p = (x,y)$ for some $x \in A$ and
206 | $y \in B \setminus D$. Therefore $y \in B$ and $y \notin D$. So, $p
207 | \in (A \times B)$ and since $ y \notin D$, it follows that $(x,y)
208 | \notin C \times D$. So, $p \in (A \times B) \setminus (C \times D)$.
209 |
210 | Case 2. $p \in ((A \setminus C) \times B)$ From the definition of
211 | cartesian product, it follows that $p = (x,y)$ for some $x \in A
212 | \setminus C$ and $y \in B$. From $x \in A \setminus C$, it follows
213 | that $x \in A$ and $x \notin C$. So, $p \in (A \times B)$ and $p
214 | \notin C \times D$ since $x \notin C$. So, $p \in (A \times B)
215 | \setminus (C \times D)$.
216 |
217 | \section{Problem 10}
218 |
219 | Suppose $A \times B$ and $C \times D$ are disjoint. Let $p$ be an
220 | arbitrary element in $A \times B$. Since they are disjoint, $p \notin
221 | C \times D$. From the definition of cartesian product, it follows that
222 | $p = (x,y)$ for some $x \in A$ and $y \in B$. Now since $(x,y) \in C
223 | \times D$, so either $x \notin C$ or $y \notin D$. So, if $x \notin
224 | C$, then $A \cap C = \emptyset$ or if $y \notin D$ then $B \cap D = \emptyset$.
225 |
226 | \section{Problem 11}
227 |
228 | Solution(a)
229 |
230 | Let $p$ be an arbitrary element in $\cup_{i \in I} (A_i \times B_i)$.
231 | It follows that $\exists i \in I(p \in A_i \times B_i)$. So $p \in A_i
232 | \times B_i$ for some $i$. From the definition of cartesian product, it
233 | follows that $p = (x,y)$ for some $x \in A_i$ and $y \in B_i$. From $x
234 | \in A_i$, we can conclude that $x \in \cup_{i \in I} A_i$. Similarly,
235 | $y \in \cup_{i \in I} B_i$. Therefore $p \in (\cup_{i \in I}A_i \times
236 | \cup_{i \in I}B_i)$. Since $p$ is an arbitrary, we can conclude that
237 | $\cup_{i \in I}(A_i \times B_i) \subseteq \cup_{i \in I})A_i \times
238 | \cup_{i \in I} B_i$.
239 |
240 | Solution (b)
241 |
242 | ($\Rightarrow$) Let $h$ be an arbitrary element in $\cup_{p in P}C_p$.
243 | It follows that $\exists p \in P(h \in C_p)$. So, $h \in A_i \times
244 | B_j$. From the definition of cartesian product, it follows that $h =
245 | (x,y)$ for some $x \in A_i$ and $y \in B_j$. From $x \in A_i$, we can
246 | conclude that $x \in \cup_{i \in I}A_i$. Similarly, $y \in \cup_{i \in
247 | I}B_i$. Therefore $h \in (\cup_{i \in I}A_i) \times (\cup_{i \in I}B_i)$.
248 |
249 | ($\Leftarrow$) Let $h$ be an arbitrary element in $(\cup_{i \in I}A_i)
250 | \times (\cup_{i \in I}B_i)$. Then by the definition of cartesian
251 | product, it follows that $h = (x,y)$ for some $x \in \cup_{i \in
252 | I}A_i$ and $y \in \cup_{j \in I}B_j$. From $x \in \cup_{i \in
253 | I}A_i$, it follows that for some $i$ in $I$, $x \in A_i$. Similarly,
254 | $y \in B_j$. Therefore $(x,y) \in A_i \times B_j$. So, $(x,y) \in
255 | C_{(i,j)}$ for some $i$ and $j$. Therefore $h \in \cup_{p \in P}C_p$.
256 |
257 | \section{Problem 12}
258 |
259 | What about $A = \emptyset $, $B = \{1\}$, $C = \emptyset$, $D =
260 | \{2\}$, huh?
261 |
262 | \end{document}
263 |
264 |
265 |
266 |
267 |
--------------------------------------------------------------------------------
/chapter 2/section2-2.md:
--------------------------------------------------------------------------------
1 | Equivalences Involving Quantifiers
2 | -----------------------------------
3 | -----------------------------------
4 |
5 | Notes
6 | -----
7 |
8 | * Universal quantification distributes over conjunction:
9 | ∀x(E(x) ∧ T (x)) is equivalent to ∀x E(x) ∧ ∀x T (x)
10 | * Existential quantification doesn't distributes over conjunction.
11 | * Existential quantification distributes over disjunction.
12 | * Universal quantification doesn't distribute over disjunction.
13 |
14 | Exercise 1
15 | -----------
16 |
17 | Negate these statements and then reexpress the results as equivalent
18 | positive statements. (See Example 2.2.1.)
19 | (a) Everyone who is majoring in math has a friend who needs help with
20 | his homework.
21 | (b) Everyone has a roommate who dislikes everyone.
22 | (c) A ∪ B ⊆ C \ D.
23 | (d) ∃x∀y[y > x → ∃z(z 2 + 5z = y)]
24 |
25 | (a)
26 |
27 | * Everyone who is majoring in math has a friend who needs help with
28 | his homework.
29 | * ∀x(if x is majoring in math, then x has a friend who needs help
30 | with his homework.)
31 | * ∀x(M(x) -> ∃y(F(y,x) ∧ H(y)))
32 |
33 | where
34 |
35 | M(x) = x is majoring in Math.
36 | F(y,x) = y is a friend of x.
37 | H(y) = y needs help with his homework.
38 |
39 | Now negate the above logical form:
40 |
41 | * ¬∀x(M(x) -> ∃y(F(y,x) ∧ H(y)))
42 | * ∃x¬(¬M(x) ∨ ∃y(F(y,x) ∧ H(y)))
43 | * ∃x(M(x) ∧ ¬(∃y(F(y,x) ∧ H(y))))
44 | * ∃x(M(x) ∧ ∀y(¬F(y,x) ∨ ¬H(y)))
45 | * ∃x(M(x) ∧ ∀y(F(y,x) -> ¬H(y)))
46 |
47 | There is some `x` who is majoring in Maths and for all `y`, if `y` is
48 | a friend of `x` then `y` doesn't need help with this homework.
49 |
50 | (b)
51 |
52 | Everyone has a roommate who dislikes everyone.
53 |
54 | R(x,y) = x has a roommate y.
55 | L(y,x) = y likes x.
56 |
57 | * ∀x∃y(R(x,y) ∧ ∀z¬L(y,z))
58 |
59 | Now negate the above logical form:
60 |
61 | * ¬∀x∃y(R(x,y) ∧ ∀z¬L(y,z))
62 | * ∃x¬y(R(x,y) ∧ ∀z¬L(y,z))
63 | * ∃x∀y¬(R(x,y) ∧ ∀z¬L(y,z))
64 | * ∃x∀y(¬R(x,y) ∨ ¬∀z¬L(y,z))
65 | * ∃x∀y(¬R(x,y) ∨ ∃zL(y,z))
66 | * ∃x∀y(R(x,y) -> ∃zL(y,z))
67 |
68 | For some `x` and for all `y, if x has a roommate y, then y
69 | likes someone. Or
70 | There is someone such that all of his roommates have somebody
71 | they like.
72 |
73 | (c)
74 |
75 | * `A ∪ B ⊆ C \ D`
76 | * `∀x(x ∈ A ∪ B -> x ∈ C \ D)`
77 | * `∀x(x ∉ A ∪ B ∨ x ∈ C \ D)`
78 | * `¬∀x(x ∉ A ∪ B ∨ x ∈ C \ D)` (Negation)
79 | * `∃x¬(x ∉ A ∪ B ∨ x ∈ C \ D)`
80 | * `∃x(x ∈ A ∪ B ∧ x ∉ C \ D)`
81 |
82 | (d)
83 |
84 | * `∃x∀y[y > x → ∃z(z^2 + 5z = y)]`
85 | * `¬∃x∀y[y > x → ∃z(z^2 + 5z = y)]`
86 | * `∀x¬∀y[y > x → ∃z(z^2 + 5z = y)]`
87 | * `∀x∃y¬[y > x → ∃z(z^2 + 5z = y)]`
88 | * `∀x∃y¬[¬(y > x) ∨ ∃z(z^2 + 5z = y)]`
89 | * `∀x∃y(y > x) ∧ ∀z(z^2 + 5z ≠ y)`
90 |
91 | Exercise 2
92 | -----------
93 |
94 | Negate these statements and then reexpress the results as equivalent
95 | positive statements. (See Example 2.2.1.)
96 | (a) There is someone in the freshman class who doesn’t have a roommate.
97 | (b) Everyone likes someone, but no one likes everyone.
98 | (c) ∀a ∈ A∃b ∈ B(a ∈ C ↔ b ∈ C).
99 | (d) ∀y > 0∃x(ax 2 + bx + c = y).
100 |
101 | (a)
102 |
103 | F(x) = x is in the freshman class
104 | R(x,y) = x has roommate y.
105 |
106 | Original Statement:
107 |
108 | * `∃x F(x) ∧ ¬∃yR(x,y)`
109 |
110 | Negate:
111 |
112 | * `¬∃x F(x) ∧ ∃yR(x,y)`
113 | * `∀x¬(F(x) ∧ ∃yR(x,y))`
114 | * `∀x¬F(x) ∨ ∃yR(x,y)`
115 | * `∀x ¬F(x) ∨ ∃yR(x,y)`
116 | * `∀x F(x) -> ∃yR(x,y)`
117 |
118 | Everyone in the freshman class has at least one roommate.
119 |
120 | (b)
121 |
122 | L(x,y) = x likes y
123 |
124 | * `∀x∃y L(x,y) ∧ ¬(Someone likes everyone)`
125 | * `∀x∃y L(x,y) ∧ ¬∃z∀a(L(z,a))`
126 |
127 | Negate:
128 |
129 | * `¬∀x∃y L(x,y) ∧ ¬∃z∀a(L(z,a))`
130 | * `∃x∀y ¬(L(x,y) ∧ ¬∃z∀a(L(z,a)))`
131 | * `(∃x∀y ¬L(x,y)) ∨ (∃z∀a(L(z,a)))`
132 |
133 | * (There exists `x` such that for all `y`, x doesn't like y) ∨ (Someone
134 | likes everyone)
135 | * (Someone doesn't like anyone) or (Someone likes everyone)
136 |
137 | Either someone doesn't like anyone or someone likes everyone.
138 |
139 | (c)
140 |
141 | * `¬(∀a ∈ A∃b ∈ B(a ∈ C ↔ b ∈ C))`
142 | * `∃a ∈ A ∀b ∈ B ¬(a ∈ C ↔ b ∈ C)`
143 |
144 | (d)
145 |
146 | * `¬∀y > 0 ∃x(ax^2 + bx + c = y)`
147 | * `∃y > 0 ∀x¬(ax^2 + bx + c = y)`
148 | * `∃y > 0 ∀x(ax^2 + bx + c ≠ y)`
149 |
150 | Exercise 3
151 | ----------
152 |
153 | Are these statements true or false? The universe of discourse is N.
154 | (a) ∀x(x < 7 → ∃a∃b∃c(a 2 + b 2 + c 2 = x)).
155 | (b) ∃!x((x − 4)^2 = 9).
156 | (c) ∃!x((x − 4)^2 = 25).
157 | (d) ∃x∃y((x − 4)^2 = 25 ∧ (y − 4)^2 = 25).
158 |
159 | (a)
160 |
161 | `∀x(x < 7 → ∃a∃b∃c(a^2 + b^2 + c^2 = x))`
162 |
163 | True (Substitute value for x=6,5,4...0 and check if for yourself)
164 |
165 | (b)
166 |
167 | `∃!x((x − 4)^2 = 9)`
168 |
169 | Solving the equation, I can find two `x` solutions: 7,1 both falling
170 | in the domain of N.
171 |
172 | False
173 |
174 | (c)
175 |
176 | `∃!x((x − 4)^2 = 25)`
177 |
178 | Solving the equation, I can find two `x` solutions: 9,-1. But the
179 | solution is only 9 since -1 falls out of the domain.
180 |
181 | True
182 |
183 | (d)
184 |
185 | True (from above)
186 |
187 | Exercise 4
188 | -----------
189 |
190 | Show that the second quantifier negation law, which says that ¬∀x P(x)
191 | is equivalent to ∃x¬P(x), can be derived from the first, which says that
192 | ¬∃x P(x) is equivalent to ∀x¬P(x). (Hint: Use the double negation
193 | law.)
194 |
195 | Solution:
196 |
197 | Given:
198 | ¬∃x P(x) is equivalent to ∀x¬P(x)
199 | or
200 | ¬∃x ¬P(x) is equivalent to ∀PX(x) (Replacing P(x) by ¬P(x))
201 | or
202 | ∃x ¬P(x) is equivalent to ¬∀xP(x) (Double negation)
203 |
204 | Exercise 5
205 | ----------
206 |
207 | Show that ¬∃x ∈ A P(x) is equivalent to ∀x ∈ A¬P(x)
208 |
209 | * ¬∃x ∈ A P(x)
210 | * ¬∃x((x ∈ A) ∧ P(x))
211 | * ∀x¬((x ∈ A) ∧ P(x))
212 | * ∀x(¬(x ∈ A) ∨ ¬P(x))
213 | * ∀x(x ∈ A -> ¬P(x))
214 | * ∀x ∈ A ¬P(x)
215 |
216 | Exercise 6
217 | -----------
218 |
219 | Show that the existential quantifier distributes over disjunction. In other
220 | words, show that ∃x(P(x) ∨ Q(x)) is equivalent to ∃x P(x) ∨ ∃x
221 | Q(x).
222 |
223 | * ∃x(P(x) ∨ Q(x))
224 | * ¬¬∃x(P(x) ∨ Q(x))
225 | * ¬∀x¬(P(x) ∨ Q(x))
226 | * ¬∀x(¬P(x) ∧ ¬Q(x))
227 | * ¬(∀x¬P(x) ∧ ∀x¬Q(x))
228 | * ∃xP(x) ∨ ∃xQ(x)
229 |
230 | Exercise 7
231 | -----------
232 |
233 | Show that ∃x(P(x) → Q(x)) is equivalent to ∀x P(x) → ∃x Q(x)
234 |
235 | * `∃x(P(x) → Q(x))`
236 | * `∃x(¬P(x) ∨ Q(x))`
237 | * `¬∀x P(x) ∨ ∃xQ(x)`
238 | * `∀x P(x) -> ∃xQ(x)`
239 |
240 | Exercise 8
241 | -----------
242 |
243 | Show that (∀x ∈ A P(x)) ∧ (∀x ∈ B P(x)) is equivalent to ∀x ∈
244 | (A ∪ B)P(x). (Hint: Start by writing out the meanings of the bounded
245 | quantifiers in terms of unbounded quantifiers.)
246 |
247 | * `(∀x ∈ A P(x)) ∧ (∀x ∈ B P(x))`
248 | * `(∀x((x ∈ A) -> P(x))) ∧ (∀x((x ∈ B) -> P(x)))`
249 | * `∀x((x ∈ A) -> P(x)) ∧ ((x ∈ B) -> P(x))`
250 | * `∀x((¬(x ∈ A) ∨ P(x)) ∧ (¬(x ∈ B) ∨ P(x)))`
251 | * `∀x(((¬(x ∈ A) ∨ P(x)) ∧ (¬(x ∈ B))) ∨ ((¬(x ∈ A) ∨ P(x)) ∧ P(x)))`
252 | * `∀x(((¬(x ∈ A) ∨ P(x)) ∧ (¬(x ∈ B))) ∨ P(x)))`
253 | * `∀x((¬(x ∈ A) ∧ ¬(x ∈ B)) ∨ P(x) ∨ P(x))`
254 | * `∀x((¬(x ∈ A) ∧ ¬(x ∈ B)) ∨ P(x))`
255 | * `∀x¬((x ∈ A) ∨ (x ∈ B)) ∨ P(x)`
256 | * `∀x((x ∈ A) ∨ (x ∈ B)) -> P(x)`
257 | * `∀x ∈ (A ∪ B)P(x)`
258 |
259 | Exercise 9
260 | -----------
261 |
262 | Is ∀x(P(x) ∨ Q(x)) equivalent to ∀x P(x) ∨ ∀x Q(x)? Explain. (Hint: Try
263 | assigning meanings to P(x) and Q(x).)
264 |
265 | I found this interesting example on
266 | [stackexchange](http://math.stackexchange.com/questions/6410/distribution-of-universal-quantifiers)
267 | to argue this:
268 |
269 | P(x) = Chess coin is black.
270 | Q(x) = Chess coin is white.
271 |
272 | ∀x(P(x) ∨ Q(x)) means: For all chess coins x, x is either black or x
273 | is white.
274 | ∀x P(x) ∨ ∀x Q(x) means: All chess coins are black or All chess coins
275 | are white.
276 |
277 | False
278 |
279 | Exercise 10
280 | ------------
281 |
282 | (a) Show that ∃x ∈ A P(x) ∨ ∃x ∈ B P(x) is equivalent to ∃x ∈ (A ∪ B)
283 | P(x).
284 | (b) Is ∃x ∈ A P(x) ∧ ∃x ∈ B P(x) equivalent to ∃x ∈ (A ∩ B) P(x)?
285 | Explain
286 |
287 | (a)
288 |
289 | * `∃x ∈ A P(x) ∨ ∃x ∈ B P(x)`
290 | * `∃x ((x ∈ A) ∧ P(x)) ∨ ∃x (x ∈ B ∧ P(x))`
291 | * `∃x ((x ∈ A) ∧ P(x)) ∨ (x ∈ B ∧ P(x))`
292 | * `∃x ((x ∈ A) ∨ (x ∈ B ∧ P(x))) ∧ (P(x) ∨ (x ∈ B ∧ P(x)))`
293 | * `∃x ((x ∈ A) ∨ (x ∈ B ∧ P(x))) ∧ P(x)`
294 | * `∃x ((x ∈ A) ∨ P(x)) ∧ (x ∈ A ∨ x ∈ B) ∧ P(x)`
295 | * `∃x P(x) ∧ (x ∈ A ∨ x ∈ B)` (I love absorption law!)
296 | * `∃x ∈ (A ∪ B) P(x)`
297 |
298 | (b)
299 |
300 | * `∃x ∈ (A ∩ B) P(x)`
301 | * `∃x x ∈ A ∧ x ∈ B ∧ P(x)`
302 |
303 | Existential quantification doesn't distribute over conjunction, so
304 | they aren't equivalent.
305 |
306 | Exercise 11
307 | ------------
308 |
309 | Show that the statements A ⊆ B and A \ B = ∅ are equivalent by writing
310 | each in logical symbols and then showing that the resulting formulas are
311 | equivalent.
312 |
313 |
314 | * `A ⊆ B`
315 | * `∀x (x ∈ A -> x ∈ B)`
316 |
317 | * `A \ B = ∅`
318 | * `¬∃x (x ∈ A) ∧ ¬(x ∈ B)` (Negating it because no element exists)
319 | * `¬∃x ¬(¬(x ∈ A) ∨ (x ∈ B))`
320 | * `¬∃x ¬(x ∈ A -> x ∈ B)`
321 | * `∀x (x ∈ A -> x ∈ B)`
322 |
323 | Hence they are equivalent.
324 |
325 | Exercise 12
326 | -----------
327 |
328 | Let T (x, y) mean “x is a teacher of y.” What do the following statements
329 | mean? Under what circumstances would each one be true? Are any of
330 | them equivalent to each other?
331 | (a) ∃!yT (x, y).
332 | (b) ∃x∃!yT (x, y).
333 | (c) ∃!x∃yT (x, y).
334 | (d) ∃y∃!x T (x, y).
335 | (e) ∃!x∃!yT (x, y).
336 | (f ) ∃x∃y[T (x, y) ∧ ¬∃u∃v(T (u, v) ∧ (u = x ∨ v = y))
337 |
338 | (a)
339 |
340 | * `∃!yT (x, y)`
341 |
342 | x is a teacher of one person.
343 |
344 | (b)
345 |
346 | * `∃x∃!yT (x, y)`
347 |
348 | Literally: For some x there exists one y such that x is a teacher of y.
349 |
350 | There is a teacher who teaches exactly one student.
351 |
352 | (c)
353 |
354 | * `∃!x∃yT (x, y)`
355 |
356 | Literally: There exists only one x for some y such that x is a teacher of y.
357 |
358 | There is a student who has only one teacher.
359 |
360 | (d)
361 |
362 | * `∃y∃!x T (x, y)`
363 |
364 | Literally: For some y there exits one x such that x is a teacher of y.
365 |
366 | There is a student who has only one teacher. (equivalent to c)
367 |
368 | (e)
369 |
370 | * `∃!x∃!yT (x, y)`
371 |
372 | Literally: There exists exactly one x and y such that x is a teacher of y.
373 |
374 | There is exactly one teacher who teaches exactly one student.
375 |
376 | (f)
377 |
378 | * `∃x∃y[T (x, y) ∧ ¬∃u∃v(T (u, v) ∧ (u ≠ x ∨ v ≠ y))`
379 | * `∃x∃y[T (x, y) ∧ ∀u∀v(¬T (u, v) ∨ ¬(u ≠ x ∨ v ≠ y))]`
380 | * `∃x∃y[T (x, y) ∧ ∀u∀v(¬T (u, v) ∨ ((u = x) ∧ v = y))]`
381 | * `∃x∃y[T (x, y) ∧ ∀u∀v(T(u,v) -> ((u = x) ∧ v = y))]`
382 | * `∃!x∃!yT (x, y)`
383 |
384 | f is equivalent to e
385 |
--------------------------------------------------------------------------------
/chapter 2/section2-1.md:
--------------------------------------------------------------------------------
1 | Quantifiers
2 | ------------
3 |
4 | Interesting threads:
5 | * [Relationship between quantifiers and logic](http://philosophy.stackexchange.com/questions/4165/how-do-quantifiers-work-in-predicate-logic)
6 |
7 | Exercise 1
8 | ----------
9 |
10 | Analyze the logical forms of the following statements.
11 | (a) Anyone who has forgiven at least one person is a saint.
12 | (b) Nobody in the calculus class is smarter than everybody in the discrete
13 | math class.
14 | (c) Everyone likes Mary, except Mary herself.
15 | (d) Jane saw a police officer, and Roger saw one too.
16 | (e) Jane saw a police officer, and Roger saw him too.
17 |
18 | (a)
19 |
20 | F(x,y) = x has forgiven y
21 | S(x) = x is a saint.
22 |
23 | ∀x(∃yF(x,y) -> S(x))
24 |
25 | (b)
26 |
27 | * ¬(Somebody in the calculus room is smarter than everybody in the
28 | discrete math class)
29 | * There does not exist someone in the calculus room And such that for
30 | all students y, if y is enrolled in Discrete math, then x is
31 | smarter than y.
32 |
33 | C(x) = x is in calculus room.
34 | D(x) = x is in discrete math room.
35 | S(x,y) = x is smarter than y.
36 |
37 | ¬∃x(C(x) ∧ ∀y( D(y) -> S(x,y)))
38 |
39 |
40 | (c)
41 |
42 | * If someone doesn't like Mary, then that someone must be Mary
43 | * If someone isn't Mary, then someone likes Mary.
44 |
45 | L(x,y) = x likes y
46 | ∀x(¬(x = m) -> L(x,m))
47 |
48 | (d)
49 |
50 | S(x,y) = x saw y
51 | P(x) = x is a police officer.
52 | j = Jane
53 | r = Roger
54 |
55 | ∃x (P(x) ∧ S(j,x)) ∧ ∃y (P(y) ∧ S(r,y))
56 |
57 | (e)
58 |
59 | ∃x (P(x) ∧ S(j,x) ∧ S(r,x))
60 |
61 | Exercise 2
62 | -----------
63 |
64 | Analyze the logical forms of the following statements.
65 | (a) Anyone who has bought a Rolls Royce with cash must have a rich
66 | uncle.
67 | (b) If anyone in the dorm has the measles, then everyone who has a friend
68 | in the dorm will have to be quarantined.
69 | (c) If nobody failed the test, then everybody who got an A will tutor
70 | someone who got a D.
71 | (d) If anyone can do it, Jones can.
72 | (e) If Jones can do it, anyone can.
73 |
74 | (a)
75 |
76 | If x bought Rolls Royce with cash, then he must have a rich uncle.
77 |
78 | R(x) = x bought Rolls Royce with cash.
79 | U(x) = x has a rich uncle.
80 |
81 | ∀x(R(x) -> U(x))
82 |
83 | (b)
84 |
85 | D(x) = x is in the dorm
86 | M(x) = x in the dorm has measles
87 | F(y,x) = y is a friend of x
88 | Q(y) = Quarantine y
89 |
90 | * (Someone in the dorm has measles) -> (everyone who has a friend in
91 | the dorm will have to be quarantined)
92 | * `∃x(D(x) ∧ M(x)` -> ∀y(If `y` is a friend of `z` and `z` lives in
93 | dorm then `y` has to be quarantined)
94 | * `∃x(D(x) ∧ M(x)) -> ∀y(∃z(F(y,z) ∧ D(z)) -> Q(y))`
95 |
96 | (c)
97 |
98 | F(x) = x failed the test
99 | G(x,y) = x got y grade.
100 | T(x,y) = x will tutor y
101 |
102 |
103 | * (Nobody failed the test) -> (Everybody who got an A will tutor
104 | someone who got a D)
105 | * ¬(Somebody failed the test) -> ∀y(If y got A grade, then y will
106 | tutor someone who got a D)
107 | * `¬(∃x F(x)) -> ∀y(G(y,'A') -> ∃z(T(y,z) ∧ G(z,'D')))`
108 |
109 | (d)
110 |
111 | D(x) = x can do it
112 |
113 | ∃x D(x) -> D(J)
114 |
115 | (e)
116 |
117 | D(x) = x can do it
118 |
119 | D(J) -> ∀xD(x)
120 |
121 | Exercise 3
122 | -----------
123 |
124 | Analyze the logical forms of the following statements. The universe of
125 | discourse is R. What are the free variables in each statement?
126 | (a) Every number that is larger than x is larger than y.
127 | (b) For every number a, the equation ax^2 + 4x − 2 = 0 has at least one
128 | solution iff a ≥ −2.
129 | (c) All solutions of the inequality x^3 − 3x < 3 are smaller than 10.
130 | (d) If there is a number x such that x^2 + 5x = w and there is a number y
131 | such that 4 − y^2 = w, then w is between −10 and 10.
132 |
133 | (a)
134 |
135 | ∀z (z > x) -> (z > y)
136 |
137 | Free variables: x and y
138 |
139 | (b)
140 |
141 | ∀a∃x (ax^2 + 4x − 2 = 0) <-> (a >= -2)
142 |
143 | No free variables.
144 |
145 | (c)
146 |
147 | ∀x [x^3 − 3x < 3 -> x < 10]
148 |
149 | No free variables.
150 |
151 | (d)
152 |
153 | (∃x (x^2 + 5x = w) ∧ ∃y (4 − y^2 = w)) -> (-10 < w < 10)
154 |
155 | Free variable: w
156 |
157 | Exercise 4
158 | -----------
159 |
160 | Translate the following statements into idiomatic English.
161 | (a) ∀x[(H (x) ∧ ¬∃y M(x, y)) → U (x)], where H (x) means “x is a man,”
162 | M(x, y) means “x is married to y,” and U (x) means “x is unhappy.”
163 | (b) ∃z(P(z, x) ∧ S(z, y) ∧ W (y)), where P(z, x) means “z is a parent of
164 | x,” S(z, y) means “z and y are siblings,” and W (y) means “y is a
165 | woman.”
166 |
167 | (a)
168 |
169 | * `∀x[(H (x) ∧ ¬∃y M(x, y)) → U (x)]`
170 | * `∀x`(if x is a man and for some y, x isn't married to y then x is
171 | unhappy)
172 | * for all x, if x is a man and for some y, x isn't married to y then
173 | x is unhappy.
174 | * Every unmarried man is unhappy.
175 |
176 | (b)
177 |
178 | * `∃z(P(z, x) ∧ S(z, y) ∧ W (y))`
179 | * `∃z`(z is a parent of x AND z and y are siblings AND y is a woman)
180 | * y is a sibling to z.
181 |
182 | Exercise 5
183 | -----------
184 |
185 | Translate the following statements into idiomatic mathematical English.
186 | (a) ∀x[(P(x) ∧ ¬(x = 2)) → O(x)], where P(x) means “x is a prime
187 | number” and O(x) means “x is odd.”
188 | (b) ∃x[P(x) ∧ ∀y(P(y) → y ≤ x)], where P(x) means “x is a perfect
189 | number.”
190 |
191 | (a)
192 |
193 | * `∀x[(P(x) ∧ ¬(x = 2)) → O(x)]`
194 | * `∀x`(If x is a prime number AND x is not equal to 2, then x is an
195 | odd number)
196 | * Every prime number except 2 is an odd number.
197 |
198 | (b)
199 |
200 | * `∃x[P(x) ∧ ∀y(P(y) → y ≤ x)]`
201 | * ∃x(P(x) ∧ ∀y(If y is a perfect number then y is less than or equal
202 | to x.))
203 | * ∃x(P(x) ∧ Every perfect number is less than or equal to x.)
204 | * There exists a perfect number such that all the perfect numbers are
205 | either less than or equal to it.
206 |
207 | Exercise 6
208 | -----------
209 |
210 | Are these statements true or false? The universe of discourse is the set of
211 | all people, and P(x, y) means “x is a parent of y.”
212 | (a) ∃x∀y P(x, y).
213 | (b) ∀x∃y P(x, y).
214 | (c) ¬∃x∃y P(x, y).
215 | (d) ∃x¬∃y P(x, y).
216 | (e) ∃x∃y¬P(x, y).
217 |
218 | (a)
219 |
220 | * `∃x∀y P(x, y)`
221 | * There exists some parent x who is parent to all the peoples.
222 |
223 | This may appear true theoretically, but one cannot be a parent to
224 | himeself and hence False.
225 |
226 | False
227 |
228 | (b)
229 |
230 | * `∀x∃y P(x, y)`
231 | * Everyone is parent to someone.
232 |
233 | This is clearly `False` because if x is parent to y, then y is not a
234 | parent.
235 |
236 | (c)
237 |
238 | See also this
239 | [thread](http://math.stackexchange.com/questions/903549/concluding-truth-value-from-universe-of-discourse)
240 | for the discussion.
241 |
242 | * `¬∃x∃y P(x, y)`
243 | * `¬∃x(∃y(P(x, y)))`
244 | * There does not exist some x, there exists some y such that x is a
245 | parent of y.
246 | * There is no x such that x is a parent of someone.
247 | * There does not exist anyone who is a parent of someone.
248 |
249 | False
250 |
251 | (d)
252 |
253 | * `∃x¬∃y P(x, y)`
254 | * For some x, there exists no y such that x is a parent of y.
255 | * `x` has no children.
256 | * Some parents have no children.
257 |
258 | True
259 |
260 | (e)
261 |
262 | * `∃x∃y¬P(x, y)`
263 | * There exists some x and y such that x is not a parent to y.
264 |
265 | True
266 |
267 | Exercise 7
268 | -----------
269 |
270 | Are these statements true or false? The universe of discourse is N.
271 | (a) ∀x∃y(2x − y = 0).
272 | (b) ∃y∀x(2x − y = 0).
273 | (c) ∀x∃y(x − 2y = 0).
274 | (d) ∀x(x < 10 → ∀y(y < x → y < 9)).
275 | (e) ∃y∃z(y + z = 100).
276 | (f) ∀x∃y(y > x ∧ ∃z(y + z = 100)).
277 |
278 | (a)
279 |
280 | * `∀x∃y(2x − y = 0)`
281 | * For all x and some y such that `2x − y = 0`
282 |
283 | True
284 |
285 | (b)
286 |
287 | * `∃y∀x(2x − y = 0)`
288 | * There exists some number y such that `2x − y = 0` for any number x.
289 |
290 | False
291 |
292 | (c)
293 |
294 | * `∀x∃y(x − 2y = 0)`
295 | * For any number x, there exists some number y such that `x − 2y = 0`.
296 |
297 | False (CounterExample: x = 3)
298 |
299 | (d)
300 |
301 | * `∀x(x < 10 → ∀y(y < x → y < 9))`
302 | * For any number x, if `x < 10` then for any number y, `y < x` implies
303 | `y < 9`.
304 |
305 | True
306 |
307 | (e)
308 |
309 | * `∃y∃z(y + z = 100)`
310 |
311 | True
312 |
313 | (f)
314 |
315 | * `∀x∃y(y > x ∧ ∃z(y + z = 100))`
316 | * `∀x∃y(x < y ∧ ∃z(y + z = 100))`
317 | * Counterexample: y = 1000, x = 999, z = 0
318 |
319 | False
320 |
321 | Exercise 8
322 | ----------
323 |
324 | Are these statements true or false? The universe of discourse is R.
325 | (a) ∀x∃y(2x − y = 0).
326 | (b) ∃y∀x(2x − y = 0).
327 | (c) ∀x∃y(x − 2y = 0).
328 | (d) ∀x(x < 10 → ∀y(y < x → y < 9)).
329 | (e) ∃y∃z(y + z = 100).
330 | (f) ∀x∃y(y > x ∧ ∃z(y + z = 100)).
331 |
332 | (a)
333 |
334 | * `∀x∃y(2x − y = 0)`
335 | * For all x, there exist some y such that `2x - y = 0`
336 |
337 | True
338 |
339 | (b)
340 |
341 | * `∃y∀x(2x − y = 0)`
342 | * There exists some `y`, such that for any `x` it is `2x - y = 0`
343 |
344 | False
345 |
346 | (c)
347 |
348 | * `∀x∃y(x − 2y = 0)`
349 | * For all x and y `x - 2y = 0`
350 |
351 | True
352 |
353 | (d)
354 |
355 | * `∀x(x < 10 → ∀y(y < x → y < 9))`
356 | * For all x, if `x < 10` then for all y `(y < x → y < 9)`
357 |
358 | False (x = 9.9, y = 9.8)
359 |
360 | (e)
361 |
362 | * `∃y∃z(y + z = 100)`
363 | * For some y and z, `y + z = 100`
364 |
365 | True
366 |
367 | (f)
368 |
369 | * `∀x∃y(y > x ∧ ∃z(y + z = 100))`
370 | * For all `x` and some `y`, `y > x` and there exists some `z` such
371 | that `y + z = 100`
372 |
373 | True
374 |
375 | Exercise 9
376 | -----------
377 |
378 | Are these statements true or false? The universe of discourse is Z.
379 | (a) ∀x∃y(2x − y = 0).
380 | (b) ∃y∀x(2x − y = 0).
381 | (c) ∀x∃y(x − 2y = 0).
382 | (d) ∀x(x < 10 → ∀y(y < x → y < 9)).
383 | (e) ∃y∃z(y + z = 100).
384 | (f) ∀x∃y(y > x ∧ ∃z(y + z = 100)).
385 |
386 | (a)
387 |
388 | * `∀x∃y(2x − y = 0)`
389 | * For all `x`, there exist some `y` such that `2x - y = 0`
390 |
391 | True
392 |
393 | (b)
394 |
395 | * `∃y∀x(2x − y = 0)`
396 | * There exists some `y`, such that for any `x` it is `2x - y = 0`
397 |
398 | False
399 |
400 | (c)
401 |
402 | * `∀x∃y(x − 2y = 0)`
403 | * For any `x`, there exists some `y` such that `x − 2y = 0`
404 |
405 | False (Counterexample: x = 3)
406 |
407 | (d)
408 |
409 | * `∀x(x < 10 → ∀y(y < x → y < 9))`
410 | * For all `x`, if `x < 10` then for all `y` `(y < x → y < 9)`
411 |
412 | True
413 |
414 | (e)
415 |
416 | * `∃y∃z(y + z = 100)`
417 | * For some `y` and `z`, `y + z = 100`
418 |
419 | True
420 |
421 | (f)
422 |
423 | * `∀x∃y(y > x ∧ ∃z(y + z = 100))`
424 | * For all `x` and some `y`, `y > x` and there exists some `z` such
425 | that `y + z = 100`
426 |
427 | True
428 |
--------------------------------------------------------------------------------
/chapter 2/section2-3.tex:
--------------------------------------------------------------------------------
1 | %%%%%%%%%%%%%%%%%%%%%%%%%%%%%%%%%%%%%%%%%
2 | % Minimalists Report template
3 | % Author: Sibi
4 | %%%%%%%%%%%%%%%%%%%%%%%%%%%%%%%%%%%%%%%%%
5 | \documentclass{article}
6 | \usepackage{graphicx}
7 | \usepackage{verbatim}
8 | \usepackage{amsmath}
9 | \usepackage{amsfonts}
10 | \usepackage{amssymb}
11 | \begin{document}
12 | \title{Chapter 2 (Section 2.3)}
13 | \author{Sibi}
14 | \date{\today}
15 | \maketitle
16 | \newpage
17 |
18 | \section{Problem 1}
19 |
20 | \begin{center}
21 | (1.1) $F \subseteq \wp(A)$
22 | \end{center}
23 | \begin{align*}
24 | \forall x (x \in F \implies x \in \wp(A)) \\
25 | \forall x (x \in F \implies x \subseteq A) \\
26 | \forall x (x \in F \implies \forall y (y \in x \implies y \in A))
27 | \end{align*}
28 |
29 | \begin{center}
30 | (1.2) $A \subseteq \{2n + 1 | n \in \mathbb{N}\}$ \\
31 | \end{center}
32 | \begin{align*}
33 | \forall x (x \in A \implies x \in \{2n + 1 | n \in \mathbb{N}\}) \\
34 | \forall x (x \in A \implies \exists n \in \mathbb{N} (x=2n+1))
35 | \end{align*}
36 |
37 | \begin{center}
38 | (1.3) $\{n^2 + n + 1 | n \in \mathbb{N} \} \subseteq \{2n + 1 | n \in \mathbb{N} \}$ \\
39 | \end{center}
40 | \begin{align*}
41 | \forall x (x \in \left\{n^2 + n + 1 \mid n \in \mathbb{N} \right\}) \implies (x \in \left\{2n + 1 \mid n \in \mathbb{N} \right\}) \\
42 | \forall x (\exists n \in \mathbb{N} (x = n^2 + n + 1)) \implies (\exists n \in \mathbb{N} (x=2n+1)) \\
43 | \end{align*}
44 |
45 | \begin{center}
46 | (1.4) $\wp(\cup_{i \in I} A_i) \nsubseteq \cup_{i \in I} \wp(A_i)$
47 | \end{center}
48 | \begin{align*}
49 | \exists x(x \in \wp(\cup_{i \in I} A_i) \land x \notin \cup_{i \in I} \wp(A_i)) \\
50 | \exists x(x \subseteq \cup_{i \in I} A_i \land x \notin \cup_{i \in I} \wp(A_i)) \\
51 | \exists x( \forall y (y \in x \implies y \in \cup_{i \in I} A_i) \land
52 | x \notin \cup_{i \in I} \wp(A_i)) \\
53 | \exists x( \forall y (y \in x \implies \exists i \in I(y \in A_i))
54 | \land x \notin \cup_{i \in I} \wp(A_i)) \\
55 | \exists x( \forall y (y \in x \implies \exists i \in I(y \in A_i))
56 | \land \neg (x \in \cup_{i \in I} \wp(A_i))) \\
57 | \exists x( \forall y (y \in x \implies \exists i \in I(y \in A_i))
58 | \land \neg (\exists i \in I(x \in \wp(A_i)))) \\
59 | \exists x( \forall y (y \in x \implies \exists i \in I(y \in A_i))
60 | \land \neg (\exists i \in I(x \subseteq A_i))) \\
61 | \exists x( \forall y (y \in x \implies \exists i \in I(y \in A_i))
62 | \land \neg (\exists i \in I(\forall z (z \in x \implies z \in A_i)))) \\
63 | \exists x( \forall y (y \in x \implies \exists i \in I(y \in A_i))
64 | \land (\forall i \in I(\exists z \neg(z \in x \implies z \in A_i)))) \\
65 | \exists x( \forall y (y \in x \implies \exists i \in I(y \in A_i))
66 | \land (\forall i \in I(\exists z (z \in x \land \neg(z \in A_i))))) \\
67 | \end{align*}
68 |
69 | \section{Problem 2}
70 | \begin{center}
71 | (2.1) $x \in \cup F \setminus \cup G $
72 | \end{center}
73 | \begin{align*}
74 | x \in \cup F \land x \notin \cup G \\
75 | \exists A \in F(x \in A) \land \exists A \in G(x \notin A)
76 | \end{align*}
77 |
78 | \begin{center}
79 | (2.2) $\left\{ x \in B \mid x \notin C\right\} \in \wp(A)$
80 | \end{center}
81 | \begin{align*}
82 | \left\{ x \in B \mid x \notin C\right\} \subseteq A \\
83 | \forall y(y \in \left\{ x \in B \mid x \notin C\right\} \implies y \in A) \\
84 | \forall y(y \in B \land y \notin C \implies y \in A) \\
85 | \end{align*}
86 |
87 | \begin{center}
88 | (2.3) $ x \in \cap_{i \in I}(A_i \cup B_i)$
89 | \end{center}
90 | \begin{align*}
91 | \forall i \in I(x \in (A_i \cup B_i)) \\
92 | \forall i \in I(x \in A_i \lor x \in B_i)
93 | \end{align*}
94 |
95 | \begin{center}
96 | (2.4) $x \in (\cap_{i \in I}A_i) \cup (\cap_{i \in I}B_i)$
97 | \end{center}
98 | \begin{align*}
99 | \forall i \in I(x \in A_i)) \lor (\forall i \in I(x \in B_i)
100 | \end{align*}
101 |
102 | \section{Problem 3}
103 | \begin{align*}
104 | \left\{\left\{\varnothing\right\}, \varnothing \right\}
105 | \end{align*}
106 |
107 | \section{Problem 4}
108 | \begin{align*}
109 | F = \left\{ \left\{red, green, blue \right\}, \left\{orange, red,
110 | green\right\}, \left\{purple, red, green, blue \right\} \right\} \\
111 | \cup F = \left\{ red, green, blue, orange, purple\right\} \\
112 | \cap F = \left\{ red, green\right\} \\
113 | \end{align*}
114 |
115 | \section{Problem 5}
116 | \begin{align*}
117 | F = \left\{ \left\{3,7,12 \right\}, \left\{5,7,16
118 | \right\}, \left\{5,12,23 \right\} \right\} \\
119 | \cup F = \left\{ 3,7,12,5,16,23 \right\} \\
120 | \cap F = \varnothing \\
121 | \end{align*}
122 |
123 | \section{Problem 6}
124 | \begin{center}
125 | $I = \left\{2,3,4,5\right\}$ \\
126 | $A_i = \left\{i, i+1, i-1, 2*i\right\}$ \\
127 | \end{center}
128 | \begin{align*}
129 | A_2 = \left\{2,3,1,4\right\} \\
130 | A_3 = \left\{3,4,2,6\right\} \\
131 | A_4 = \left\{4,5,3,8\right\} \\
132 | A_5 = \left\{5,6,4,10\right\} \\
133 | \cap_{i \in I}A_i = \left\{4 \right\} \\
134 | \cup_{i \in I}A_i = \left\{1,2,3,4,5,6,8,10 \right\} \\
135 | \end{align*}
136 |
137 | \section{Problem 7}
138 | Too lazy to figure out when they live.
139 | Or if you want a more reasonable answer: Left as an exercise to the reader :P
140 | \section{Problem 8}
141 | \begin{center}
142 | $I = \left\{2,3\right\}$ \\
143 | $A_i = \left\{ i,2i \right\}$ \\
144 | $B_i = \left\{ i, i + 1 \right\}$ \\
145 | \end{center}
146 | \begin{align*}
147 | A_2 = \left\{ 2, 4 \right\} \\
148 | A_3 = \left\{ 3, 6 \right\} \\
149 | B_2 = \left\{ 2, 3 \right\} \\
150 | B_3 = \left\{ 3, 4 \right\} \\ \\
151 | \cap_{i \in I}(A_i \cup B_i) = \left\{3,4 \right\} \\
152 | (\cap_{i \in I}A_i) \cup (\cap_{i \in I}B_i) = \left\{ 3\right\} \\ \\
153 | No they are not equivalent. 2(c) and 2(d) are not equivalent to each other.
154 | \end{align*}
155 |
156 | \section{Problem 9}
157 | \begin{align*}
158 | Let X = \cup_{i \in I} (A_i \cap B_i) \\
159 | Let Y = (\cup_{i \in I}A_i) \cap (\cup_{i \in I} B_i) \\ \\
160 | x \in X is equivalent to x \in \cup_{i \in I} (A_i \cap B_i) \\
161 | \exists i \in I(x \in (A_i \cap B_i))) \\
162 | \exists i \in I(x \in A_i \land x \in B_i) \\ \\
163 | x \in Y is equivalent to x \in (\cup_{i \in I}A_i) \cap (\cup_{i \in I} B_i) \\
164 | (x \in \cup_{i \in I}A_i) \land (x \in \cup_{i \in I} B_i) \\
165 | \exists i \in I(x \in A_i) \land \exists i \in I (x \in B_i) \\ \\
166 | \end{align*}
167 | Now clearly they are not equivalent to each other. A trivial example of them would be:
168 | $Let I = \left\{1,2\right\}$ \\
169 | $A_i = \left\{i\right\}$ \\
170 | $B_i = \left\{2i\right\}$ \\
171 |
172 | \section{Problem 10}
173 | \begin{align*}
174 | x \in \wp(A \cap B) \\
175 | x \subseteq (A \cap B) \\
176 | \forall y (y \in x \implies y \in (A \cap B)) \\
177 | \forall y (y \in x \implies y \in A \land y \in B) \\
178 | \forall y (y \notin x \lor (y \in A \land y \in B)) \\
179 | \forall y ((y \notin x \lor y \in A) \land (y \notin x \lor y \in B)) \\
180 | \forall y(y \in x \implies y \in A) \land \forall y(y \in x \implies y \in B) \\ \\
181 | x \in \wp(A) \cap \wp(B) \\
182 | x \in \wp(A) \land x \in \wp(B) \\
183 | x \subseteq A \land x \subseteq B \\
184 | \forall y(y \in x \implies y \in A) \land \forall y(y \in x \implies y \in B) \\
185 | \end{align*}
186 | %% \left\{ \right\}
187 | \section{Problem 11}
188 | \begin{align*}
189 | A = \left\{1 \right\} \\
190 | B = \left\{2 \right\} \\
191 | \wp(A \cup B) = \left\{\left\{1,2 \right\},\left\{1 \right\}, \left\{2 \right\}, \varnothing \right\} \\
192 | \wp(A) \cup \wp(B) = \left\{ \left\{1 \right\},\left\{2 \right\}, \varnothing \right\}
193 | \end{align*}
194 |
195 | \section{Problem 12}
196 | Problem (a)
197 | \begin{align*}
198 | \cup_{i \in I}(A_i \cup B_i) = (\cup_{i \in I}A_i) \cup (\cup_{i \in I}B_i) \\\\
199 | x \in \cup_{i \in I}(A_i \cup B_i) \\
200 | \exists i \in I(x \in A_i \cup B_i) \\
201 | \exists i \in I(x \in A_i \lor x \in B_i)\\\\
202 | x \in (\cup_{i \in I}A_i) \cup (\cup_{i \in I}B_i) \\
203 | x \in (\cup_{i \in I}A_i) \lor x \in (\cup_{i \in I}B_i) \\
204 | \exists i \in I(x \in A_i) \lor \exists i \in I(x \in B_i) \\
205 | \exists i \in I(x \in A_i \lor x \in B_i) \\
206 | \end{align*}
207 | Problem (b)
208 | \begin{align*}
209 | (\cap F) \cap (\cap G) = \cap(F \cup G)\\
210 | x \in (\cap F) \cap (\cap G) \\
211 | x \in (\cap F) \land x \in (\cap G) \\
212 | \forall A \in F(x \in A) \land \forall A \in G(x \in A)\\\\
213 | x \in \cap(F \cup G) \\
214 | \forall A \in (F \cup G)(x \in A)\\
215 | \forall A(A \in (F \cup G) \implies x \in A) \\
216 | \forall A(A \in F \lor A \in G \implies x \in A)\\
217 | \forall A((A \notin F \land A \notin G) \lor x \in A)\\
218 | \forall A((A \notin F \lor x \in A) \land (A \notin G \lor x \in A))\\
219 | \forall A(A \notin F \lor x \in A) \land \forall A(A \notin G \lor x \in A)\\
220 | \forall A(A \in F \implies x \in A) \land \forall A(A \in G \implies x \in A)\\
221 | \forall A \in F(x \in A) \land \forall A \in G(x \in A)\\
222 | \end{align*}
223 | Problem (c)
224 | \begin{align*}
225 | \cap_{i \in I}(A_i \setminus B_i) = (\cap_{i \in I}A_i) \setminus (\cup_{i \in I}B_i)\\
226 | x \in \cap_{i \in I}(A_i \setminus B_i) \\
227 | \forall i \in I(x \in (A_i \setminus B_i)) \\
228 | \forall i \in I(x \in A_i \land x \notin B_i)\\
229 | \forall i \in I(x \in A_i) \land \forall i \in I(x \notin B_i)\\\\
230 | x \in (\cap_{i \in I}A_i) \setminus (\cup_{i \in I}B_i)\\
231 | x \in (\cap_{i \in I}A_i) \land x \notin (\cup_{i \in I}B_i) \\
232 | \forall i \in I(x \in A_i) \land \neg (x \in \cup_{i \in I}B_i) \\
233 | \forall i \in I(x \in A_i) \land \neg (\exists i \in I(x \in B_i)) \\
234 | \forall i \in I(x \in A_i) \land \forall i \in I (x \notin B_i)
235 | \end{align*}
236 |
237 | \section{Problem 13}
238 | $$ I = \{1,2\} $$
239 | $$ J = \{3,4\} $$
240 | $$ A_{i,j} = \{i, j, i+j\} $$
241 | Problem (a)
242 | \begin{align}
243 | B_j = U_{i \in I}A_{i,j} = A_{1,j} \cup A_{2,j} \\
244 | B_3 = A_{1,3} \cup A_{2,3} \\
245 | = \{ 1,3,4 \} \cup \{ 2,3,5 \} \\
246 | = \{1,2,3,4,5\} \\ \\
247 | B_4 = A_{1,4} \cup A_{2,4} \\
248 | = \{1,4,5\} \cup \{2,4,6\} \\
249 | = \{1,2,4,5,6\} \\
250 | \end{align}
251 | Problem (b)
252 | \begin{align*}
253 | \cap_{j \in J}B_j \\
254 | B_3 \cap B_4 \\
255 | \{1,2,4,5\} \\
256 | \end{align*}
257 | Problem (c)
258 | \begin{align*}
259 | \cup_{i \in I}(\cap_{j \in J}A_{i,j}) \\
260 | \cup_{i \in I}(A_{i,3} \cap A_{i,4}) \\
261 | (A_{1,3} \cap A_{1,4}) \cup (A_{2,3} \cap A_{2,4}) \\
262 | (\{1,3,4\} \cap \{1,4,5\}) \cup (\{2,3,5\} \cap \{2,4,6\}) \\
263 | \{1,4\} \cup \{2\} \\
264 | \{1,2,4\} \\
265 | \end{align*}
266 | No, they are not equivalent. \\
267 | Problem (d)
268 | \begin{align*}
269 | x \in \cap_{j \in J}(\cup_{i \in I}A_{i,j}) \\
270 | \forall j \in J(x \in \cup_{i \in I}A_{i,j})\\
271 | \forall j \in J(\exists i \in I(x \in A_{i,j}))\\ \\
272 | x \in \cup_{i \in I}(\cap_{j \in J}A_{i,j}) \\
273 | \exists i \in I(x \in (\cap_{j \in J}A_{i,j})) \\
274 | \exists i \in I(\forall j \in J(x \in A_{i,j}))\\
275 | \end{align*}
276 | No, they are not equivalent.
277 | \section{Problem 14}
278 | Problem (a)
279 | \begin{align*}
280 | x \in \cup F \\
281 | \exists A \in F(x \in A) \\
282 | \exists A (A \in F \land x \in A) \\
283 | \end{align*}
284 | Now if $F$ is $\emptyset$ then the statement is obviously false. There
285 | is no $A$ that will satisfy that logical form. Hence $\cup \emptyset =
286 | \emptyset$ \\
287 |
288 | Problem (b)
289 | \begin{align*}
290 | x \in \cap F \\
291 | \forall A \in F(x \in A) \\
292 | \forall x (A \in F \implies x \in A) \\
293 | \end{align*}
294 | Now if $F$ is $\emptyset$, then the antecedent is false. But the
295 | logical form is vacously true. So every element in $U$ will satisfy
296 | that. Therefore, $\cap \emptyset = U$
297 | \end{document}
298 |
299 |
--------------------------------------------------------------------------------
/chapter 1/section1-5.md:
--------------------------------------------------------------------------------
1 | The Conditional and Biconditional Connectives
2 | -----------------------------------------------
3 |
4 | Notes from Keith Devlin's lectures:
5 |
6 | * Implication has a truth part and causality part.
7 | * Leave the causation part to the philosophers and just in mathematics
8 | we will just focus on the truth part.
9 | * Whenever we have genuine implication (with causality), the truth
10 | behaviour of the conditional is the correct one.
11 | * When we do have a genuine implication, the definition of the
12 | conditional will agree with the way implication behaves. And when we
13 | don't have genuine implication, the conditional will still be
14 | defined.
15 |
16 | The following all mean p implies q:
17 |
18 | * if p, then q
19 | * p is sufficient for q
20 | * p only if q
21 | * q if p
22 | * q whenever p
23 | * q is necessary for p
24 |
25 | For bicondtional, p <=> q these are all the same:
26 |
27 | * p is equivalent to q is itself equivalent to.
28 | * p is necessary and sufficient for q.
29 | * p if and only if q
30 |
31 | Exercise 1
32 | -----------
33 |
34 | Analyze the logical forms of the following statements:
35 | (a) If this gas either has an unpleasant smell or is not explosive, then it
36 | isn’t hydrogen.
37 | (b) Having both a fever and a headache is a sufficient condition for George
38 | to go to the doctor.
39 | (c) Both having a fever and having a headache are sufficient conditions
40 | for George to go to the doctor.
41 | (d) If x =/ 2, then a necessary condition for x to be prime is that
42 | x be odd.
43 |
44 | (a)
45 |
46 | S = Gas has an unpleasant smell.
47 | E = Gas is explosive.
48 | H = Gas is hydrogen.
49 |
50 | S ∨ ¬E → ¬H
51 |
52 | (b)
53 |
54 | F = George is having Fever.
55 | H = George is having Headache.
56 | D = George goes to doctor.
57 |
58 | (F ∧ H) → D
59 |
60 | (c)
61 |
62 | (F ∨ H) → D
63 | ¬(F ∨ H) ∨ D
64 | (¬F ∧ ¬H) ∨ D
65 | (D ∨ ¬F) ∧ (D ∨ ¬H)
66 | (F -> D) ∧ (H -> D)
67 |
68 | (d)
69 |
70 | T = x is two.
71 | P = x is prime.
72 | O = x is odd.
73 |
74 | p -> q means Q is necessary condition for P
75 | necessary condition for x to be prime is that x be odd
76 | necessary condition for P is O
77 | O is necessary condition for P means (P -> O)
78 |
79 | ¬T -> (P -> O)
80 |
81 | Exercise 2
82 | -----------
83 |
84 | Analyze the logical forms of the following statements:
85 | (a) Mary will sell her house only if she can get a good price and find a
86 | nice apartment.
87 | (b) Having both a good credit history and an adequate down payment is a
88 | necessary condition for getting a mortgage.
89 | (c) John will kill himself, unless someone stops him. (Hint: First try to
90 | rephrase this using the words if and then instead of unless.)
91 | (d) If x is divisible by either 4 or 6, then it isn’t prime.
92 |
93 | (a)
94 |
95 | p -> q is the same as "p only if q"
96 |
97 | H = Mary will sell her house.
98 | P = Mary will get a good price.
99 | A = Mary will find a nice apartment.
100 |
101 | H -> P ∧ A
102 |
103 | (b)
104 |
105 | p -> q is the same as "q is the necessary for p"
106 |
107 | C = Having good credit history.
108 | D = Having adequate down payment.
109 | M = Getting mortgage.
110 |
111 | M -> C ∧ D
112 |
113 | (c)
114 |
115 | S = Someone stops John
116 | K = John kills himself
117 |
118 | Not being stopped by someone is a necessary and sufficient condition
119 | for John to kill himself. If he is stopped, then he cannot kill
120 | himself and if he kills himself, then he was not stopped.
121 |
122 | ¬S <-> K
123 |
124 | (d)
125 |
126 | D(x,y) = x is divisible by y
127 | P(x) = x is prime.
128 |
129 | D(x,4) ∨ D(x,6) -> ¬P(x)
130 |
131 | Exercise 3
132 | ----------
133 |
134 | Analyze the logical form of the following statement:
135 | (a) If it is raining, then it is windy and the sun is not shining.
136 | Now analyze the following statements. Also, for each statement determine
137 | whether the statement is equivalent to either statement (a) or its converse.
138 | (b) It is windy and not sunny only if it is raining.
139 | (c) Rain is a sufficient condition for wind with no sunshine.
140 | (d) Rain is a necessary condition for wind with no sunshine.
141 | (e) It’s not raining, if either the sun is shining or it’s not windy.
142 | (f) Wind is a necessary condition for it to be rainy, and so is a lack of
143 | sunshine.
144 | (g) Either it is windy only if it is raining, or it is not sunny only if it is
145 | raining.
146 |
147 | (a)
148 |
149 | R = It is raining.
150 | W = It is windy.
151 | S = Sun is shining.
152 |
153 | R -> W ∧ ¬S
154 |
155 | (b)
156 |
157 | p -> q is the same as p only if q
158 |
159 | * It is windy and not sunny only if it is raining.
160 | * If it is windy and not sunny, then it is raining.
161 |
162 | W ∧ ¬S -> R (Converse (a))
163 |
164 | (c)
165 |
166 | p -> q is the same as "p is sufficient for q"
167 |
168 | R -> W ∧ ¬S (Equivalent to (a))
169 |
170 | (d)
171 |
172 | p -> q is the same as "q is necessary for p"
173 |
174 | W ∧ ¬S -> R (Converse of (a))
175 |
176 | (e)
177 |
178 | p -> q is the same as "q if p"
179 |
180 | S ∨ ¬W -> ¬R (Equivalent to (a))
181 |
182 | (f)
183 |
184 | p -> q is the same as "q is necessary for p"
185 |
186 | (R -> W) ∧ (R -> ¬S)
187 | (¬R ∨ W) ∧ (¬R ∨ ¬S)
188 | (((¬R ∨ W) ∧ ¬R) ∨ ((¬R ∨ W) ∧ ¬S))
189 | (¬R ∨ (¬R ∧ W)) ∨ ((¬R ∨ W) ∧ ¬S)
190 | ¬R ∨ ((¬R ∨ W) ∧ ¬S)
191 | ¬R ∨ ((¬R ∧ ¬S) ∨ (¬S ∧ W))
192 | ¬R ∨ (¬S ∧ W)
193 | R -> ¬S ∧ W (Equivalent to (a))
194 |
195 |
196 | (g)
197 |
198 | p -> q is the same as "p only if q"
199 |
200 | (W -> R) ∨ (¬S -> R)
201 | (¬W ∨ R) ∨ (S ∨ R)
202 | ¬W ∨ R ∨ S ∨ R
203 | R ∨ ¬W ∨ S
204 | ¬R -> ¬W ∨ S (Converse of (a))
205 |
206 | Exercise 4
207 | -----------
208 |
209 | Use truth tables to determine whether or not the following arguments are
210 | valid:
211 | (a) Either sales or expenses will go up. If sales go up, then the boss will
212 | be happy. If expenses go up, then the boss will be unhappy. Therefore,
213 | sales and expenses will not both go up.
214 | (b) If the tax rate and the unemployment rate both go up, then there will
215 | be a recession. If the GNP goes up, then there will not be a recession.
216 | The GNP and taxes are both going up. Therefore, the unemployment
217 | rate is not going up.
218 | (c) The warning light will come on if and only if the pressure is too high and
219 | the relief valve is clogged. The relief valve is not clogged. Therefore,
220 | the warning light will come on if and only if the pressure is too
221 | high.
222 |
223 | (a)
224 |
225 | S = Sales will go up.
226 |
227 | E = Expenses will go up.
228 |
229 | B = Boss will be happy.
230 |
231 | S ∨ E
232 | S -> B
233 | E -> ¬B
234 | ---------
235 | ¬(S ∧ E)
236 |
237 | Conjunction of predicate: `(S ∨ E) ∧ (S -> B) ∧ (E -> ¬B)`
238 |
239 | Conclusion: `¬(S ∧ E)`:
240 |
241 | Truth table of `((S | E) & ((S -> B) & (E -> ~B))) -> ~(S & E)`
242 |
243 | B E S | (((S | E) & ((S -> B) & (E -> ~B))) -> ~(S & E))
244 | --------------------------------------------------------
245 | T T T | T
246 | T T F | T
247 | T F T | T
248 | T F F | T
249 | F T T | T
250 | F T F | T
251 | F F T | T
252 | F F F | T
253 |
254 | Voila, the argument is valid.
255 |
256 | (b)
257 |
258 | T = Tax rate goes up
259 |
260 | U = Unemployment rate goes up.
261 |
262 | R = There will be recession.
263 |
264 | G = GNP goes up.
265 |
266 | (T ∧ U) -> R
267 | G -> ¬R
268 | G ∧ T
269 | -------------
270 | ¬U
271 |
272 | Disjunction of premises: `((T ∧ U) -> R) ∧ (G -> ¬R) ∧ (G ∧ T)`
273 | Conclusion: `¬U`
274 |
275 | Truth table of `((T ∧ U) -> R) ∧ (G -> ¬R) ∧ (G ∧ T) -> ¬U`:
276 |
277 | G R T U | ((((T & U) -> R) & ((G -> ~R) & (G & T))) -> ~U)
278 | ----------------------------------------------------------
279 | T T T T | T
280 | T T T F | T
281 | T T F T | T
282 | T T F F | T
283 | T F T T | T
284 | T F T F | T
285 | T F F T | T
286 | T F F F | T
287 | F T T T | T
288 | F T T F | T
289 | F T F T | T
290 | F T F F | T
291 | F F T T | T
292 | F F T F | T
293 | F F F T | T
294 | F F F F | T
295 |
296 | The argument is valid.
297 |
298 | (c)
299 |
300 | W = Warning light will come
301 |
302 | P = Pressure is too high
303 |
304 | R = Relief valve is clogged.
305 |
306 | W <-> (P ∧ R)
307 | ¬R
308 | -------------
309 | W <-> P
310 |
311 | Disjunction of premises: `(W <-> (P ∧ R)) ∧ ¬R`
312 | Conclusion: `W <-> P`
313 |
314 | Truth table for `((W <-> (P ∧ R)) ∧ ¬R) -> (W <-> P)`:
315 |
316 | P R W | (((W <-> (P & R)) & ~R) -> (W <-> P))
317 | ---------------------------------------------
318 | T T T | T
319 | T T F | T
320 | T F T | T
321 | T F F | F
322 | F T T | T
323 | F T F | T
324 | F F T | T
325 | F F F | T
326 |
327 | The truth table isn't `T` for all the cases and hence the argument is invalid.
328 |
329 | Exercise 5
330 | -----------
331 |
332 | (a) Show that P ↔ Q is equivalent to (P ∧ Q) ∨ (¬P ∧ ¬Q).
333 | (b) Show that (P → Q) ∨ (P → R) is equivalent to P → (Q ∨ R).
334 |
335 | (a)
336 |
337 | P <-> Q
338 | (P -> Q) ∧ (Q -> P)
339 | (¬P ∨ Q) ∧ (¬Q ∨ P)
340 | ((¬P ∨ Q) ∧ ¬Q) ∨ ((¬P ∨ Q) ∧ P)
341 | (¬P ∧ ¬Q) ∨ (Q ∧ P)
342 | (P ∧ Q) ∨ (¬P ∧ ¬Q)
343 |
344 | (b)
345 |
346 | (P -> Q) ∨ (P -> R)
347 | (¬P ∨ Q) ∨ (¬P ∨ R)
348 | ¬P ∨ Q ∨ ¬P ∨ R
349 | ¬P ∨ (Q ∨ R)
350 | P -> (Q ∨ R)
351 |
352 | Exercise 6
353 | -----------
354 |
355 | (a) Show that (P → R) ∧ (Q → R) is equivalent to (P ∨ Q) → R.
356 | (b) Formulate and verify a similar equivalence involving (P → R) ∨
357 | (Q → R).
358 |
359 | (a)
360 |
361 | (P -> R) ∧ (Q -> R)
362 | (¬P ∨ R) ∧ (¬Q ∨ R)
363 | (¬P ∧ ¬Q) ∨ R [Distributive Law]
364 | ¬(P ∨ Q) ∨ R
365 | (P ∨ Q) -> R
366 |
367 | (b)
368 |
369 | (P -> R) ∨ (Q -> R)
370 | (¬P ∨ R) ∨ (¬Q ∨ R)
371 | ¬P ∨ ¬Q ∨ R
372 | ¬(P ∧ Q) ∨ R
373 | (P ∧ Q) -> R
374 |
375 | Exercise 7
376 | -----------
377 |
378 | (a) Show that (P → Q) ∧ (Q → R) is equivalent to (P → R) ∧
379 | [(P ↔ Q) ∨ (R ↔ Q)].
380 | (b) Show that (P → Q) ∨ (Q → R) is a tautology.
381 |
382 | (a)
383 |
384 | Truth table of `(P → Q) ∧ (Q → R)`:
385 |
386 | P Q R | ((P -> Q) & (Q -> R))
387 | -----------------------------
388 | T T T | T
389 | T T F | F
390 | T F T | F
391 | T F F | F
392 | F T T | T
393 | F T F | F
394 | F F T | T
395 | F F F | T
396 |
397 | Truth table of `(P → R) ∧ [(P ↔ Q) ∨ (R ↔ Q)]`:
398 |
399 | P Q R | ((P -> R) & ((P <-> Q) | (R <-> Q)))
400 | --------------------------------------------
401 | T T T | T
402 | T T F | F
403 | T F T | F
404 | T F F | F
405 | F T T | T
406 | F T F | F
407 | F F T | T
408 | F F F | T
409 |
410 | As seen in the truth table, they are equivalent.
411 |
412 | (b)
413 |
414 | Truth table of `(P → Q) ∨ (Q → R)`:
415 |
416 | P Q R | ((P -> Q) | (Q -> R))
417 | -----------------------------
418 | T T T | T
419 | T T F | T
420 | T F T | T
421 | T F F | T
422 | F T T | T
423 | F T F | T
424 | F F T | T
425 | F F F | T
426 |
427 | As every row in fourth column is `T`, they are tautology.
428 |
429 | Alternative Solution:
430 |
431 | (P → Q) ∨ (Q → R)
432 | (¬P ∨ Q)∨ (¬Q v R) Conditional Law
433 | (¬P ∨ R)∨ (¬Q v Q) Associative Law
434 | In the above statement (¬Q v Q) is a tautology,
435 | (¬P ∨ R)∨ (Tautology)
436 | From tautology laws, (Statement) v (tautology) is a tautology
437 | Hence the complete statement is a tautology
438 |
439 | Exercise 8
440 | -----------
441 |
442 | Find a formula involving only the connectives ¬ and → that is equivalent
443 | to P ∧ Q.
444 |
445 | Soln:
446 |
447 | P ∧ Q
448 | ¬(¬P ∨ ¬Q)
449 | ¬(P -> ¬Q)
450 |
451 | Exercise 9
452 | -----------
453 |
454 | Find a formula involving only the connectives ¬ and → that is equivalent
455 | to P ↔ Q.
456 |
457 | Soln:
458 |
459 | P <-> Q
460 | (P -> Q) ∧ (Q -> P)
461 | ¬(¬(P -> Q) ∨ ¬(Q -> P))
462 | ¬((P -> Q) -> ¬(Q -> P))
463 |
464 | Exercise 10
465 | ------------
466 |
467 | Which of the following formulas are equivalent?
468 | (a) P → (Q → R).
469 | (b) Q → (P → R).
470 | (c) (P → Q) ∧ (P → R).
471 | (d) (P ∧ Q) → R.
472 | (e) P → (Q ∧ R).
473 |
474 | (a)
475 |
476 | P → (Q → R)
477 | ¬P ∨ ¬Q ∨ R
478 |
479 | (b)
480 |
481 | Q → (P → R)
482 | ¬Q ∨ ¬P ∨ R
483 |
484 | (c)
485 |
486 | (P → Q) ∧ (P → R)
487 | (¬P ∨ Q) ∧ (¬P ∨ R)
488 | ¬P ∨ (Q ∧ R)
489 |
490 | (d)
491 |
492 | (P ∧ Q) → R
493 | ¬P ∨ ¬Q ∨ R
494 |
495 | (e)
496 |
497 | P → (Q ∧ R)
498 | ¬P ∨ (Q ∧ R)
499 |
500 | (a), (b) & (d) are equivalent.
501 | (c) & (e) are equivalent.
502 |
--------------------------------------------------------------------------------
/chapter 5/section5-3.tex:
--------------------------------------------------------------------------------
1 | %%%%%%%%%%%%%%%%%%%%%%%%%%%%%%%%%%%%%%%%%
2 | % Author: Sibi
3 | %%%%%%%%%%%%%%%%%%%%%%%%%%%%%%%%%%%%%%%%%
4 | \documentclass{article}
5 | \usepackage{graphicx}
6 | \usepackage{verbatim}
7 | \usepackage{amsmath}
8 | \usepackage{amsfonts}
9 | \usepackage{amssymb}
10 | \usepackage{tabularx}
11 | \usepackage{mathtools}
12 | \newcommand{\BigO}[1]{\ensuremath{\operatorname{O}\bigl(#1\bigr)}}
13 | \setlength\parskip{\baselineskip}
14 | \begin{document}
15 | \title{Chapter 5 (Section 5.3)}
16 | \author{Sibi}
17 | \date{\today}
18 | \maketitle
19 |
20 | % See here: http://tex.stackexchange.com/a/43009/69223
21 | \DeclarePairedDelimiter\abs{\lvert}{\rvert}%
22 | \DeclarePairedDelimiter\norm{\lVert}{\rVert}%
23 |
24 | % Swap the definition of \abs* and \norm*, so that \abs
25 | % and \norm resizes the size of the brackets, and the
26 | % starred version does not.
27 | \makeatletter
28 | \let\oldabs\abs
29 | \def\abs{\@ifstar{\oldabs}{\oldabs*}}
30 | %
31 | \let\oldnorm\norm
32 | \def\norm{\@ifstar{\oldnorm}{\oldnorm*}}
33 | \makeatother
34 | \newpage
35 |
36 | \section{Solution 1}
37 | $R^{-1}(p) = $ the person immediately right to $p.$
38 |
39 | \section{Solution 2}
40 | $F^{-1}(X) = \text{unique } Y \text{ such that} F(Y) = X$.
41 | So, \begin{align*}
42 | F(Y) = X \\
43 | A \setminus Y = X \\
44 | A \setminus X = Y \\ \\
45 | F^{-1}(X) = A \setminus X
46 | \end{align*}
47 |
48 | \section{Solution 3}
49 | We will try to find a function $g: R \to R$ such that $f \circ g =
50 | i_\mathbb{R}$ and $g \circ f = i_\mathbb{R}$.
51 |
52 | We are hoping to find $g = f^{-1}$. So, $f(x) = \text{unique }y$ such
53 | that $f^{-1}(y) = x$. So, $\frac{2x + 5}{3} = y$. Solving it, we get
54 | $x = \frac{3y-5}{2}$. So, $g(x) = \frac{3x-5}{2}$.
55 |
56 | Let's check for $f \circ g = id$.
57 | \begin{align*}
58 | f(g(x)) \\
59 | = f(\frac{3x-5}{2}) \\
60 | = \frac{3x - 5 + 5}{3} \\
61 | = x \\
62 | = id_{\mathbb{R}}(x)
63 | \end{align*}
64 |
65 | Let's check for $g \circ f = id $
66 | \begin{align*}
67 | g(f(x)) \\
68 | = g(\frac{2x + 5}{3}) \\
69 | = \frac{2x + 5 - 5}{2} \\
70 | = x \\
71 | = id_{\mathbb{R}}(x)
72 | \end{align*}
73 |
74 | From theorem 5.3.4, we can conclude that $f$ is one-to-one and onto.
75 |
76 | \section{Solution 4}
77 | We will try to find a function $g: R \to R$ such that $f \circ g =
78 | i_\mathbb{R}$ and $g \circ f = i_\mathbb{R}$.
79 |
80 | We are hoping to find $g = f^{-1}$. So, $f(x) = \text{unique }y$ such
81 | that $f^{-1}(y) = x$. So, $2x^3 - 3 = y$. Solving it, we get
82 | $x = (\frac{3+y}{2})^{1/3}$. So, $g(x) = (\frac{3+x}{2})^{1/3}$.
83 |
84 | Let's check for $f \circ g = id$.
85 | \begin{align*}
86 | f \circ g(x) \\
87 | = f((\frac{3+x}{2})^{1/3}) \\
88 | = 3 + x - 3 \\
89 | = x \\
90 | = id_R \\
91 | \end{align*}
92 |
93 | Similarly, $g \circ f = id_R$
94 |
95 | From theorem 5.3.4, we can conclude that $f$ is one-to-one and onto.
96 |
97 | \section{Solution 5}
98 | We will try to find a function $g: R^{+} \to R$ such that $f \circ g =
99 | i_\mathbb{R^{+}}$ and $g \circ f = i_\mathbb{R}$.
100 |
101 | We are hoping to find $g = f^{-1}$. So, $f(x) = \text{unique }y$ such
102 | that $f^{-1}(y) = x$.So,
103 | \begin{align*}
104 | 10^{2-x} = y \\
105 | \log_{10}10^{2-x} = \log_{10}y \\
106 | 2 - x = log_{10}y \\
107 | x = 2 - log y
108 | \end{align*}
109 |
110 | So, $g(x) = 2 - log(x)$. Let's verify $f \circ g = id_{\mathbb{R^{+}}}$
111 |
112 | \begin{align*}
113 | f \circ g(x) \\
114 | = f(2 - log x) \\
115 | = 10^{2 + log x - 2} \\
116 | = 10^{log x} \\
117 | = x \\
118 | = id_{\mathbb{R^{+}}}
119 | \end{align*}
120 |
121 | Also, let's check $g \circ f = id_R$
122 | \begin{align*}
123 | g \circ f(x) \\
124 | = g(10^{2 - x}) \\
125 | = 2 - log_{10} 10^{2-x}\\
126 | = 2 - (2 - x) \\
127 | = x \\
128 | \end{align*}
129 |
130 | From theorem 5.3.4, we can conclude that $f$ is one-to-one and onto.
131 |
132 | \section{Solution 6}
133 | \subsection{Solution 6(a)}
134 | We will try to find a $g: B \to A$ such that $f \circ g = i_A$ and $g
135 | \circ f = i_A$.
136 |
137 | $f(x) = $ unique $y$ such that $f^{-1}(y) = x$.
138 |
139 | So, $\frac{3x}{x-2} = y$ \\
140 | Solving it, we get $f^{-1}(y) = \frac{2y}{y-3}$
141 |
142 | No $g$ is undefined at $x = 3$. So, let the set $B$ be $R \setminus {3}$.
143 |
144 | Now, let's verify $f \circ g = i_B$
145 | \begin{align*}
146 | f \circ g(b) \\
147 | = f(g(b)) \\
148 | = f(\frac{2b}{b-3}) \\
149 | = \frac{6b}{b} \\
150 | = b \\
151 | = i_B(b)
152 | \end{align*}
153 |
154 | Also, $g \circ f = i_A$
155 | \begin{align*}
156 | g \circ f(a) \\
157 | = g(f(a)) \\
158 | = g(\frac{3a}{a-2}) \\
159 | = \frac{6a}{a}
160 | = a \\
161 | = i_A(a)
162 | \end{align*}
163 |
164 | \subsection{Solution 6(b)}
165 | From theorem $5.3.4$, it follows that $f^{-1}(x) = \frac{2x}{x-3}$
166 |
167 | \section{Solution 7}
168 | \subsection{Solution 7(a)}
169 | Let $a,b$ be arbitrary element in $R$.
170 |
171 | $(\Rightarrow)$ Suppose $(a,b) \in f$. It follows that
172 | $b = a + \frac{7}{5}$. So, $(a,a+\frac{7}{5}) \in f$. Applying $a$ to
173 | $f_2 \circ f_1$, we get $a + \frac{7}{5}$. So, $(a, a+\frac{7}{5}) \in
174 | f_2 \circ f_1$. So, $f \subseteq f_2 \circ f_1$.
175 | $(\Leftarrow)$ Similar to above.
176 |
177 | \subsection{Solution 7(b)}
178 | Applying $a$ to $f^{-1}$, we get $5a - 7$. Solving for inverse
179 | functions, we get:
180 | \begin{align*}
181 | f_1^{-1}(x) = x - 7 \\
182 | f_2^{-1}(x) = 5x
183 | \end{align*}
184 |
185 | Solving them:
186 | \begin{align*}
187 | f_1^{-1}(f_2^{-1}(a)) \\
188 | = f_1^{-1}(51) \\
189 | = 5a - 7
190 | \end{align*}
191 |
192 | So, yes they are same!
193 |
194 | \section{Solution 8}
195 | \subsection{Solution (a)}
196 | Let $b$ be arbitrary element of $B$. Let $a = f^{-1}(b) \in A$. Then
197 | $(b,a) \in f^{-1}$. Thus,
198 | \begin{align*}
199 | f \circ f^{-a}(b) \\
200 | = f(f^{-1}(b)) \\
201 | = f(a) \\
202 | = b \\
203 | = i_B(b)
204 | \end{align*}
205 |
206 | Since $b$ was arbitrary, we can conclude that $f \circ f^{-1} = i_B$.
207 |
208 | \subsection{Solution (b)}
209 | We know that,
210 | \begin{align*}
211 | f^{-1} \circ f = i_A \\
212 | f^{-1} \circ f \circ f^{-1} = i_A \circ f^{-1}
213 | \end{align*}
214 |
215 | Let $b$ be arbitrary element in $B$. Suppose $f(a) = b$. Then $(a,b)
216 | \in f$ and $(b,a) \in f^{-1}$. So,
217 | \begin{align*}
218 | f^{-1} \circ (f \circ f^{-1})(b) = i_A \circ f^{-1}(b) \\
219 | f^{-1}(b) = i_A(a) \\
220 | f^{-1}(b) = a \\
221 | f \circ f^{-1}(b) = f(a) \tag{Apply f} \\
222 | f \circ f^{-a} (b) = b \\
223 | f \circ f^{-a} (b) = i_B(b)
224 | \end{align*}
225 | Since $b$ was arbitrary, we can conclude that $f \circ f^{-1} = i_B$.
226 |
227 | \section{Solution 9}
228 | Suppose $g: B \to A$ and $f \circ g = i_B$. Let $b$ be arbitrary
229 | element in $B$. Then $f \circ g(b) = i_B(b)$. Since $g$ is a function
230 | $\exists a \in A$ such that $g(b) = a$. So, $f(a) = b$. Since $b$ was
231 | arbitrary, we can conclude that $f$ is onto.
232 |
233 | \section{Solution 10}
234 | Suppose $f: A \to B, g: B \to A, g \circ f = i_A$ and $f \circ g =
235 | i_B$. Let $(b,a)$ be arbitrary element of $B \times A$ such that
236 | $(b,a) \in g$. Then $f \circ g(b) = f(a)$. From $f \circ g = i_B$, it
237 | follows that $f(a) = b$. So, $(a,b) \in f^{-1}$. So, $g \subseteq
238 | f^{-1}$. Similarly, do the other case with $g \circ f = i_A$ and show
239 | $f^{-1} \subseteq g$.
240 |
241 | \section{Solution 11}
242 | \subsection{Solution (a)}
243 | Suppose $f:A \to B$ and $g:B \to A$. Suppose $f$ is one to one and $f
244 | \circ g = i_B$. From theorem $5.3.3$, it follows that $f$ is onto. Now
245 | from $5.3.4$, it follows that $g \circ f = i_A$. Now from theorem
246 | $5.3.5$, we can conclude that $g = f^{-1}$.
247 |
248 | \subsection{Solution (b)}
249 | Suppose $f:A \to B$ and $g:B \to A$. Suppose $f$ is onto and $g \circ
250 | f = i_A$. From theorem, $5.3.3$, it follows that $f$ is one to one.
251 | From $5.3.4$, it follows that $f \circ g = i_B$. From $5.3.5$, it
252 | follows that $g = f^{-1}$
253 |
254 | \subsection{Solution (c)}
255 | Suppose $f \circ g = i_B$ and $g \circ f \neq i_A$.
256 |
257 | \subsubsection{Solution (i)}
258 | From theorem $5.3.3$, it follows that $f$ is onto.
259 |
260 | \subsubsection{Solution (ii)}
261 | We know that $g \circ f \neq i_A$. So, $g \neq f^{-1}$. Applying
262 | contrapositive law to $(a)$, we get $g \neq f^{-1} \implies f \circ g
263 | \neq i_B \lor f \text{ is not one to one}$. We know that $f \circ g =
264 | i_B$. So $f$ is not one to one.
265 |
266 | Rest of the proofs are based on the above technique.
267 |
268 | \section{Solution 12}
269 | Suppose $f:A \to B$ and $f$ is one to one. Let $B^{-1} = Ran(f)$. Let
270 | $b$ be an arbitrary element in $B^{-1}$.
271 |
272 | Existence proof. Since $b \in Ran(f)$, $\exists a \in A$ such that
273 | $f(a) = b$. So, $(b,a) \in f^{-1}$.
274 |
275 | Uniqueness proof. Let $(b,a_1) \in f^{-1}$ and $(b,a_2) \in f^{-1}$.
276 | It follows that $(a_1, b) \in f$ and $(a_2, b) \in f$. Since $f$ is
277 | one to one, it follows that $a_1 = a_2$.
278 |
279 | \section{Solution 13}
280 | Suppose $f: A \to B$ and $f$ is onto. Let $R = \{(x,y) \in A \times A
281 | \mid f(x) = f(y) \}$
282 |
283 | \subsection{Solution (a)}
284 | Let $X$ be an arbitrary element in $A/R$. Then it follows that there
285 | is some $x \in A$ such that $x \in X$.
286 |
287 | Existence. Since $x \in A$ and $f: A \to B$, it follows that $\exists
288 | b \in B$ such that $f(x) = b$. So, $(X,b) \in h$.
289 |
290 | Uniqueness. Suppose $(X,b_1) \in h$ and $(X, b_2) \in h$. From $(X,
291 | b_1) \in h$, it follows that $\exists x \in X$ such that $f(x)=b_1$.
292 | Similarly from $(X,b_1) \in h$, it follows that $\exists y \in X$ such
293 | that $f(y)=b_1$. We know that $xRy$. From the definition of $R$, it
294 | follows that $f(x) = f(y)$. So, $b_1 = b_2$.
295 |
296 | \subsection{Solution (b)}
297 | From $R$, it follows that $\forall x \in A \forall y \in A(xRy \iff
298 | f(x) = f(y))$. From exercise 18 of 5.1, it follows that $h$ is one to
299 | one.
300 |
301 | Onto proof. Let $b$ be arbitrary element in $B$. Since $f$ is onto, it
302 | follows that $\exists a \in A$ such that $f(a)=b$. Since $a \in A$,
303 | there is some $X \in A/R$ such that $a \in X$. We know that $h(X)=b$.
304 | Since $b$ was arbitrary, we can conclude that $h$ is onto.
305 |
306 | \subsection{Solution (c)}
307 | Let $b$ be an arbitrary element in $B$.
308 | $(\Rightarrow)$ Suppose $a \in h^{-1}(b)$. It follows that $a \in A$
309 | and $b \in B$. All we have to prove is $f(a) = b$. Since $f$ is onto,
310 | it follows that $f(a) =b$. So, $a \in \{x \in A \mid f(x) = b\}$.
311 |
312 | $(\Leftarrow)$ Let $a \in \{x \in A \mid f(x) = b\}$. It follows that
313 | $f(a)=b$. Now, we know that $a \in A$ so there is some $X \in A/R$
314 | such that $a \in X$. So, $h(X) = b$ or $h^{-1}(b) = X$. So, $a \in h^{-1}(b)$.
315 |
316 | \subsection{Solution (d)}
317 | The question is likely wrong. Specifically the statement $\forall b
318 | \in B(g(b) \in h(b))$. The Domain of $h$ is $A/R$ and $b \notin A/R$.
319 |
320 | \section{Solution 14}
321 | \subsection{Solution (a)}
322 | Let $x$ be an arbitrary element in $A'$. $g \circ f(x) = g(f(x))$.
323 | There is some $b \in B$ such that $g(b) = x$. So, $g(f(x)) =
324 | g(f(g(b))) = g(b) = x$. Since $x$ was arbitrary, $\forall x \in A'(g
325 | \circ f)(x) = x$.
326 |
327 | \subsection{Solution (b)}
328 | $g$ function can be written as $B \to A'$. We know that $(f \mid A' \circ
329 | g) = i_B$. From (a), it follows that $(g \circ f \mid A') = i_A$. From
330 | theorem 5.3.4, it follows that $f \mid A'$ is one to one and onto
331 | function. From theorem 5.3.5, it follows that $g=(f \mid A)^{-1}$.
332 |
333 | \section{Solution 15}
334 | We know from Example $5.3.6$ that $f \circ g = i_B$. Since $g(x)=
335 | \sqrt{2}$, it follows that $Ran(g) = R^{+} = B$. So, $g$ function can
336 | be written as $g: B \to B$. It follows that $(f \mid B \circ g) =
337 | i_B)$. From 14 (a), it follows that $(g \circ f \mid B) = i_B$. From
338 | 14 (b), it follows that $g = (f \mid B)^{-1}$.
339 |
340 | \section{Solution 16}
341 |
342 | \subsection{Solution (a)}
343 | $y \in Ran(f)$, if $y = 4x - x^2$. Forming an equation, we get
344 | $x^2 - 4x + y = 0$. The equations will be real only if
345 | $b^2 - 4ac >= 0$ on a standard equation $a + bx^2 + c$. So,
346 | \begin{align*}
347 | (-4)^2 - 4.1.y >= 0 \\
348 | 16 - 4y >= 0 \\
349 | 16 >= 4y \\
350 | 4 >= y \\
351 | B = \{x \mid 4 >= y \}
352 | \end{align*}
353 |
354 |
355 | \subsection{Solution (b)}
356 | From 14, we know that we have to find a $g: R \to R$ such that $f
357 | \circ g = i_R$. Then $A = Ran(g)$. Let's find $g$.
358 | \begin{align*}
359 | f \circ g(x) = i_R(x) \\
360 | f(g(x)) = x \\
361 | \end{align*}
362 | Let $g(x) = y$. Then $f(y) = x$. So, $y^2 - 4y + x = 0$.
363 | $y = 2 + \sqrt{4-x}$. So, $g(x) = 2 + \sqrt{4 - x}$. We have to
364 | confirm that $g \circ f = i_R$.
365 | \begin{align*}
366 | g \circ f(x) = i_R(x) \\
367 | g(4x - x^2) = 2 + \sqrt{4 - 4x + x^2} \\
368 | = 2 + \sqrt{(x - 2)^2} \\
369 | = 2 + x - 2 \\
370 | = x
371 | \end{align*}
372 |
373 | So, $f \mid A$ is one to one and onto function and $(f \mid A)^{-1}
374 | (x) = 2 + \sqrt{4 - x}$.
375 | \end{document}
376 |
--------------------------------------------------------------------------------
/chapter 4/section4-2.tex:
--------------------------------------------------------------------------------
1 | %%%%%%%%%%%%%%%%%%%%%%%%%%%%%%%%%%%%%%%%%
2 | % Author: Sibi
3 | %%%%%%%%%%%%%%%%%%%%%%%%%%%%%%%%%%%%%%%%%
4 | \documentclass{article}
5 | \usepackage{graphicx}
6 | \usepackage{verbatim}
7 | \usepackage{amsmath}
8 | \usepackage{amsfonts}
9 | \usepackage{amssymb}
10 | \usepackage{tabularx}
11 | \setlength\parskip{\baselineskip}
12 | \begin{document}
13 | \title{Chapter 4 (Section 4.2)}
14 | \author{Sibi}
15 | \date{\today}
16 | \maketitle
17 | \newpage
18 |
19 | \section{Problem 1}
20 | Solution (a)
21 |
22 | Domain = $\{ p \mid \text{p is a living parent} \}$
23 | Range = $\{ q \mid \text{q is a living child} \}$
24 |
25 | Solution(b)
26 |
27 | Domain = $\mathbb{R}$
28 | Range = $\mathbb{R^+}$
29 |
30 | \section{Problem 2}
31 |
32 | Solution(a)
33 |
34 | Domain = $\{ p \mid \text{p is a brother to some living person }\}$
35 | Range = $\{q \mid \text{q has some living brother.}\}$
36 |
37 | Solution(b)
38 |
39 | Domain = $\mathbb{R}$
40 | Range = $\{ x \mid x \in \mathbb{R} \land x > -1 \land x < 1 \}$
41 |
42 | \section{Problem 3}
43 |
44 | \subsection{Solution (a)}
45 | \noindent
46 |
47 | $L = \{(s,r) \in S \times R \mid \text{the student s lives in the dorm room
48 | r}\}$
49 |
50 | $L^{-1} = \{(r,s)\} \in R \times S | (s,r) \in L\}$
51 |
52 | $L^{-1} \circ L \text{ is relation on} S$
53 |
54 | $L^{-1} \circ L = \{(s,t) \in S \times S \mid \exists r \in R((s,r) \in L \land
55 | (r,t) \in L^{-1})\}$
56 |
57 | $ = \{(s,t) \in S \times S \mid \exists r \in R (\text{ student s lives in
58 | dorm room r and so is student t})\}$
59 |
60 | $ = \{(s,t) \in S \times S \mid \text{student s and t both live in some dorm
61 | room} \}$
62 |
63 |
64 | \subsection{Solution b}
65 | \noindent
66 |
67 | $E \circ (L^{-1} \circ L)$ is relation from S to C.
68 |
69 | $E \circ (L^{-1} \circ L) = \{(t,c) \in S \times C \mid \exists s \in
70 | S((t,s) \in (S \times S) \land (s,c) \in S \times C\}$
71 |
72 | \begin{multline}
73 | =\{(t,c) \in S \times C \mid \exists s \in S( \text{student s and t} \\
74 | \text{both live in some dorm room and student s is enrolled in course c)}\}
75 | \end{multline}
76 |
77 | $=\{(t,c) \in S \times C \mid \text{ t is a student who shares the
78 | dorm \\ with another student who is enrolled in course c}\}$
79 |
80 | \section{Problem 4}
81 |
82 | \subsection{Solution (a)}
83 | \noindent
84 |
85 | $S \circ R = \{(a,b) \in A \times B \mid \exists c \in B((a,c) \in R
86 | \land (c,b) \in S)$
87 |
88 | $ = {(1,5), (1,6), (1,4), (2,4), (3,6)}$
89 |
90 | \subsection{Solution (b)}
91 | \noindent
92 |
93 | $S \circ S^{- 1} = \{(a,b) \in B \times B \mid \exists c \in B((a,c)
94 | \in S^{-1} \land (c,b) \in S\}$
95 |
96 | $S \circ S^{- 1} = \{(a,b) \in B \times B \mid \exists c \in B((c,a)
97 | \in S \land (c,b) \in S\}$
98 |
99 | $ = \{(5,6),(6,5),(5,5),(6,6),(4,4)\}$
100 |
101 | \section{Problem 5}
102 |
103 | \subsection{Solution (a)}
104 | \noindent
105 | $S^{-1} \circ R = \{(a,b) \in A \times B \mid \exists c \in C((a,c)
106 | \in R \land (c,b) \in S^{-1}\}$
107 |
108 | $S^{-1} \circ R = \{(a,b) \in A \times B \mid \exists c \in C((a,c)
109 | \in R \land (b,c) \in S\}$
110 |
111 | = \{(1,4), (3,4), (3,5)\}
112 |
113 | \subsection{Solution (b)}
114 | \noindent
115 | $R^{-1} \circ S = \{(b,a) \in B \times A \mid \exists c \in C((b,c)
116 | \in S \land (c,a) \in R^{-1})\}$
117 |
118 | $= \{(b,a) \in B \times A \exists c \in C((b,c) \in S \land (a,c) \in R\}$
119 |
120 | $={(4,1),(4,3),(5,3)}$
121 |
122 | \section{Problem 6}
123 |
124 | \subsection{Solution (a)}
125 |
126 | Note that both $Dom(R)$ and $Ran(R^{-1})$ are subsets of $A$. Let $a$
127 | be an arbitrary element of $Dom(R)$.
128 |
129 | \begin{align*}
130 | a \in Dom (R) \iff \exists b \in B ((a,b) \in R) \\
131 | \iff \exists b \in B((b,a) \in R^{-1}) \\
132 | \iff a \in Ran(R^{-1})
133 | \end{align*}
134 |
135 | \subsection{Solution (b)}
136 |
137 | We know that $Dom(R^{-1}) = Ran(R)$. So, it follows that
138 | $Dom((R^{-1})^{-1}) = Ran (R^{-1})$. From, $(R^{-1})^{-1} = R$, we can
139 | conclude that $Dom (R) = Ran (R^{-1})$.
140 |
141 | \subsection{Solution (c)}
142 |
143 | Suppose $(a,d)$ be an arbitrary element in $(T \circ S) \circ R$. By
144 | the definition of composition, we can chose some $b \in B$ such that
145 | $(a,b) \in R$ and $(b,d) \in T \circ S$. Similarly from $T \circ S$,
146 | we can chose some $c \in C$ such that $(b,c) \in S$ and $(c,d) \in T$.
147 | From $(a,b) \in R$ and $(b,c) \in S$, it follows that $(a,c) \in S
148 | \circ R$. Also, from $(a,c) \in S \circ R$ and $(c,d) \in T$, it
149 | follows that $(a,d) \in T \circ (S \circ R)$.
150 |
151 | \subsection{Solution(d)}
152 | \noindent
153 |
154 | $(\Rightarrow)$ Let $(c,a)$ be an arbitrary element in $(S \circ
155 | R)^{-1}$. Then it follows that $(a,c) \in (S \circ R)$. From the
156 | definition of composition, it follows that there exists some $b \in B$
157 | such that $(a,b) \in R$ and $(b,c) \in S$. It follows that $(b,a) \in
158 | R^{-1}$ and $(c,b) \in S^{-1}$. From that we can conclude that $(c,a)
159 | \in R^{-1} \circ S^{-1}$.
160 |
161 | $(\Leftarrow)$ Let $(c,a)$ be an arbitrary element in $R^{-1} \circ
162 | S^{-1}$. From the definition of composition, it follows that there
163 | exists some $b \in B$ such that $(c,b) \in S^{-1}$ and $(b,a) \in
164 | R^{-1}$. It follows that $(b,c) \in S$ and $(a,b) \in R$. Composing,
165 | them we get $(a,c) \in S \circ R$. So, $(c,a) \in (S \circ R)^{-1}$.
166 |
167 | \section{Problem 7}
168 |
169 | An enemy of one's enemy is one's friend \\
170 | An enemy of John's enemy is John's friend
171 |
172 | Assume: John's Enemy is Josepth, so $(Joseph, John) \in E$
173 | Joseph's Enemy is Jane, so $(Jane, Joseph) \in E$
174 |
175 | Now, Jane and John are friend. So, $(Jane, John) \in F$
176 | $E \circ E = \{(Jane, Joseph)\}$
177 |
178 | $E \circ E \subseteq F$
179 |
180 | \section{Problem 8}
181 | \subsection{Solution(a)}
182 | Let $a$ be an arbitrary element in $Dom(S \circ R)$. Thenit follows
183 | that there exists some element $c \in C$ such that $(a,c) \in S \circ
184 | R$. From the definition of composition, it follows that there exists
185 | some element $b \in B$ such that $(a,b) \in R$ and $(b,c) \in S$. So,
186 | it follows that for some $b$, $(a,b) \in R$. Then clearly, $a \in
187 | Dom(R)$. Since $a$ is arbitrary, we can conclude that $Dom(S \circ R)
188 | \subseteq Dom(R)$.
189 |
190 | \subsection{Solution (b)}
191 |
192 | Suppose $Ran (R) \subseteq Dom(S)$.
193 |
194 | $(\Rightarrow)$ Let $a$ be an arbitrary element in $Dom(S \circ R)$.
195 | Then it follows that for some $c \in C$, $(a,c) \in S \circ R$. From
196 | the definition of composition it follows that there exists some $b \in
197 | B$ such that $(a,b) \in R$ and $(b,c) \in S$. From $(a,b) \in R$, it
198 | follows that $a \in Dom(R)$.
199 |
200 | $(\Leftarrow)$ Let $a$ be an arbitrary element in $Dom(R)$. Then it
201 | follows that for some element $b \in B$ there exists $(a,b) \in R$.
202 | From $Ran(R) \subseteq Dom(S)$, it follows that $b \in Dom(S)$. So,
203 | there exists some $c \in C$ such that $(b,c) \in S$. From $(a,b) \in
204 | R$ and $(b,c) \in S$, it follows that $(a,c) \in S \circ R$. Therefore
205 | $a \in Dom(S \circ R)$.
206 |
207 | \subsection{Solution (c)}
208 |
209 | (a) $Dom(S \circ R) \subseteq Dom(R)$ \\
210 | $ \iff Ran((S \circ R)^{-1}) \subseteq Ran(R^{-1})$ \\
211 | $ \iff Ran(R^{-1} \circ S^{-1}) \subseteq Ran(R^{-1})$
212 |
213 | (b) $Ran(R) \subseteq Dom(S) \implies Dom(S \circ R) = Dom(R)$ \\
214 | $ \iff Dom(R^{-1}) \subseteq Ran(S^{-1}) \implies Ran((S \circ
215 | R)^{-1}) = Ran(R^{-1})$ \\
216 | $ \iff Dom(R^{-1}) \subseteq Ran(S^{-1}) \implies Ran(R^{-1} \circ
217 | S^{-1}) = Ran(R^{-1})$
218 |
219 | \section{Problem 9}
220 |
221 | \subsection{Solution (a)}
222 |
223 | Let $(a,b)$ be an arbitrary element in $R$. Then it follows that $a
224 | \in Dom(R)$ and $b \in Ran(R)$. So, clearly $(a,b) \in Dom(R) \times Ran(R)$.
225 |
226 | \subsection{Solution(b)}
227 |
228 | Suppose $R \subseteq S$. Let $(b,a)$ be an arbitrary element in
229 | $R^{-1}$. It follows that $(a,b) \in R$. From $R \subseteq S$, it
230 | follows that $(a,b) \in S$. So, $(b,a) \in S^{-1}$. Since $(b,a)$ is
231 | an arbitrary element, it follows that $R^{-1} \subseteq S^{-1}$.
232 |
233 | \subsection{Solution (c)}
234 |
235 | $(\Rightarrow)$ Let $(b,a)$ be an arbitrary element in $(R \cup
236 | S)^{-1}$. It follows that $(a,b) \in R \cup S$. Let us consider the
237 | cases separately:
238 |
239 | Case 1. $(a,b) \in R$ It follows that $(b,a) \in R^{-1}$. So $(b,a)
240 | \in R^{-1} \cup S^{-1}$.
241 | Case 2. $(a,b) \in S$ It follows that $(b,a) \in S^{-1}$. So $(b,a)
242 | \in R^{-1} \cup S^{-1}$.
243 |
244 | $(\Leftarrow)$ Let $(b,a)$ be an arbitrary element in $R^{-1} \cup
245 | S^{-1}$. Let us consider the cases separately:
246 |
247 | Case 1. $(b,a) \in R^{-1}$ It follows that $(a,b) \in R$. So $(a,b)
248 | \in R \cup S$. Therefore $(b,a) \in (R \cup S)^{-1}$.
249 | Case 2. $(b,a) \in S^{-1}$ It follows that $(a,b) \in S$. So, $(a,b)
250 | \in R \cup S$ Therefore $(b,a) \in (R \cup S)^{-1}$
251 |
252 | \section{Problem 10}
253 |
254 | We will prove contrapositive in both directions.
255 |
256 | $(\Rightarrow)$ Suppose $(Ran(R) \cap Dom(S)) \neq \emptyset$. Then by
257 | existential instantiation, $b \in Ran(R)$ and $b \in Dom(S)$. From $b
258 | \in Ran(R)$, it follows that there exists some $a$ such that $(a,b)
259 | \in R$. Similarly, $(b,c) \in S$. From $(a,b)in B$ and $(b,c) \in S$,
260 | it follows that $(a,c) \in S \circ R$. So $S \circ R \neq \emptyset$
261 |
262 | $(\Leftarrow)$ Suppose $S \circ R \neq \emptyset$. Then it follows
263 | that there exists some element $(a,c)$ in $S \circ R$. From the
264 | definition of composition, it follows that $(a,b) \in R$ and $(b,c)
265 | \in S$ for some $b \in B$. From $(a,b) \in R$, it follows that $b \in
266 | Ran(R)$. Similarly $b \in Dom(S)$. Therefore, $b \in Ran(R) \cap
267 | Dom(S)$. So, $Ran (R) \cap Dom(S) \neq \emptyset$
268 |
269 | \section{Problem 11}
270 | \subsection{Solution (a)}
271 | Let $(a,c)$ be an arbitrary element in $(S \circ R) \setminus (T \circ
272 | R)$. It follows that $(a,c) \in S \circ R$ and $(a,c) \notin T \circ
273 | R$. From $(a,c) \in S \circ R$, it follows that there is some $b \in
274 | B$ such that $(a,b) \in R$ and $(b,c) \in S$. From $(a,c) \notin T
275 | \circ R$, it follows that there is some $b \in B$ such that $(a,b)
276 | \notin R \land (b,c) \notin T$. From $(b,c) \in S$ and $(b,c) \notin
277 | T$, it follows that $(b,c) \in S \setminus T$. From $(a,b) \in R$, we
278 | can conclude that $(a,c) \in (S \setminus T) \circ R$.
279 |
280 | \subsection{Solution (b)}
281 | $b$ is not arbitrary there. There can be some other $d in D$ such that
282 | $(a,d) \in R$ and $(d,c) \in T$ making $(a,c) \in T$.
283 |
284 | \subsection{Solution (c)}
285 | Trick: Trace the wrong proof to find out counterexamples!
286 | \begin{align*}
287 | R = \{(1,3),(1,4)\} \\
288 | S = \{(3,2)\} \\
289 | T = \{(4,2)\} \\ \\
290 | S \setminus T = \{(3,2)\} \\
291 | (S \setminus T) \circ R = \{(1,2)\} \\ \\
292 | S \circ R = \{(1,2)\} \\
293 | T \circ R = \{(1,2)\} \\
294 | (S \circ R) \setminus (T \circ R) = \emptyset
295 | \end{align*}
296 |
297 | \section{Problem 12}
298 |
299 | \subsection{Solution (a)}
300 |
301 | Suppose $S \subseteq T$. Let $(a,c)$ be an arbitrary element in $S
302 | \circ R$. By existential instantiation, it follows that $(a,b) \in R$
303 | and $(b,c) \in S$. From $S \subseteq T$, it follows that $(b,c) \in
304 | T$. From $(a,b) \in R$ and $(b,c) \in T$ it follows that $(a,c) \in T
305 | \circ R$. Since $(a,c)$ is an arbitrary element, we can conclude that
306 | $S \circ R \subseteq T \circ R$. Therefore $S \subseteq T \implies S
307 | \circ R \subseteq T \circ R$.
308 |
309 | \subsection{Solution (b)}
310 |
311 | Let $(a,c)$ be an arbitrary element in $(S \cap T) \circ R$. By
312 | existential instantiation, $(a,b) \in R$ and $(b,c) \in S \cap T$.
313 | From $(b,c) \in S \cap T$, we get $(b,c) \in T$. From $(b,c) \in S
314 | \cap T$, we get $(b,c) \in S$ and $(b,c) \in T$. From $(a,b) \in R$
315 | and $(b,c) \in S$, it follows that $(a,c) \in S \circ R$. Similarly
316 | from $(a,b) \in R$ and $(b,c) \in T$, it follows that $(a,c) \in T
317 | \circ R$. So, $(a,c) \in (S \circ R) \cap (T \circ R)$. Since $(c,c)$
318 | is an arbitrary element, we can conclude that $(S \cap T) \circ R
319 | \subseteq (S \circ R) \cap (T \circ R)$
320 |
321 | \subsection{Solution (c)}
322 | \begin{align*}
323 | R = \{(2,1),(2,4)\} \\
324 | S = \{(1,1)\} \\
325 | T = \{(4,1)\} \\ \\
326 | S \cap T = \emptyset \\
327 | (S \cap T) \circ R = \emptyset \\ \\
328 | S \circ R = \{(2,1)\} \\
329 | T \circ R = \{(2,1)\} \\
330 | (S \circ R) \cap (T \circ R) = \{(2,1)\}
331 | \end{align*}
332 |
333 | \subsection{Solution (d)}
334 |
335 | $(\Rightarrow)$ Let $(a,c)$ be an arbitrary element in $(S \cup T)
336 | \circ R$. By existential instantiation, $(a,b) \in R$ and $(b,c) \in S
337 | \cup T$. From $(b,c) \in S \cup T$, it follows that either $(b,c) \in
338 | S$ or $(b,c) \in T$. Let us consider the cases separately:
339 |
340 | Case 1. $(b,c) \in S$ From $(a,b) \in R$, it follows that $(a,c) \in S
341 | \circ R$. So $(a,c) \in (S \circ R) \cup (T \circ R)$.
342 | Case 2. $(b,c) \in T$ From $(a,b) \in R$, it follows that $(a,c) \in T
343 | \circ R$. So $(a,c) \in (S \circ R) \cup (T \circ R)$.
344 |
345 | $(\Leftarrow)$ Let $(a,c)$ be an arbitrary element in $(S \circ R)
346 | \cup (T \circ R)$. Let us consider the cases separately:
347 |
348 | Case 1. $(a,c) \in S \circ R$. By existential instantiation, it
349 | follows that $(a,b) \in R$ and $(b,c) \in S$. So, $(b,c) \in S \cup
350 | T$. From $(a,b) \in R$, it follows that $(a,c) \in (S \cup T) \circ R$.
351 | Case 2. $(a,c) in T \circ R$ By existential instantiation, it follows
352 | that $(a,b) \in R$ and $(b,c) \in T$. So, $(b,c) \in S \cup T$. From
353 | $(a,b) \in R$, it follows that $(a,c) \in (S \cup T) \circ R$.
354 |
355 | \end{document}
356 |
--------------------------------------------------------------------------------
/chapter 1/chapter1-2.md:
--------------------------------------------------------------------------------
1 | Truth Tables
2 | -------------
3 |
4 | Exercise 1
5 | -----------
6 |
7 | a)
8 |
9 |
10 | p q | (¬p ∨ q)
11 | --------------
12 | T T | T
13 | T F | F
14 | F T | T
15 | F F | T
16 |
17 |
18 | b)
19 |
20 | g s | ((s ∨ g) ∧ (¬s ∨ ¬g))
21 | ---------------------------
22 | T T | F
23 | T F | T
24 | F T | T
25 | F F | F
26 |
27 | Exercise 2
28 | -----------
29 |
30 | a)
31 |
32 | p q | ¬(p ∧ (q ∨ ¬p))
33 | ---------------------
34 | T T | F
35 | T F | T
36 | F T | T
37 | F F | T
38 |
39 | b)
40 |
41 | p q r | ((p ∨ q) ∧ (¬p ∨ r))
42 | ----------------------------
43 | T T T | T
44 | T T F | F
45 | T F T | T
46 | T F F | F
47 | F T T | T
48 | F T F | T
49 | F F T | F
50 | F F F | F
51 |
52 | Exercise 3
53 | -----------
54 |
55 | a)
56 |
57 | p q | p + q
58 | T T | F
59 | T F | T
60 | F T | T
61 | F F | F
62 |
63 | b)
64 |
65 | Find a formula using only the connectives ∧, ∨, and ¬ that is equiv-
66 | alent to P + Q. Justify your answer with a truth table.
67 |
68 | Now in the above truth table (3 (a)) see for which rows you are
69 | getting True and write down their logical connective:
70 |
71 | (p ∧ ¬q) ∨ (¬p ∧ q)
72 |
73 | p q | ((p & ~q) | (~p & q))
74 | ---------------------------
75 | T T | F
76 | T F | T
77 | F T | T
78 | F F | F
79 |
80 | Exercise 4
81 | ----------
82 |
83 | Find a formula using only the connectives ∧ and ¬ that is equivalent to
84 | P ∨ Q. Justify your answer with a truth table.
85 |
86 | The truth table for `P v Q` follows like this:
87 |
88 | p q | (p v q)
89 | -------------
90 | T T | T
91 | T F | T
92 | F T | T
93 | F F | F
94 |
95 | From De-morgan's theorem:
96 |
97 | ¬(P v Q) ≡ ¬P ∧ ¬Q
98 | (P v Q) ≡ ¬(¬P ∧ ¬Q)
99 |
100 | Exercise 5
101 | ----------
102 |
103 | (a)
104 |
105 | p q | p ↓ q
106 | T T | F
107 | T F | F
108 | F T | F
109 | F F | T
110 |
111 | (b)
112 |
113 | ¬p ∧ ¬q
114 |
115 | (c)
116 |
117 | Find formulas using only the connective ↓ that are equivalent to ¬P,
118 | P ∨ Q, and P ∧ Q.
119 |
120 | p ↓ p
121 | ¬(p ↓ q) ≡ (p ↓ q) ↓ (p ↓ q)
122 | ¬p ↓ ¬q ≡ (p ↓ p) ↓ (q ↓ q)
123 |
124 | Exercise 6
125 | ----------
126 |
127 | Some mathematicians write P | Q to mean “P and Q are not both true.”
128 | (This connective is called nand, and is used in the study of circuits in
129 | computer science.)
130 | (a) Make a truth table for P | Q.
131 | (b) Find a formula using only the connectives ∧, ∨, and ¬ that is equiv-
132 | alent to P | Q.
133 | (c) Find formulas using only the connective | that are equivalent to ¬P,
134 | P ∨ Q, and P ∧ Q.
135 |
136 | (a)
137 |
138 | p q | p | q
139 | T T | F
140 | T F | T
141 | F T | T
142 | F F | T
143 |
144 | (b)
145 |
146 | One of these:
147 |
148 | ¬(p ∧ q) (p | q ≡ ¬(p ∧ q))
149 | ¬p ∨ ¬q (p | q ≡ ¬p ∨ ¬q)
150 |
151 | (c)
152 |
153 | ¬p ≡ (p | p)
154 | p ∨ q ≡ (¬p | ¬q) ≡ ((p | p) | (q | q))
155 | p ∧ q ≡ ¬(p | q) ≡ (p | q) | (p | q)
156 |
157 |
158 | Exercise 7
159 | -----------
160 |
161 | Use truth tables to determine whether or not the arguments in exercise 7
162 | of Section 1.1 are valid.
163 |
164 | Take the conjunction of all the premises `¬(J ∧ P) ∧ (P ∨ C) ∧ J` and
165 | draw it's truth table.
166 |
167 | C J P | (~(J & P) & ((P | C) & J))
168 | ----------------------------------
169 | T T T | F
170 | T T F | T
171 | T F T | F
172 | T F F | F
173 | F T T | F
174 | F T F | F
175 | F F T | F
176 | F F F | F
177 |
178 | where
179 |
180 | J = Jane will win the math prize
181 | P = Pete will win the math prize
182 | C = Pete will win the chemistry prize.
183 |
184 | Now, the result is True when `C` and `J` are true. Hence the
185 | conclusion that Pete will win the chemistry prize is true.
186 |
187 | (b)
188 |
189 | Some symbols:
190 |
191 | B = Beef will be the main course.
192 | F = Fish will be the main course.
193 | P = Peas will be the vegetable.
194 | C = Corn will be the vegetable.
195 |
196 | Take the conjunction of all the premises `(B ∨ F) ∧ (P ∨ C) ∧ ¬(F ∧
197 | C)` and draw it's truth table:
198 |
199 | B C F P | ((B | F) & ((P | C) & ~(F & C)))
200 | ------------------------------------------
201 | T T T T | F
202 | T T T F | F
203 | T T F T | T
204 | T T F F | T
205 | T F T T | T
206 | T F T F | F
207 | T F F T | T
208 | T F F F | F
209 | F T T T | F
210 | F T T F | F
211 | F T F T | F
212 | F T F F | F
213 | F F T T | T
214 | F F T F | F
215 | F F F T | F
216 | F F F F | F
217 |
218 | Conclusion is `¬(B ∧ P)`:
219 |
220 | B P | ~(B & P)
221 | --------------
222 | T T | F
223 | T F | T
224 | F T | T
225 | F F | T
226 |
227 | The argument is invalid because `~(B & P)` is `F` when `B = T` and `P
228 | = T`. But in third row when `B = T` and `P = T` in conjunction of
229 | premise, it is `T`. To interpret it literally, they can have Beef as
230 | main course and Peas as the vegetable, but the conclusion of the
231 | statement says otherwise.
232 |
233 | (c)
234 |
235 | Symbols:
236 |
237 | J = John is telling the truth.
238 | B = Bill is telling the truth.
239 | S = Sam is telling the truth.
240 |
241 | Premises:
242 | J ∨ B
243 | ¬S ∨ ¬B
244 |
245 | Conclusion:
246 | J ∨ ¬S
247 |
248 | Conjunction of Premises: `(J ∨ B) ∧ (¬S ∨ ¬B)`
249 |
250 | B J S | ((J | B) & (~S | ~B))
251 | -----------------------------
252 | T T T | F
253 | T T F | T
254 | T F T | F
255 | T F F | T
256 | F T T | T
257 | F T F | T
258 | F F T | F
259 | F F F | F
260 |
261 | Conclusion truth table:
262 |
263 | J S | (J | ~S)
264 | --------------
265 | T T | T
266 | T F | T
267 | F T | F
268 | F F | T
269 |
270 | Whenever the conclusion truth table is `T`, for those parameters of
271 | `J` and `S`, the premises truth table is also true which implies that
272 | the conlusion is valid.
273 |
274 | (d)
275 |
276 | Symbols:
277 |
278 | S = Sales will go up.
279 | B = Boss will be happy.
280 | E = Expenses will go up.
281 |
282 | Premises:
283 | (S ∧ B) ∨ (E ∧ ¬B)
284 |
285 | Premise truth table:
286 |
287 | B E S | ((S & B) | (E & ~B))
288 | ----------------------------
289 | T T T | T
290 | T T F | F
291 | T F T | T
292 | T F F | F
293 | F T T | T
294 | F T F | T
295 | F F T | F
296 | F F F | F
297 |
298 | Conclusion:
299 | ¬(S ∧ E)
300 |
301 | Conclusion truth table:
302 |
303 | E S | ~(S & E)
304 | --------------
305 | T T | F
306 | T F | T
307 | F T | T
308 | F F | T
309 |
310 | Argument is invalid because the according to the conclusion when `E =
311 | F` and `S = F`, the premise should be valid which is not the case.
312 |
313 |
314 | Exercise 8
315 | ----------
316 |
317 | Use truth tables to determine which of the following formulas are equiv-
318 | alent to each other:
319 | (a) (P ∧ Q) ∨ (¬P ∧ ¬Q).
320 | (b) ¬P ∨ Q.
321 | (c) (P ∨ ¬Q) ∧ (Q ∨ ¬P).
322 | (d) ¬(P ∨ Q).
323 | (e) (Q ∧ P) ∨ ¬P
324 |
325 | P Q | ((P & Q) | (~P & ~Q))
326 | ---------------------------
327 | T T | T
328 | T F | F
329 | F T | F
330 | F F | T
331 |
332 | P Q | (~P | Q)
333 | --------------
334 | T T | T
335 | T F | F
336 | F T | T
337 | F F | T
338 |
339 | P Q | ((P | ~Q) & (Q | ~P))
340 | ---------------------------
341 | T T | T
342 | T F | F
343 | F T | F
344 | F F | T
345 |
346 | P Q | ~(P | Q)
347 | --------------
348 | T T | F
349 | T F | F
350 | F T | F
351 | F F | T
352 |
353 | P Q | ((Q & P) | ~P)
354 | --------------------
355 | T T | T
356 | T F | F
357 | F T | T
358 | F F | T
359 |
360 | From the truth table `(a)` and `(c)` are equivalent to each other.
361 | Another thing that is equivalent is `(b)` and `(e)`.
362 |
363 | Exercise 9
364 | ----------
365 |
366 | Use truth tables to determine which of these statements are tautologies,
367 | which are contradictions, and which are neither:
368 | (a) (P ∨ Q) ∧ (¬P ∨ ¬Q).
369 | (b) (P ∨ Q) ∧ (¬P ∧ ¬Q).
370 | (c) (P ∨ Q) ∨ (¬P ∨ ¬Q).
371 | (d) [P ∧ (Q ∨ ¬R)] ∨ (¬P ∨ R).
372 |
373 | Truth tables:
374 |
375 | P Q | ((P | Q) & (~P | ~Q))
376 | ---------------------------
377 | T T | F
378 | T F | T
379 | F T | T
380 | F F | F
381 |
382 | P Q | ((P | Q) & (~P & ~Q))
383 | ---------------------------
384 | T T | F
385 | T F | F
386 | F T | F
387 | F F | F
388 |
389 | P Q | ((P | Q) | (~P | ~Q))
390 | ---------------------------
391 | T T | T
392 | T F | T
393 | F T | T
394 | F F | T
395 |
396 | P Q R | ((P & (Q | ~R)) | (~P | R))
397 | -----------------------------------
398 | T T T | T
399 | T T F | T
400 | T F T | T
401 | T F F | T
402 | F T T | T
403 | F T F | T
404 | F F T | T
405 | F F F | T
406 |
407 | From the truth tables, `(b)` is contradiction. `(c)` and `(d)` are
408 | both tautologies. `(a)` is neither of them.
409 |
410 | Exercise 10
411 | -----------
412 |
413 | The first was checked in text, so I will skip it.
414 | Distributive law: `P ∧ (Q ∨ R)` is equivalent to `(P ∧ Q) ∨ (P ∧ R)`
415 |
416 | P Q R | (P & (Q | R))
417 | ---------------------
418 | T T T | T
419 | T T F | T
420 | T F T | T
421 | T F F | F
422 | F T T | F
423 | F T F | F
424 | F F T | F
425 | F F F | F
426 |
427 | P Q R | ((P & Q) | (P & R))
428 | ---------------------------
429 | T T T | T
430 | T T F | T
431 | T F T | T
432 | T F F | F
433 | F T T | F
434 | F T F | F
435 | F F T | F
436 | F F F | F
437 |
438 | Duh, they are equivalent.
439 |
440 | Exercise 11
441 | ------------
442 |
443 | Use the laws stated in the text to find simpler formulas equivalent to these
444 | formulas. (See Examples 1.2.5 and 1.2.7.)
445 | (a) ¬(¬P ∧ ¬Q).
446 | (b) (P ∧ Q) ∨ (P ∧ ¬Q).
447 | (c) ¬(P ∧ ¬Q) ∨ (¬P ∧ Q).
448 |
449 | (a)
450 |
451 | ¬(¬P ∧ ¬Q)
452 | => P ∨ Q (Demorgan's law)
453 |
454 | (b)
455 |
456 | (P ∧ Q) ∨ (P ∧ ¬Q)
457 | => ((P ∧ Q) ∨ P) ∧ ((P ∧ Q) ∨ ¬Q) [Distributive law]
458 | => (P ∨ (P ∧ Q)) ∧ (¬Q ∨ (P ∧ Q)) [Associative law]
459 | => P ∧ ((¬Q ∨ P) ∧ (¬Q ∨ Q)) [Absorption law & Distributive law]
460 | => P ∧ (¬Q ∨ P)
461 | => P [Absorption law]
462 |
463 | (c)
464 |
465 | ¬(P ∧ ¬Q) ∨ (¬P ∧ Q)
466 | => (¬P ∨ Q) ∨ (¬P ∧ Q) [Demorgan's law]
467 | => (¬P ∨ Q ∨ ¬P) ∧ (¬P ∨ Q ∨ Q) [Distributive law]
468 | => (¬P ∨ Q) ∧ (¬P ∨ Q) [Idempotent law]
469 | => (¬P ∨ Q) [Idempotent law]
470 |
471 | Exercise 12
472 | ------------
473 |
474 | Use the laws stated in the text to find simpler formulas equivalent to these
475 | formulas. (See Examples 1.2.5 and 1.2.7.)
476 | (a) ¬(¬P ∨ Q) ∨ (P ∧ ¬R).
477 | (b) ¬(¬P ∧ Q) ∨ (P ∧ ¬R).
478 | (c) (P ∧ R) ∨ [¬R ∧ (P ∨ Q)]
479 |
480 | (a)
481 |
482 | ¬(¬P ∨ Q) ∨ (P ∧ ¬R)
483 | => (¬¬P ∧ ¬Q) ∨ (P ∧ ¬R) [Demorgan's law]
484 | => (P ∧ ¬Q) ∨ (P ∧ ¬R) [Double Negation law]
485 | => P ∧ (¬Q ∨ ¬R) [Distributive law]
486 |
487 | (b)
488 |
489 | ¬(¬P ∧ Q) ∨ (P ∧ ¬R)
490 | => (P ∨ ¬Q) ∨ (P ∧ ¬R) [Demorgan's law]
491 | => (P ∨ ¬Q ∨ P) ∧ (P ∨ ¬Q ∨ ¬R) [Distributive law]
492 | => (P v ¬Q) ∧ (P ∨ ¬Q ∨ ¬R) [Idempotent law]
493 | => (P v ¬Q) [Absorption law]
494 |
495 | (c)
496 |
497 | (P ∧ R) ∨ [¬R ∧ (P ∨ Q)]
498 | => (P ∧ R) ∨ (¬R ∧ P) ∨ (¬R ∧ Q) [Distributive law]
499 | => (P ∧ (R ∨ ¬R)) ∨ (¬R ∧ Q) [Distributive law]
500 | => P ∨ (¬R ∧ Q)
501 |
502 | Exercise 13
503 | ------------
504 |
505 | Use the first DeMorgan’s law and the double negation law to derive the
506 | second DeMorgan’s law.
507 |
508 | First Demorgan's law: `¬(P ∧ Q) is equivalent to ¬P ∨ ¬Q`
509 | Second Demorgan's law: `¬(P ∨ Q) is equivalent to ¬P ∧ ¬Q`
510 |
511 | ¬(P ∨ Q) [LHS of second demorgan's law]
512 | => ¬(¬¬P ∨ ¬¬Q) [Double negation]
513 | => ¬(¬(¬P ∧ ¬Q)) [Demorgan's first law]
514 | => (¬P ∧ ¬Q) [RHS of second demorgan's law]
515 |
516 | Exercise 14
517 | -----------
518 |
519 | Note that the associative laws say only that parentheses are unnecessary
520 | when combining three statements with ∧ or ∨. In fact, these laws can be
521 | used to justify leaving parentheses out when more than three statements
522 | are combined. Use associative laws to show that [P ∧ (Q ∧ R)] ∧ S is
523 | equivalent to (P ∧ Q) ∧ (R ∧ S).
524 |
525 | So, the solution goes like this:
526 |
527 | [P ∧ (Q ∧ R)] ∧ S
528 | => ((P ∧ Q) ∧ R) ∧ S
529 | => (P ∧ Q) ∧ (R ∧ S)
530 |
531 | Exercise 15
532 | -----------
533 |
534 | That's easy: 2^n
535 |
536 | Exercise 16
537 | -----------
538 |
539 | Let's take conjugate of all the rows where it is true and then take
540 | the disjunctions of them. Or more simpler way would be to take
541 | conjugate of where it is false and inverse them in this case:
542 |
543 | ¬(¬P ∧ Q)
544 | => P ∨ ¬Q
545 |
546 | Exercise 17
547 | -----------
548 |
549 | Solve using the same strategy as above:
550 |
551 | (¬P ∧ Q) ∨ (P ∧ ¬Q)
552 | => ((¬P ∧ Q) ∨ P) ∧ ((¬P ∧ Q) ∨ ¬Q)
553 | => (P ∨ Q) ∧ (¬P ∨ ¬Q)
554 |
555 | Exercise 18
556 | -----------
557 |
558 | Suppose the conclusion of an argument is a tautology. What can you
559 | conclude about the validity of the argument? What if the conclusion is
560 | a contradiction? What if one of the premises is either a tautology or a
561 | contradiction?
562 |
563 | If the conclusion of an argument is a tautology, then no matter what
564 | the condition of the premise it is true for all the cases. Ain't that
565 | wonderful! Similarly if the conclusion is a contradiction, then no
566 | matter what the condition of the premises it is false for all the
567 | cases. How prejudiced it is! We cannot say anything about the validity
568 | of the argument in this case.
569 |
570 | What if one of the premises is either a tautology or a contradiction?
571 | Doesn't really matter as the conclusion of an argument is an
572 | tautology.
573 |
--------------------------------------------------------------------------------
/chapter 6/section6-1.tex:
--------------------------------------------------------------------------------
1 | %%%%%%%%%%%%%%%%%%%%%%%%%%%%%%%%%%%%%%%%%
2 | % Author: Sibi
3 | %%%%%%%%%%%%%%%%%%%%%%%%%%%%%%%%%%%%%%%%%
4 | \documentclass{article}
5 | \usepackage{graphicx}
6 | \usepackage{verbatim}
7 | \usepackage{amsmath}
8 | \usepackage{amsfonts}
9 | \usepackage{amssymb}
10 | \usepackage{tabularx}
11 | \usepackage{mathtools}
12 | \newcommand{\BigO}[1]{\ensuremath{\operatorname{O}\bigl(#1\bigr)}}
13 | \setlength\parskip{\baselineskip}
14 | \begin{document}
15 | \title{Chapter 6 (Section 6.1)}
16 | \author{Sibi}
17 | \date{\today}
18 | \maketitle
19 |
20 | % See here: http://tex.stackexchange.com/a/43009/69223
21 | \DeclarePairedDelimiter\abs{\lvert}{\rvert}%
22 | \DeclarePairedDelimiter\norm{\lVert}{\rVert}%
23 |
24 | % Swap the definition of \abs* and \norm*, so that \abs
25 | % and \norm resizes the size of the brackets, and the
26 | % starred version does not.
27 | \makeatletter
28 | \let\oldabs\abs
29 | \def\abs{\@ifstar{\oldabs}{\oldabs*}}
30 | %
31 | \let\oldnorm\norm
32 | \def\norm{\@ifstar{\oldnorm}{\oldnorm*}}
33 | \makeatother
34 | \newpage
35 |
36 | \section{Solution 1}
37 | By mathematical induction, \\
38 | Base case. When $n = 0$, $0 = \frac{n(n+1)}{2} = 0$.
39 | Induction step. Let $n$ be an arbitrary element and suppose $0 + 1 + 2
40 | + ... + n = \frac{n(n+1)}{2}$. Then,
41 | \begin{align*}
42 | 0 + 1 + 2 + ... + n + (n + 1) = \frac{n(n+1)}{2} + (n + 1) \\
43 | = (n+1)(\frac{n}{2} + 1) \\
44 | = \frac{(n+1)(n+2)}{2}
45 | \end{align*}
46 |
47 | \section{Solution 2}
48 | By mathematical induction,
49 |
50 | Base case. When $n = 0$, both sides of the equation becomes $0$.
51 |
52 | Induction step. Let $n$ be arbitrary and suppose $0^2 + 1^2 + 2^2 +
53 | ... + n^2 = \frac{n(n+1)(2n+1)}{6}$. Then,
54 | \begin{align*}
55 | 0^2 + 1^2 + 2^2 + ... + n^2 + (n+1)^2 = \frac{n(n+1)(n+2)}{6} + (n +
56 | 1)^2 \\
57 | = (n+1)(\frac{n(2n + 1)}{6} + (n + 1)) \\
58 | = (n+1)(\frac{2n^2 + n + 6n + 6}{6}) \\
59 | = \frac{(n+1)(n+2)(n+3)}{6} \\
60 | \end{align*}
61 | \section{Solution 3}
62 | By mathematical induction,
63 |
64 | Base case. When $n=0$, both sides of the equation becomes $0$.
65 |
66 | Induction step. Let $n$ be an arbitrary element in $N$. Suppose $0^3 +
67 | 1^3 + 2^3 + ... + n^3 = [\frac{n(n+1)}{2}]^2$. Then,
68 |
69 | \begin{align*}
70 | 0^3 + 1^3 + 2^3 + ... + n^3 + (n+1)^3= [\frac{n(n+1)}{2}]^2 + (n+1)^3 \\
71 | = (n+1)^2(\frac{n^2 + 4n + 4}{4}) \\
72 | = \frac{(n+1)^2(n+2)^2}{4} \\
73 | = [\frac{(n+1)(n+2)}{2}]^2
74 | \end{align*}
75 |
76 | \section{Solution 4}
77 | By mathematical induction,
78 |
79 | Base case. When $n=1$, both sides of the equation becomes 1.
80 |
81 | Induction step. Let $n$ be an arbitrary element and suppose $1+3+5+...
82 | + (2n-1) = n^2$. Then,
83 |
84 | \begin{align*}
85 | 1 + 3 + 5 + ... + (2n - 1) = n^2 \\
86 | 1 + 3 + 5 + ... + (2n - 1) + (n+1)^2 = n^2 + (n+1)^2 \\
87 | 1 + 3 + 5 + ... + (2n - 1) + n^2 + 1 + 2n = n^2 + (n+1)^2 \\
88 | 1 + 3 + 5 + ... + (2n + 1) = (n+1)^2.
89 | \end{align*}
90 |
91 | \section{Solution 5}
92 | By mathematical induction,
93 |
94 | Base case. When $n=0$, both sides of the equation becomes $0$.
95 |
96 | Inductive step. Let $n$ be an arbitrary element and suppose $0.1 + 1.2
97 | + 2.3 + ... + n(n+1) = \frac{n(n+1)(n+2)}{3}$. Then,
98 |
99 | \begin{align*}
100 | 0.1 + 1.2 + 2.3 + ... + n(n+1) + (n+1)(n+2) = \frac{n(n+1)(n+2)}{3}
101 | + (n+1)(n+2) \\
102 | = (n+1)(n+2)(\frac{n}{3} + 1) \\
103 | = \frac{(n+1)(n+2)(n+3)}{3}
104 | \end{align*}
105 | \section{Solution 6}
106 | Guess: $\frac{n(n+1)(n+2)(n+3)}{4}$
107 |
108 | By mathematical induction,
109 |
110 | Base case. When $n=0$, both sides of the equation becomes $0$.
111 |
112 | Induction step. Let $n$ be an arbitrary element. Suppose $0.1.2 +
113 | 1.2.3 + 2.3.4 + ... + n(n+1)(n+2) = \frac{n(n+1)(n+2)(n+3)}{4}$. Then,
114 |
115 | \begin{align*}
116 | 0.1.2 + 1.2.3 + 2.3.4 + ... + n(n+1)(n+2) + (n+1)(n+2)(n+3) =
117 | \frac{n(n+1)(n+2)(n+3)}{4} + (n+1)(n+2)(n+3) \\
118 | = (n+1)(n+2)(n+3)(\frac{n}{4} + 1) \\
119 | = \frac{(n+1)(n+2)(n+3)(n+4)}{4}
120 | \end{align*}
121 |
122 | \section{Solution 7}
123 | Guess: $\frac{3^{n+1} - 1}{2}$
124 |
125 | By mathematical induction,
126 |
127 | Base case. When $n=0$, both sides of the equation becomes 1.
128 |
129 | Induction case. Let $n$ be an arbitrary element. Suppose $3^0 + 3^1 +
130 | 3^2 + ... + 3^n = \frac{3^{n+1} - 1}{2}$. Then,
131 |
132 | \begin{align*}
133 | 3^0 + 3^1 + 3^2 + ... + 3^n + 3^{n+1} = \frac{3^{n+1} - 1}{2} +
134 | 3^{n+1} \\
135 | = \frac{3^{n+1} - 1 + 2.3^{n+1}}{2} \\
136 | = \frac{3^{n+2} - 1}{2}
137 | \end{align*}
138 |
139 | \section{Solution 8}
140 | By mathematical induction,
141 |
142 | Base case. When $n = 1$, both sides of the equation becomes $1/2$.
143 |
144 | Induction step. Let $n$ be an arbitrary element such that $n >= 1$ and
145 | suppose $1 - \frac{1}{2} + \frac{1}{3} + ... + \frac{1}{2n - 1} -
146 | \frac{1}{2n} = \frac{1}{n + 1} + \frac{1}{n+2} + \frac{1}{n+3} + ...
147 | + \frac{1}{2n} $. Then,
148 | \begin{align*}
149 | 1 - \frac{1}{2} + \frac{1}{3} + ... + \frac{1}{2n - 1} -
150 | \frac{1}{2n} + \frac{1}{2n+1} - \frac{1}{2n+2} \\
151 | = \frac{1}{n+1} + \frac{1}{n+2} + \frac{1}{n+3} + ... +
152 | \frac{1}{2n} + \frac{1}{2n+1} - \frac{1}{2n+2} \\
153 | = \frac{1}{n+2} + \frac{1}{n+3} + ... + \frac{1}{2n} +
154 | \frac{1}{2n+1} + \frac{1}{2n+2}
155 | \end{align*}
156 |
157 | as required.
158 |
159 | \section{Solution 9}
160 | \subsection{Solution (a)}
161 | By mathematical induction,
162 |
163 | Base case. When $n = 0$, $\exists k \in Z$ such that $2k = n^2 + 2$.
164 | Here if $k = 0$, then base case holds.
165 |
166 | Induction step. Let $n$ be an arbitrary element in $N$. Suppose $2k =
167 | n^2 + n$ for some $k \in Z$. Thus,
168 | \begin{align*}
169 | (n + 1)^2 (n + 1) = n^2 + 1 + 2n + n + 1 \\
170 | = n^2 + 3n + 2 \\
171 | = n^2 + n + 2n + 2 \\
172 | = 2k + 2n + 2 \\
173 | = 2(k + n + 2)
174 | \end{align*}
175 |
176 | Therefore $2 \mid (n+1)^2 + (n+1)$ as required.
177 |
178 | \subsection{Solution (b)}
179 | By mathematical induction,
180 |
181 | Base case. When $ n = 0$, it holds that $6 \mid (n^3 - n)$ for $0 \in
182 | Z$.
183 |
184 | Induction step. Let $n$ be an arbitrary element in $N$ such that $6
185 | \mid (n^3 - n)$.
186 | \begin{align*}
187 | (n+1)^3 - (n+1) \\
188 | = (n+1)((n+1)^2 - 1) \\
189 | = (n+1)(n^2 + 1 + 2n - 1) \\
190 | = (n+1)(n^2 + 2n) \\
191 | = n^3 + 3n^2 + 2n \\
192 | = n^3 - n + 3n^2 + 2n \\
193 | = 6k + 3n^2 + 3n \\
194 | = 6(k + n^2/2 + n/2) \\
195 | = 6(k + n(n+1)/2)
196 | \end{align*}
197 | Now $n(n+1)/2$ is the sum of number series upto $n$. So, $6 \mid
198 | (n+1)^3 - (n+1)$ holds.
199 |
200 | \section{Solution 10}
201 | By mathematical induction,
202 |
203 | Base case. When $n = 0$, $64 \mid (9^n - 8n - 1)$ holds for $0$.
204 |
205 | Induction step. Let $n$ be an arbitrary element in $N$ such that $64k
206 | = (9^n - 8n - 1)$. Then,
207 | \begin{align*}
208 | 9^{n+1} - 8(n+1) - 1 = 9^{n+1} - 8n - 9 \\
209 | = 9^{n+1} - 9n - 9 + n \\
210 | = 9(9^n - n - 1) + n \\
211 | = 9(9^n - 8n - 1 + 7n) + n \\
212 | = 9(64k + 7n) + n \\
213 | = 64(9k + n) \\
214 | \end{align*}
215 | So, $64 \mid (9^{n+1} - 8(n+1) - 1)$ as required.
216 |
217 | \section{Solution 11}
218 | By mathematical induction,
219 |
220 | Base case. When $n = 0$, $9 \mid (4^n + 6n - 1)$ holds for $0$.
221 |
222 | Induction step. Let $n$ be an arbitrary element in $N$ and suppose $9k
223 | = 4^n + 6n - 1$. Thus,
224 | \begin{align*}
225 | 4^{n+1} + 6(n+1) - 1 = 4^{n+1} + 6n + 5 \\
226 | = 4^{n+1} + 4n + 2n + 5 \\
227 | = 4(4^{n} + n) + 2n + 5 \\
228 | = 4(4^{n} + 6n - 1 - 5n + 1) + 2n + 5 \\
229 | = 4(9k - 5n + 1) + 2n + 5 \\
230 | = 4.9k - 18n + 9 \\
231 | = 9(4k - 2n + 1)
232 | \end{align*}
233 | So, $9 \mid 4^{n+1} + 6(n+1) - 1$ as required.
234 |
235 | \section{Solution 12}
236 | Let $a$ and $b$ be arbitrary integer. By mathematical induction,
237 |
238 | Base case. When $n = 0$, $(a-b) \mid (a^n - b^n)$ holds for $0$.
239 | Induction step. Let $n$ be arbitrary element in $N$ and suppose
240 | $(a-b)k = a^n - b^n$ . Thus,
241 | \begin{align*}
242 | a^{n+1} - b^{n+1} = a(a^n - b^n) + b^n(a-b) \\
243 | = a(a-b)k + b^n(a-b) \\
244 | = (a-b)(ak + b^n) \\
245 | \end{align*}
246 | So, $(a-b) \mid (a^{n+1} - b^{n+1})$ as required.
247 |
248 | \section{Solution 13}
249 | Let $a$ and $b$ be arbitrary element. By mathematical induction,
250 |
251 | Base case. When $n=0$, $(a+b) \mid (a^{2n+1} + b^{2n + 1})$ holds for
252 | $1 \in Z$.
253 |
254 | Induction step. Let $n$ be an arbitrary element in $N$ such that
255 | $(a+b)k = a^{2n+1} + b^{2n+1}$. Thus,
256 | \begin{align*}
257 | a^{2(n+1) + 1} + b^{2(n+1) + 1} = a^{2n + 3} + b^{2n + 3} \\
258 | = a^2(a^{2n + 1}+ b^{2n+1}) + b^{2n+1}(b^2 - a^2) \\
259 | = a^2(a+b)k + b^{2n+1}(b-a)(b+a) \\
260 | = (a+b)(a^2k + b^{2n+1}(b-a))
261 | \end{align*}
262 | So, $(a+b) \mid a^{2(n+1) + 1} + b^{2(n+1) + 1}$ as required.
263 |
264 | \section{Solution 14}
265 | By mathematical induction,
266 |
267 | Base case. When $n = 10$, $2^n > n^3$ holds since $2^{10} > 2^3$.
268 |
269 | Induction step. Let $n$ be an arbitrary element and suppose $n >= 10$
270 | and $2^n > n^3$. Then,
271 | \begin{align*}
272 | 2^{n+1} = 2.2^n \\
273 | > 2.n^3 (\text{Inductive hypothesis}) \\
274 | = n^3 + n^3 \\
275 | >= n^3 + 10n^2 (\text{Since n >= 10}) \\
276 | = n^3 + 3n^2 + 7n^2 \\
277 | >= n^3 + 3n^2 + 70n (\text{Since n >= 10}) \\
278 | = n^3 + 3n^2 + 3n + 67n \\
279 | > n^3 + 3n^2 + 3n + 1 \\
280 | > (n+1)^3
281 | \end{align*}
282 | So, $2^{n+1} > (n+1)^3$ as required.
283 |
284 | \section{Solution 15}
285 | By mathematical induction,
286 |
287 | Base case. When $n=0$, $n \equiv 0 (mod 3)$ holds for $0 \in Z$.
288 |
289 | Induction step. Let $n$ be an arbitrary element in $N$. Suppose
290 | $n \equiv 0 (\text{mod } 3)$ or $n \equiv 1 (\text{mod } 3)$ or
291 | $n \equiv 2 (\text{mod } 3)$.
292 |
293 | Case 1. $n \equiv 0 (\text{mod } 3)$. It follows that $n = 3a$ for
294 | some $a \in Z$. Adding $1$ on both sides, we get $n+1 = 3a + 1$. So,
295 | $(n+1) - 1 = 3a$. Therefore, $n+1 = 1(\text{mod } 3)$. So, either
296 | $n+1 = 0(\text{mod } 3)$ or $n+1 = 1(\text{mod } 3)$ or
297 | $n+1 = 2(\text{mod } 3)$.
298 |
299 | Case 2. $n \equiv 1 (\text{mod } 3)$. It follows that $n-1=3b$ for
300 | some $b \in Z$. We can rewrite it as $(n+1) - 2 = 3b$. So,
301 | $(n+1) = 2(\text{mod } 3)$. So, $(n+1) \equiv 2(\text{mod } 3)$. So,
302 | either $n+1 = 0(\text{mod } 3)$ or $n+1 = 1(\text{mod } 3)$ or
303 | $n+1 = 2(\text{mod } 3)$.
304 |
305 | Case 3. $n \equiv 2 (\text{mod } 3)$. It follows that $n-2 = 3c$ for
306 | some $c \in Z$. Rewriting it, we get $n+1 = 3(c+1)$. So,
307 | $n+1 \equiv 0 (\text{mod } 3)$. So, either $n+1 = 0(\text{mod } 3)$ or
308 | $n+1 = 1(\text{mod } 3)$ or $n+1 = 2(\text{mod } 3)$.
309 |
310 | \section{Solution 16}
311 | By mathematical induction,
312 |
313 | Base case. When $n = 1$, both sides of the equation becomes $8$.
314 |
315 | Induction step. Let $n >=1$ be an arbitrary element. Suppose $2.2^1 +
316 | 3.2^2 + 4.2^3 + ... + (n+1)2^n = n2^{n+1}$. Then,
317 | \begin{align*}
318 | 2.2^1 + 3.2^2 + 4.2^3 + ... + (n+2)2^{n+1} \\
319 | = (2.2^1 + 3.2^2 + ... + (n+1)2^n) + (n+2)2^{n+1} \\
320 | = n2^{n + 1} + (n+2)2^{n+1} \\
321 | = 2^{n+1}(n + n + 2) \\
322 | = 2^{n+2}(n+1)
323 | \end{align*}
324 |
325 | \section{Solution 17}
326 | \subsection{Solution (a)}
327 | Base case doesn't hold. (When $n = 0$).
328 |
329 | Let $f(n) = 1.3^0 + 3.3^1 + 5.3^2 + ... + (2n+1)3^n$
330 | When $n = 0$, $f(n) = 1$,
331 | $n \neq 0, f(n) = n3^{n + 1}$.
332 |
333 | Proof. Copy the question of 17(a) :-)
334 |
335 | \section{Solution 18}
336 | Suppose $a$ is a real number and $a = 0$. Let $n$ be an arbitrary
337 | element in $N$. Let us consider the cases.
338 |
339 | Case 1. n is even. By mathematical induction,
340 |
341 | Base case. When $n = 0$, then $a^n > 0$ since $a^n = 1$.
342 |
343 | Induction step. Let $n$ be an even arbitrary number in $N$. Suppose
344 | $a^n > 0$. We have to prove $a^{n+2} > 0$. We know that $a^n > 0$.
345 | Since $a$ is a negative real number, $a^2$ will be a positive number.
346 | So, $a^n.a^2 > 0$. So, $a^{n+2} > 0$ as required.
347 |
348 | Case 2. n is odd. By mathematical induction,
349 |
350 | Base case. When $n=1$, $a^{n} < 0$ since $a$ is a negative number.
351 |
352 | Induction step. Let $n$ be an odd arbitrary number in $N$. Suppose
353 | $a^n < 0$. We have to prove that $a^{n+2} < 0$. Since $a < 0$, $a^2 >
354 | 0$ since $a$ is a negative number. Multiplying $a^2$ with $a^n < 0$,
355 | we get $a^{n+2} < 0$ as required.
356 |
357 | \section{Solution 19}
358 | Suppose $a$ and $b$ are real numbers and $0 < a < b$.
359 | \subsection{Solution (a)}
360 | By mathematical induction,
361 |
362 | Base case. When $n=1$, $0 < a^n < b^n$.
363 |
364 | Induction step. Let $n$ be an arbitrary element such that $n>= 1$.
365 | Suppose $0 < a^n < b^n$. Then,
366 | $0 < a^n < b^n$ \\
367 | \begin{equation}\label{eq:1}
368 | 0 < a^{n+1} < ab^n
369 | \end{equation}
370 | We know that $a < b$. So, $ab^n < b^{n+1}$. From \ref{eq:1} , it follows
371 | that $0 < a^{n+1} < b^{n+1}$ as required.
372 |
373 | \subsection{Solution (b)}
374 | By mathematical induction,
375 |
376 | Base case. When $n = 2$, $0 < \sqrt[n]{a} < \sqrt[n]{b}$ since $0 < a
377 | < b$.
378 |
379 | Induction step. Let $n$ be an arbitrary element such that $n >= 2$.
380 | Suppose $0 < \sqrt[n]{a} < \sqrt[n]{b}$. Then,
381 | \begin{align*}
382 | 0 < \sqrt[n]{a} < \sqrt[n]{b} \\
383 | 0 < \sqrt{a} x \sqrt[n]{a} < \sqrt{a} x \sqrt[n]{b} \\
384 | \end{align*}
385 | \begin{equation} \label{eq:2}
386 | 0 < (n+1)\sqrt[n+1]{a} < \sqrt{a}.\sqrt[n]{b}
387 | \end{equation}
388 |
389 | We know that $a < b$. So, $\sqrt{a} < \sqrt{b}$. Multiplying it by
390 | $\sqrt[n]{b}$ on both sides, $\sqrt{n}. \sqrt[n]{b} < \sqrt[n+1]{b}$.
391 | From \ref{eq:2}, $0 < \sqrt[n+1]{a} < \sqrt[n+1]{b}$ as required.
392 |
393 | \subsection{Solution (c)}
394 | By mathematical induction,
395 |
396 | Base case. Since $a > 0$ and $b>0$, it follows that $(a-b)^2 > 0$. So,
397 | $a^2 + b^2 - 2ab > 0$. So, $a^2 + b^2 > 2ab$.
398 |
399 | Induction step. Let $n$ be an arbitrary element such that $n >= 1$.
400 | Suppose $ab^n + ba^n < a^{n+1} + b^{n+1}$. Then,
401 | \begin{align*}
402 | 0 < a^{n+1} + b^{n+1} - ba^n - ab^n \\
403 | 0 < a(a^n - b^n) + b(b^n - a^n) \\
404 | (a^n - b^n)(a - b) > 0 \\
405 | \end{align*}
406 | From (a), it follows that $a^{n+1} < b^{n+1}$ . We know that $a - b >
407 | 0$. Multiplying both,
408 | \begin{align*}
409 | (a^{n+1} - b^{n+1})(a-b) > 0 \\
410 | ab^{n+1} + ba^{n+1} < a^{n+2} + b^{n+2} \text{ as required}
411 | \end{align*}
412 |
413 | \subsection{Solution (d)}
414 | By mathematical induction,
415 |
416 | Base case. When $n = 2$, $(\frac{a+b}{2})^2 < \frac{a^2 + b^2}{2}$ since
417 | $2ab < a^2 + b^2$. (Apply n = 1, to question 19 (c))
418 |
419 | Induction step. Let $n >= 2$ be an arbitrary element and suppose
420 | $(\frac{a+b}{2})^2 < \frac{a^2 + b^2}{2}$. Then,
421 | \begin{align*}
422 | (\frac{a+b}{2})^{n+1} < \frac{a^n + b^n}{2}.\frac{a+b}{2} \\
423 | (\frac{a+b}{2})^{n+1} < \frac{a^{n+1} + b^{n+1} + ba^n + ab^n}{4} \\
424 | \end{align*}
425 | From part (c), $ab^n + ba^n < a^{n+1} + b^{n+1}$
426 | So, $\frac{a^{n+1} + b^{n+1} + ba^n + ab^n}{4} < \frac{2a^{n+1} +
427 | 2b^{n+1}}{4}$. Hence. $(\frac{a+b}{2})^{n+1} < \frac{a^{n+1} + b^{n+1}}{2}$.
428 |
429 | \end{document}
430 |
--------------------------------------------------------------------------------
/chapter 6/section6-2.tex:
--------------------------------------------------------------------------------
1 | %%%%%%%%%%%%%%%%%%%%%%%%%%%%%%%%%%%%%%%%%
2 | % Author: Sibi
3 | %%%%%%%%%%%%%%%%%%%%%%%%%%%%%%%%%%%%%%%%%
4 | \documentclass{article}
5 | \usepackage{graphicx}
6 | \usepackage{verbatim}
7 | \usepackage{amsmath}
8 | \usepackage{amsfonts}
9 | \usepackage{amssymb}
10 | \usepackage{tabularx}
11 | \usepackage{mathtools}
12 | \newcommand{\BigO}[1]{\ensuremath{\operatorname{O}\bigl(#1\bigr)}}
13 | \setlength\parskip{\baselineskip}
14 | \begin{document}
15 | \title{Chapter 6 (Section 6.2)}
16 | \author{Sibi}
17 | \date{\today}
18 | \maketitle
19 |
20 | % See here: http://tex.stackexchange.com/a/43009/69223
21 | \DeclarePairedDelimiter\abs{\lvert}{\rvert}%
22 | \DeclarePairedDelimiter\norm{\lVert}{\rVert}%
23 |
24 | % Swap the definition of \abs* and \norm*, so that \abs
25 | % and \norm resizes the size of the brackets, and the
26 | % starred version does not.
27 | \makeatletter
28 | \let\oldabs\abs
29 | \def\abs{\@ifstar{\oldabs}{\oldabs*}}
30 | %
31 | \let\oldnorm\norm
32 | \def\norm{\@ifstar{\oldnorm}{\oldnorm*}}
33 | \makeatother
34 | \newpage
35 |
36 | \section{Solution 1}
37 |
38 |
39 | $R' = R \cap (A' \times A')$
40 | $A' = A \setminus \{a\}$
41 |
42 | \subsection{Solution (a)}
43 | Reflexive.
44 |
45 | Let $b$ be an arbitrary element on $A'$. Since $A' \subseteq A$, it
46 | follows that $(b,b) \in R$ since $R$ is reflexive. Also,
47 | $(b,b) \in A' \times A'$. So, $(b,b) \in R \cap (A' \times A')$. Since
48 | $b$ is arbitrary we can conclude that $R$ is reflexive.
49 |
50 | Transitive.
51 |
52 | Let $b,c,d$ be arbitrary element on $A'$ such that $(b,c) \in R'$ and
53 | $(c,d) \in R'$. Since $R$ is transitive, it follows that
54 | $(b,d) \in R$. Also, $(b,d) \in A' \times A'$. So,
55 | $(b,d) \in R \cap (A' \times A')$.
56 |
57 | Anti-symmetric.
58 |
59 | Let $x,y$ be arbitrary element on $A'$ such that $xR'y$ and $yR'x$.
60 | Since $R$ is anti-symmetric, it follows that $x=y$.
61 |
62 | \subsection{Solution (b)}
63 | Reflexive.
64 |
65 | Let $x$ be an arbitrary element in $A$. Then either $x \in A'$ or
66 | $x \in \{a\}$. Let us consider the cases:
67 |
68 | Case 1. $x \in A'$ Since $T'$ is a total order in $A'$, it follows
69 | that $(x,x) \in T$.
70 | Case 2. $x \in \{a\}$. Then $x = a$. It follows that
71 | $(a,a) \in \{a\} \times A$. So, $(x,x) \in T$.
72 |
73 | Transitive.
74 |
75 | Let $x,y$ and $z$ be arbitrary element in $A$ such that $(x,y) \in T$
76 | and $(y,z) \in T$. Let us consider the cases.
77 |
78 | Case 1. $x \neq a$. Then $x \in A'$. Since
79 | $T = T' \cup (\{a\} \times A)$, it follows that $y \in A'$ and
80 | $z \in A'$. Since $T'$ is transitive, it follows that $(x,z) \in T$.
81 | Case 2. $x = a$ Since $z \in A$, it follows that
82 | $(a,z) \in \{a\} \times A$. So, $(x,z) \in T$.
83 |
84 | Anti-symmetric
85 |
86 | Let $x,y$ be arbitrary element on $A$ such that $xTy$ and $yTx$. Let
87 | us consider the cases:
88 | Case 1. $x \neq a$. Then $x \in A'$, $y \in A'$ since $(x,y) \in T'
89 | \cup ({a} \times A)$ and $x \in A'$. So, $(x,y) \in T'$. Similarly,
90 | $(y,x) \in T'$. Since $T'$ is a total order on $A'$, it follows that
91 | $x = y$.
92 | Case 2. $x = a$. So, $(y,a) \in T$. Therefore $(y,a) \in T' \cup \{a\}
93 | \times A$. Now $T'$ is a relation on $A'$ which doesn't have $a$. So,
94 | $(y,a) \notin T'$. So, $(y,a) \in \{a\} \times A$. Now, clearly $y =
95 | a$. So, $x = y$.
96 |
97 | $\forall x \in A \forall y \in A (xTy \lor yTx)$
98 |
99 | Let $x,y$ be arbitrary element on $A$. Let us consider the cases:
100 | Case 1. $x = a$. Since $y \in A$, it follows that $(a,y) \in \{a\}
101 | \times A$. So, $xTy \lor yTx$.
102 | Case 2. $x \neq a$. Now if $y = a$, then $xRa$ will contradict the $R$
103 | minimality of $A$. If $y \neq a$, then $y \in A'$. So, $(x,y) \in R'$.
104 | Since $R' \subseteq T' \subseteq T$, it follows that $(x,y) \in T$.
105 |
106 | \section{Solution 3}
107 | We will show by induction that for every natural number $n >= 1$,
108 | every subset of $A$ with $n$ elements has a smallest element.
109 |
110 | Base case. When $n =1$, then the single element present in it is the
111 | smallest element.
112 |
113 | Induction step. Suppose $n >= 1$ and suppose that every subset of $A$
114 | has a smallest element. Now let $B$ be arbitrary subset of $A$ with
115 | $n+1$ elements. Let $b$ be an arbitrary element of $B$ and let
116 | $B' = B \setminus \{b\}$ with n elements. By inductive hypothesis,
117 | $B'$ has a smallest element $c \in B'$. Let us consider the cases.
118 |
119 | Case 1. $bRc$ We know that $\forall x \in B'(cRx)$ and $B' = B
120 | \setminus \{b\}$.
121 | Now from $bRc$, it is clear that $b$ is the smallest element. (Note
122 | that $R$ is transitive).
123 |
124 | Case 2. $cRb$ Then it follows that $\forall x \in B(cRx)$. So, $c$ is
125 | the smallest element.
126 |
127 | \section{Solution 4}
128 | \subsection{Solution (a)}
129 | We will show by mathematical induction that $\forall n >= 1 \forall B
130 | \subseteq A (\forall x \in A \forall y \in A(xRy \lor yRx) \implies
131 | \exists z \in B \forall y \in B((z,y) \in R \circ R))$.
132 |
133 | Base case. When $n=1$, then $B = \{b\}$ for some $b \in A$. Since
134 | $(b,b) \in R \circ R$, base case holds.
135 |
136 | Induction step. Suppose $n >= 1$ and suppose that every subset of $A$
137 | has some elements $z \in B$ such that $\forall y \in B((z,y) \in R
138 | \circ R)$. Now let $C$ be arbitrary subset of $A$ with $n+1$ elements.
139 | Now let $c$ be some element in $C$ and let $C' = C \setminus \{c\}$.
140 | By inductive hypothesis, $\exists z \in C' \forall y \in C'((z,y) \in
141 | R \circ R)$.
142 | Case 1. $(z,c) \in R \circ R$. Then $\forall y \in C((z,y) \in R \circ
143 | R)$.
144 | Case 2. $(z,c) \notin R \circ R$. Then we will try to prove that
145 | $\forall y \in C((c,y) \in R \circ R)$. Let $y$ be arbitrary element
146 | in $C$. If $y = c$, then since $R$ is reflexive, $(c,y) \in R \circ
147 | R$. Now, if $y \neq c$, then $y \in C'$. So, $(z,y) \in R \circ R$.
148 | This means $\exists x \in A such that (z,x) \in R$ and $(x,y) \in R$.
149 | Now if $(z,x) \in R$and $(x,c) \in R$, then $(z,c) \in R \circ R$
150 | contrary to assumption. So, $(x,c) \notin R$. From the hypothesis on
151 | $R$, $(c,x) \in R$. From $(c,x) \in R$ and $(x,y) \in R$, it follows
152 | that $(c,y) \in R \circ R$.
153 |
154 | \subsection{Solution (b)}
155 | \begin{align*}
156 | A = \text{Set of all contestants} \\
157 | R = \{(x,y) \in A \times A \mid x beats y \}
158 | \end{align*}
159 | Now $R$ satisfies this: $\forall x \in A \forall y \in A(xRy \lor
160 | yRx)$. Now from $(a)$, it follows that $B \subseteq A$, $\exists x \in
161 | B(\forall y \in B((x,y) \in R \circ R))$. Let $B=A$, then $(x,y) \in R
162 | \circ R$. So there is at least one successful player.
163 |
164 | \section{Solution 5}
165 | By mathematical induction,
166 |
167 | Base case. When $n=1$, $F_1 = F_o.F_1.F_2$. Both the sides of the
168 | equation becomes $5$ and so the base case holds.
169 |
170 | Induction step. Let $n$ be arbitrary element and suppose $F_n =
171 | 2^{(2^n)} + 1$. We have to prove that $F_{n+1} = F_0.F_1.F_2 ... F_n +
172 | 2$. By inductive hypothesis,
173 | \begin{align*}
174 | F_n = F_0.F_1.F_2 ... F_{n-1} + 2 \\
175 | F_0.F_1.F_2 ... F_{n-1}.F_n = (F_n - 2)F_n \\
176 | F_0.F_1.F_2 ... F_{n-1}.F_n + 2 = (F_n - 2)F_n + 2 \\
177 | = (F_n)^2 - 2F_n + 2 \\
178 | = (2^{(2^n)} + 1)^2 - 2(2^{2^n} + 1) + 2 \\
179 | = 2^{(2^n)} + 1 \\
180 | = F_n + 1
181 | \end{align*}
182 |
183 | \section{Solution 6}
184 | By mathematical induction,
185 |
186 | Base case. When $n=1$, $|a_1| = |a_1|$ so base case holds.
187 |
188 | Induction step. Let $n$ be arbitrary element and suppose $a_1 + a_2 +
189 | ... + a_n <= |a_1| + |a_2| + ... + |a_n|$. Let $a_{n+1}$ be arbitrary
190 | real number. Let us consider the cases.
191 |
192 | Case 1. $a_{n+1}$ is positive: From our inductive hypothesis, it
193 | follows that $a_1 + a_2 + ... + a_n + a_{n+1} <= |a_1| + |a_2| + ... +
194 | |a_n| + |a_{n+1}|$.
195 |
196 | Case 2. $a_{n+1}$ is negative: Then it follows that
197 | $a_1 + a_2 + ... + a_n + a_{n+1} < |a_1| + |a_2| + ... + |a_n| +
198 | |a_{n+1}|$. So,
199 | $a_1 + a_2 + ... + a_n + a_{n+1} <= |a_1| + |a_2| + ... + |a_n| +
200 | |a_{n+1}|$.
201 |
202 | \section{Solution 7}
203 | \subsection{Solution (a)}
204 | Let $a$ and $b$ be arbitrary positive real numbers. We know that
205 | $(a-b)^2 >= 0$. Then,
206 | \begin{align*}
207 | (a-b)^2 >= 0 \\
208 | a^2 + b^2 >= 2ab \\
209 | \frac{a}{b} + \frac{b}{a} >= 2
210 | \end{align*}
211 |
212 | QED.
213 |
214 | \subsection{Solution (b)}
215 | Suppose $a,b$ and $c$ be arbitrary real number and suppose $0 < a <= b
216 | <= c$. Then, $(c-b)(c-a) >= 0$ since $c >= b$ and $c >= a$. Then,
217 | \begin{align*}
218 | c(c-b)-a(c-b) >= 0 \\
219 | c(c-b) + ab >= ac \\
220 | \frac{ab + c(c-b)}{ac} >= 1 \\
221 | \frac{b}{c} + \frac{c-b}{a} >= 1 \\
222 | \frac{b}{c} + \frac{c}{a} - \frac{b}{a} >= 1
223 | \end{align*}
224 | QED.
225 |
226 | \subsection{Solution (c)}
227 | By mathematical induction,
228 |
229 | Base case. When $n = 2$, from 7(a), it follows that $\frac{a_1}{a_2} +
230 | \frac{a_2}{a_1} >= 2$. So base case holds.
231 |
232 | Induction step. Let $n >= 2$ be arbitrary element and suppose
233 | $\frac{a_1}{a_2} + \frac{a_2}{a_3} + \frac{a_3}{a_4} + ... +
234 | \frac{a_{n-1}}{a_n} + \frac{a_n}{a_1} >= n$. We have to prove
235 | $\frac{a_1}{a_2} + \frac{a_2}{a_3} + \frac{a_3}{a_4} + ... +
236 | \frac{a_{n-1}}{a_n} + \frac{a_n}{a_{n+1}} + \frac{a_{n+1}}{a_1} >= n +
237 | 1$. From $7(b)$, it follows that $\frac{a_n}{a_{n+1}} +
238 | \frac{a_{n+1}}{a_1} - \frac{a_n}{a_1} >= 1$.
239 | From inductive hypothesis, it follows that:
240 | $\frac{a_1}{a_2} + \frac{a_2}{a_3} + \frac{a_3}{a_4} + ... +
241 | \frac{a_{n-1}}{a_n} + \frac{a_n}{a_{n+1}} +
242 | \frac{a_{n+1}}{a_1} >= n + 1$.
243 |
244 | QED.
245 |
246 | \section{Solution 8}
247 | \subsection{Solution (a)}
248 | Let $a$ and $b$ be arbitrary positive real number. We know that
249 | $(a-b)^2 >= 0$. So,
250 | \begin{align*}
251 | (a-b)^2 >= 0 \\
252 | a^2 + b^2 >= 2ab \\
253 | a^2 + b^2 + 2ab >= 4ab \\
254 | (a+b)^2 >= 4ab \\
255 | \frac{(a+b)}{2} >= \sqrt{ab}
256 | \end{align*}
257 | QED.
258 |
259 | \subsection{Solution (b)}
260 | By mathematical induction on $n$,
261 |
262 | Base case. When $n=1$, From 8(a), base case holds.
263 |
264 | Induction step. Suppose $n>=1$ and the arithmetic geometric mean
265 | inequality holds for lists of length $2^n$. Now let $a_1,a_2,...,
266 | a_{a^{n+1}}$ be a list of $2^{n+1}$ positive real numbers. Let
267 | $m_1 = \frac{a_1 + a_2 + ... + a_{2^n}}{2^n} and m_2 = \frac{a_{2^n +
268 | 1} + a_{2^n + 2} + ... + a_{2^{n+1}}}{2^n}$.
269 |
270 | So, $a_1 + a_2 + ... + a_{2^n} = m_12^n$ and similarly $a_{2^n + 1} +
271 | a_{2^n + 2} + ... + a_{2^{n+1}} = m_22^n$. Also, by inductive
272 | hypothesis, we know that $m_1 >= \sqrt[2^n]{a_1a_2...a_{2^n}}$ and
273 | $m_2 >= \sqrt[2^n]{a_{2^n + 1}a_{2^n + 2}...a_{2^{n+1}} }$. Therefore,
274 | \begin{align*}
275 | \frac{a_1 + a_2 + ... + a_{2^n + 1}}{2^{n+1}} \\
276 | = \frac{m_12^n + m_22^n}{2^{n+1}} \\
277 | = \frac{m_1 + m_2}{2} \\
278 | >= \sqrt{m_1m_2} (\text{from Solution (a)}) \\
279 | >= \sqrt{\sqrt[2^n]{a_1a_2...a_{2^n}}\sqrt[2^n]{a_{2^n + 1}a_{2^n+2}...a_{2^{n+1}}}} \\
280 | >= \sqrt[2^{n+1}]{a_1a_2...a_{a^{n+1}}}
281 | \end{align*}
282 |
283 | \section{Solution (c)}
284 | By mathematical induction on $n$,
285 |
286 | Base case. When $n=n_0$, then base case holds because of inductive
287 | hypothesis.
288 |
289 | Induction step. Suppose $n >= n_0$ and there are positive real numbers
290 | $a_1, a_2, ..., a_n$ such that
291 | \begin{align*}
292 | \frac{a_1 + a_2 + ... + a_n}{n} < \sqrt[n]{a_1a_2...a_n}
293 | \end{align*}
294 | Let $m = \frac{a_1 + a_2 + ... + a_n}{n}$ and $a_{n+1} = m$. Then we
295 | have $m < \sqrt[n]{a_1a_2...a_n}$, so $m^n < a_1a_2...a_n$. So,
296 | $m^{n+1} < a_1a_2...a_na_{n+1}$. So, $m <
297 | \sqrt[n+1]{a_1a_2...a_{n+1}}$. Now,
298 | \begin{align*}
299 | \frac{a_1 + a_2 + ... + a_{n+1}}{n+1} \\
300 | = \frac{mn + m}{n+1} \\
301 | = m
302 | \end{align*}
303 | So, $\frac{a_1 + a_2 + ... + a_{n+1}}{n+1} <
304 | \sqrt[n+1]{a_1a_2...a_{n+1}}$
305 |
306 | \subsection{Solution (d)}
307 | $(b)$ and $(c)$ give contradiction for a list of length $n_0 < 2^n$
308 | where $n >= 1$. So the inequality must hold.
309 |
310 | \section{Solution 9}
311 | Let $\frac{1}{a_n} = k_n$. From arithmetic geometric mean inequality,
312 |
313 | \begin{align*}
314 | \frac{k_1 + k_2 + ... + k_n }{n} <= \sqrt[n]{k_1K_2...K_n} \\
315 | \frac{n}{k_1 + k_2 + ... + k_n } <= (k_1k_2...k_n)^{-1/n} \\
316 | <= (a_1a_2...a_n)^{1/n} \\
317 | \end{align*}
318 | QED.
319 |
320 | \section{Solution 10}
321 | By mathematical induction on $n$,
322 |
323 | Base case. When $n=0$, $P(A)$ has 1 element which is the empty set.
324 |
325 | Induction step. Let $n$ be an arbitrary element and suppose if $A$ has
326 | $n$ elements then $P(A)$ has $2^n$ elements. Let $A$ has $n+1$
327 | elements. Then $P(A)$ can be divided into two sets $A_1$ and $A_2$.
328 |
329 | $A_1$ = Set which doesn't have $(n+1)$th element in it.
330 | $A_2$ = Set which has $(n+1)$th element in it.
331 |
332 | By inductive hypothesis, $A_1$ has $2^n$ elements. Now $A_2 = A_1
333 | \times \{x\}$ where $x$ is the $(n+1)$th element.
334 |
335 | So, $2^n + 2^n = 2^{n+1}$ elements.
336 |
337 | \section{Solution 11}
338 | By mathematical induction,
339 | Base case. When $n=0$, $P_2(A)$ has zero elements, so base case holds.
340 |
341 | Induction step. Let $n$ be an arbitrary element and suppose if $A$ has
342 | $n$ elements then $P_2(A)$ has $n(n-1)/2$ elements. Suppose $A$ has
343 | $n+1$ elements. Let us pick an arbitrary element $a$ from $A$ such
344 | that $A' = A \setminus \{a\}$. Now $A'$ has $n$ elements and $P_2(A')$
345 | has $n(n-1)/2$ elements. There are two kinds of subsets of $A$ those
346 | that contain $a$ as an element and those that don't. The subset hat
347 | don't contain $a$ are just the subsets of $A'$ and by inductive
348 | hypothesis there are $n(n-1)/2$ of them. Now the subset of $A$ that
349 | contain $A$ and have a length of two are $n$. Summing them, we have
350 | $n(n+1)/2$ elements as required.
351 |
352 | \section{Solution 12}
353 | By mathematical induction,
354 |
355 | Base case. When $n=1$, the remaining area is a trapezoidal.
356 |
357 | Induction step. Let $n$ be an arbitrary element and suppose if an
358 | equilateral triangle is cut into $4^n$ congruent equilateral triangles
359 | and one corner is removed, then the remaining area is covered by
360 | trapezoidal tiles. Suppose equilateral triangle is cut into $4^{n+1}$
361 | congruent triangles with $4^n$ triangles in each of them. Now you can
362 | view this as $4$ congruent equilateral triangles. Now since one corner
363 | has been removed, one triangle out of the $4$ has remaining area of it
364 | is covered by trapezoidal tiles. Now for the remaining three triangle,
365 | cut a trapezoidal area such that a corner is removed from each of
366 | them. Then again applying inductive hypothesis on each of them, we get
367 | trapezoidal area in each of them.
368 |
369 | \section{Solution 13}
370 | By mathematical induction on $n$,
371 |
372 | Base case. When $n = 1$, the chord divides the circle into $2$.
373 | Calculating $\frac{n^2 + n + 2}{2}$, we know that base case holds.
374 |
375 | Induction step. Let $n$ be an arbitrary element such that $n$ chords
376 | cut the circle into $\frac{n^2 + n + 2}{2}$ regions. Now suppose
377 | $(n+1)$ chords are drawn in the circle. Now for $n$ chords, we have
378 | $\frac{n^2 + n + 2}{2}$ regions. And for the next chord, we have
379 | $(n+1)$ regions.
380 | So,
381 | \begin{align*}
382 | \frac{n^2 + n + 2}{2} + (n+1) \\
383 | = \frac{n^2 + 3n + 4}{2} \\
384 | = \frac{(n+1)^2 + (n+1) + 2}{2}
385 | \end{align*}
386 |
387 | \section{Solution 15}
388 | You cannot make any assumptions about $n$. $n \in A$ doesn't imply
389 | $(n+1) \in A$.
390 |
391 | \section{Solution 16}
392 | Induction case assumes that there is at least three elements of $A$
393 | which is not the case always. $n >= 0$, so $n$ can be $2$.
394 | \end{document}
395 |
--------------------------------------------------------------------------------
/chapter 4/section4-3.tex:
--------------------------------------------------------------------------------
1 | %%%%%%%%%%%%%%%%%%%%%%%%%%%%%%%%%%%%%%%%%
2 | % Author: Sibi
3 | %%%%%%%%%%%%%%%%%%%%%%%%%%%%%%%%%%%%%%%%%
4 | \documentclass{article}
5 | \usepackage{graphicx}
6 | \usepackage{verbatim}
7 | \usepackage{amsmath}
8 | \usepackage{amsfonts}
9 | \usepackage{amssymb}
10 | \usepackage{tabularx}
11 | \setlength\parskip{\baselineskip}
12 | \begin{document}
13 | \title{Chapter 4 (Section 4.3)}
14 | \author{Sibi}
15 | \date{\today}
16 | \maketitle
17 | \newpage
18 |
19 | \section{Problem 1}
20 |
21 | $R = \{(a,bad), (a,cab), (b,bad), (b,bed), (b,cab), (c,cab), (d,bad),
22 | (d, bed), (e,bed), (e,bed)\}$
23 |
24 | \section{Problem 2}
25 |
26 | $A = \{cat, dog, bird, rat\}$
27 | $R = \{(x,y) \in A \times A \mid there is at least one letter that
28 | occurs in both of the words x and y\}$
29 |
30 | $R = \{(cat, cat), (cat, rat), (dog, dog), (dog, birds), (bird, dog),
31 | (bird, bird), (bird, rat), (rat, cat), (rat, bird), (rat, rat)\}$
32 |
33 | \begin{itemize}
34 | \item Reflexive
35 | \item Symmetric
36 | \item Not transitive: there is an edge from bird to rat and rat to
37 | cat. But there is no edge from bird to cat.
38 | \end{itemize}
39 |
40 | \section{Problem 3}
41 |
42 | $i_{A} = \{(x,y) \in A \times A \mid x = y\}$
43 | $i_{A} = \{(1,1), (2,2), (3,3), (4,4)\}$
44 |
45 | \section{Problem 4}
46 |
47 | \subsection(Solution (a))
48 |
49 | $R = \{(a,c), (c,c), (d,a), (d,b), (b,d)\}$
50 |
51 | \begin{itemize}
52 | \item Not reflexive since $(a,a) \notin R$
53 | \item Not symmetric since $(a,c) \implies (c,a)$ doesn't hold
54 | \item Not transitive since $(d,a) \land (d,c) \implies (d,c)$ doesn't hold
55 | \end{itemize}
56 |
57 | \subsection{Solution (b)}
58 |
59 | $R = \{(a,b), (b,a), (a,d), (b,d)\}$
60 |
61 | \begin{itemize}
62 | \item Not reflexive since $(a,a) \notin R$
63 | \item Not symmetric since $(a,d) \implies (d,a)$ doesn't hold
64 | \item Not transitive since $(a,b) \land (b,a) \implies (a,a)$ doesn't hold
65 | \end{itemize}
66 |
67 | \subsection{Solution (c)}
68 |
69 | $R = \{(a,a), (c,c), (b,b), (b,d), (d,b), (d,d)\}$
70 |
71 | \begin{itemize}
72 | \item Reflexive
73 | \item Symmetric
74 | \item Transitive
75 | \end{itemize}
76 |
77 | \subsection{Solution (d)}
78 |
79 | $R = \{(a,c), (a,b), (a,d), (b,d), (c,d)\}$
80 |
81 | \begin{itemize}
82 | \item Not reflexive
83 | \item Not symmetric
84 | \item Transitive
85 | \end{itemize}
86 |
87 | \section{Solution 5}
88 |
89 | $S \circ R = \{(a,c) \in A \times C \mid \exists b \in B((a,b) \in R
90 | \land (b,c) \in S\}$
91 |
92 | $S \circ R = \{(b,x), (a,y), (c,y), (a,z), (c,z)\}$
93 |
94 | \section{Solution 6}
95 |
96 | $ D_r = \{(x,y) \in R \times R \mid |x - y| < r \}$
97 | $ D_s = \{(x,y) \in R \times R \mid |x - y| < s \}$
98 |
99 | $ D_r \circ D_s = \{(a,b) \in R \times R \mid \exists c \in R((a,c)
100 | \in D_s \land (c,b) \in D_r)\}$
101 | $= \{(a,b) \in R \times R \mid \exists c \in R (|a - c| < s \land |c -
102 | b| < r)\}$
103 |
104 | From triangle inequality, $|a + b| < |a| + |b|$
105 | $|(a-c) + (c-b)| < |a-c| + |c-b|$
106 | $|a + (-b)| < s + r$
107 | $ D_r \circ D_s = \{(a,b) \in R \times R \mid |a-b| < s + r\}$
108 |
109 | \section{Problem 7}
110 |
111 | ($\Rightarrow$) Suppose $R$ is reflexive. Then from the definition of
112 | reflexive it follows that $\forall. x \in A((x,x) \in R)$. Then
113 | clearly, $i_A \subseteq R$.
114 |
115 | ($\Leftarrow$) Suppose $i_A \subseteq R$. Let $x$ be an arbitrary
116 | element in $A$. Then $(x,x) \in i_A$. From $i_A \subseteq R$, it
117 | follows that $(x,x) \in R$. Since $x$ was arbitrary we can conclude
118 | that $R$ is reflexive.
119 |
120 | \section{Problem 8}
121 |
122 | ($\Rightarrow$) Suppose $R$ is transitive. Let $(x,z)$ be an arbitrary
123 | element in $R \circ R$. Then by existential instantiation, $(x,c) \in
124 | R$ and $(c,z) \in R$. Since $R$ is transitive, it follows that $(x,z)
125 | \in R$. So $R \circ R \subseteq R$.
126 |
127 | ($\Leftarrow$) Suppose $R \circ R \subseteq R$. Let $x$, $y$, $z$ be
128 | an arbitrary element in $A$ such that $(x,y) \in R$ and $(y,z) \in R$.
129 | So, $(x,z) \in R \circ R$. It follows that $(x,z) \in R$. Since $x,z$
130 | are arbitrary, $R$ is transitive.
131 |
132 | \section{Problem 9}
133 |
134 | \subsection{Solution (a)}
135 |
136 | ($\Rightarrow$) Suppose $R \circ i_A$. Let $(a,b)$ be an arbitrary
137 | element in $R \circ i_A$. By existential instantiation, it follows
138 | that $(a,c) \in i_A$ and $(c,b) \in R$. From $(a,c) \in i_A$, it
139 | follows that $a = c$. So, $(a,b) \in R$. Since $(a,b)$ is arbitrary it
140 | follows that $R \circ i_A \subseteq R$.
141 |
142 | ($\Leftarrow$) Suppose $R$. Let $(a,b)$ be an arbitrary element in
143 | $R$. Now $a \in A$. So, $(a,a) \in i_A$. Therefore $(a,b) \in R \circ
144 | i_A$. Since $(a,b)$ is arbitrary, we can conclude that $R \subseteq R
145 | \circ i_A$.
146 |
147 | \subsection{Solution (b)}
148 |
149 | ($\Rightarrow$) Suppose $i_B \circ R$. Let $(a,b)$ be an arbitrary element in $i_B
150 | \circ R$. By existential instantiation, $(a,c) \in R$ and $(c,b) \in
151 | i_B$. From the definition of identity relation it follows that $c =
152 | b$. So, $(a,b) \in R$. Therefore, $i_B \circ R \subseteq R$.
153 |
154 | ($\Leftarrow$) Let $(a,b)$ be an arbitrary element in $R$. Now $b \in
155 | B$. So, $(b,b) \in i_B$. So, $(a,b) \in i_B \circ R$. Since $(a,b)$ is
156 | arbitrary it follows that $R \subseteq i_B \circ R$.
157 |
158 | \section{Problem 10}
159 |
160 | Let $(a,a)$ be an arbitrary element in $i_D$. It follows that $a \in
161 | D$. By existential instantiation, $(a,b) \in S$. Also, $(b,a) \in
162 | S^{-1}$. So, $(a,a) \in S^{-1} \circ S$. Since $(a,a)$ was an
163 | arbitrary element it follows that $i_D \subseteq S^{-1} \circ S$.
164 |
165 | Let $(a,a)$ be an arbitrary element in $i_R$. It follows that $a \in
166 | R$. By existential instantiation, it follows that $(b,a) \in S$. Also,
167 | $(a,b) \in S^{-1}$. So $(a,a) \in S \circ S^{-1}$. Since $(a,a)$ was
168 | an arbitrary element it $i_R \subseteq S \circ S^{-1}$.
169 |
170 | \section{Problem 11}
171 |
172 | Suppose $R$ is reflexive. Let $(a,b)$ be an arbitrary element in $R$.
173 | It follows that both $a \in A$ and $b \in A$. Now since $R$ is
174 | reflexive it follows that $(b,b) \in R$. Therefore $(a,b) \in R \circ
175 | R$. Since $(a,b)$ is arbitrary it follows that $R \subseteq R \circ R$.
176 |
177 | \section{Problem 12}
178 |
179 | \subsection{Solution (a)}
180 | Suppose $R$ is reflexive. Let $a$ be an arbitrary element in $A$. Then
181 | it follows that $(a,a) \in R$ . Then clearly $(a,a) \in R^{-1}$. Since
182 | $a$ was arbitrary it follows that $\forall a \in A((a,a) \in R^{-1})$.
183 | Therefore, $R^{-1}$ is reflexive.
184 |
185 | \subsection{Solution (b)}
186 | Suppose $R$ is symmetric. Let $a$ and $b$ be an arbitrary element in
187 | $A$ such that $(a,b) \in R^{-1}$. From $(a,b) \in R^{-1}$ it follows
188 | that $(b,a) \in R$. Since $R$ is symmetric, it follows that $(a,b) \in
189 | R$. So, $(b,a) \in R^{-1}$. Therefore, if $(a,b) \in R^{-1}$ then
190 | $(b,a) \in R^{-1}$. Since $a$ and $b$ are arbitrary, it follows that
191 | $R^{-1}$ is symmetric.
192 |
193 | \subsection{Solution (c)}
194 | Suppose $R$ is transitive. Let $a$, $b$ and $c$ be an arbitrary
195 | element in $A$ such that $(a,b) \in R^{-1}$ and $(b,c) \in R^{-a}$. It
196 | follows that $(b,a) \in R$ and $(c,b) \in R$. Since $R$ is transitive
197 | we get $(c,a) \in R$. So, clearly $(a,c) \in R^{-1}$. Therefore
198 | $R^{-1}$ is transitive.
199 |
200 | \section{Problem 13}
201 |
202 | \subsection{Solution (a)}
203 |
204 | Let $a$ be an arbitrary element in $A$. Suppose $R_1$ and $R_2$ are
205 | reflexive. Then it follows that $(a,a) \in R_1$ and $(a,a) \in R_2$.
206 | Therefore $R_1 \cup R_2$ is reflexive.
207 |
208 | \subsection{Solution (b)}
209 |
210 | Let $a, b$ be an arbitrary element in $A$. Suppose $a (R_1 \cup R_2)b$
211 | or $(a,b) \in R_1 \cup R_2$. Let us consider the cases separately:
212 |
213 | Case 1. $(a,b) \in R_1$. Since $R_1$ is symmetric it follows that
214 | $(b,a) \in R_1$. Therefore $(b,a) \in R_1 \cup R_2$.
215 | Case 2. $(a,b) \in R_2$. Since $R_2$ is symmetric it follows that
216 | $(b,a) \in R_2$. Therefore $(b,a) \in R_1 \cup R_2$.
217 |
218 | Since a and b are arbitrary, it follows that $R_1 \cup R_2$ is symmetric.
219 |
220 | \subsection{Solution (c)}
221 |
222 | $R_1 = \{(1,2), (2,1), (1,1)$
223 | $R_2 = \{(4,2), (2,9), (4,9)\}$
224 |
225 | It's not transitive since $(4,2) \in R_1 \cup R_2 \land (2,1) \in R_1
226 | \cup R_2 \implies (4,1) \in R_1 \cup R_2$ doesn't hold.
227 |
228 | \section{Problem 14}
229 |
230 | \subsection{Solution (a)}
231 |
232 | Suppose $R_1$ and $R_2$ are reflexive. Let $a$ be an arbitrary element
233 | in $A$. Since $R_1$ and $R_2$ are reflexive, it follows that $(a,a)
234 | \in R_1 \cap R_2$. Since $a$ is arbitrary, we can conclude that $R_1
235 | \cap R_2$ is reflexive.
236 |
237 | \subsection{Solution (b)}
238 |
239 | Suppose $R_1$ and $R_2$ are symmetric. Let $x$ and $y$ be arbitrary
240 | elements in $A$ such that $(x,y) \in R_1 \cap R_2$. Since both $R_1$
241 | and $R_2$ are symmetric, it follows that $(y,x) \in R_1 \cap R_2$.
242 | Since $x$ and $y$ are arbitrary, we can conclude that $R_1 \cap R_2$
243 | is symmetric.
244 |
245 | \subsection{Solution (c)}
246 |
247 | Suppose $R_1$ and $R_2$ are transitive. Let $x, y$ and $z$ e an
248 | arbitrary element in $A$ such that $(x,y) \in R_1 \cap R_2$ and $(y,z)
249 | \in R_1 \cap R_2$. Since $R_1$ and $R_2$ are transitive, it follows
250 | that $(x,z) \in R_1 \cap R_2$. Therefore $R_1 \cap R_2$ is transitive.
251 |
252 | \section{Problem 15}
253 |
254 | \subsection{Solution (a)}
255 |
256 | Counterexample:
257 |
258 | $A = \{1\}$ \\
259 | $R_1 = \{(1,1)\}$ \\
260 | $R_2 = \{(1,1)\}$ \\
261 | $R_1 \setminus R_2 = \emptyset$
262 |
263 | \subsection{Solution (b)}
264 |
265 | Suppose $R_1$ and $R_2$ are symmetric. Let $x$ and $y$ be arbitrary
266 | element in $A$ such that $(x,y) \in R_1 \setminus R_2$. So,
267 | $(x,y) \in R_1$. Since $R_1$ is symmetric, it follows that
268 | $(y,x) \in R_1$. Also $(x,y) \notin R_2$. Using contrapositive law and
269 | symmetric property, then $(y,x) \notin R_2$. Therefore $(y,x) \in
270 | R_1 \setminus R_2$.
271 |
272 | \subsection{Solution (c)}
273 |
274 | Counterexample: \\
275 | $R_1 = \{(1,2),(2,4),(1,4)\}$ \\
276 | $R_2 = \{(1,3),(3,4),(1,4)\}$ \\
277 | $R_1 \setminus R_2 = \{(1,2), (3,4)\}$
278 |
279 | \section{Problem 16}
280 |
281 | Suppose $R$ and $S$ are reflexive. Let $a$ be an arbitrary element in
282 | $A$. Since $R$ and $S$ are reflexive, it follows that $(a,a) \in R$
283 | and $(a,a) \in S$. It follows that, $(a,a) \in R \circ S$. Since $a$
284 | is arbitrary, we can conclude that $R \circ S$ is reflexive.
285 |
286 | \section{Problem 17}
287 |
288 | Suppose R and S are symmetric.
289 |
290 | ($\Rightarrow$) Suppose $R \circ S$ is symmetric. Let $(x,y)$ be an
291 | arbitrary element in $R \circ S$. Then by existential instantiation,
292 | it follows that $(x,a) \in S$ and $(a,y) \in R$. Now since $R$ and $S$
293 | are symmetric it follows that $(a,x) \in S$ and $(y,a) \in R$. So,
294 | $(y,x) \in S \circ R$. Since $R \circ S$ is symmetric,
295 | $(y,x) \in R \circ S$. Since $x$ and $y$ are symmetric, it follows
296 | that $R \circ S \subseteq S \circ R$. Similarly we can prove that $S
297 | \circ R \subseteq R \circ S$. So, $S \circ R = R \circ S$.
298 |
299 | ($\Leftarrow$) Suppose $S \circ R = R \circ S$. Let $a,b$ be an
300 | arbitrary element in $A$ such that $(a,b) \in R \circ S$. From our
301 | assumption, it follows that $(a,b) \in S \circ R$. By existential
302 | instantiation, $(a,c) \in R$ and $(c,b) \in S$. Since $R$ and $S$ are
303 | symmetric, it follows that $(c,a) \in R$ and $(b,a) \in S$. So
304 | $(b,a) \in R \circ S$. Now since $a$ and $b$ are arbitrary it follows
305 | that $R \circ S$ is symmetric.
306 |
307 | \section{Problem 18}
308 |
309 | Suppose $S \circ R \subseteq R \circ S$. Let $a,b$ and $c$ be
310 | arbitrary elements of $A$ such that $(a,b) \in R \circ S$ and
311 | $(b,c) \in R \circ S$. By existential instantiation, it follows that
312 | $(a,d) \in S$ and $(d,b) \in R$. Similarly, $(b,e) \in S$ and
313 | $(e,c) \in R$. It follows that $(d,e) \in S \circ R$. From our
314 | assumption, it follows that $(d,e) \in R \circ S$. By existential
315 | instantiation, $(d,f) \in S$ and $(f,e) \in R$. From $(e,c) \in R$ and
316 | $(f,e) \in R$, it follows that $(f,c) \in R \circ R$. Similarly from
317 | $(d,f) \in S$ and $(a,d) \in S$, it follows that
318 | $(a,f) \in S \circ S$. Using Theorem 4.3.4(3), $(f,c) \in R$ and
319 | $(a,f) \in S$. So $(a,c) \in R \circ S$. Since $a$ and $c$ are
320 | arbitrary, $R \circ S$ is transitive.
321 |
322 | \section{Problem 19}
323 |
324 | \subsection{Solution (a)}
325 |
326 | Existential instantiation uses same variable.
327 |
328 | \subsection{Solution (b)}
329 |
330 | Counterexample: \\
331 | $R = \{(1,3), (2,9)\}$ \\
332 | $S = \{(\{1\},\{2,3\}), (\{(2,3\}, \{9\})\}$
333 |
334 | \section{Problem 20}
335 |
336 | Suppose $R$ is transitive. Let $X,Y$ and $Z$ be arbitrary elements in
337 | $B$ such that $(X,Y) \in S$ and $(Y,Z) \in S$. From $(X,Y) \in S$, it
338 | follows that $\forall x \in X \forall y \in Y xRy$. Similarly,
339 | $\forall y \in Y \forall z \in Z yRz$. Now since $R$ is transitive,
340 | $\forall x \in X \forall z \in Z xRz$. Therefore $(X,Z) \in S$. Since
341 | $X,Y$ and $Z$ are arbitrary, $S$ is transitive.
342 |
343 | \section{Problem 21}
344 |
345 | \subsection{Solution (a)}
346 |
347 | Suppose $R$ is symmetric. Let $X$ be an arbitrary element in $P(A)$.
348 | We have to prove that $(X,X) \in S$. Let $x$ be an arbitrary element
349 | in $X$. From $x \in X$ and $X \in P(A)$, it follows that $x \in A$.
350 | Since $R$ is symmetric $(x,x) \in R$. Since $x$ is arbitrary, it
351 | follows that $\forall x \in X (x,x) \in R$. Applying universal
352 | instantiation and existential generalization gives us $\forall x \in X
353 | \exists y \in X(x,y) \in R$
354 |
355 | Q.E.D
356 |
357 | \subsection{Solution(b)}
358 |
359 | Counter example:
360 |
361 | $R = \{(1,2), (2,1), (3,1), (1,3)\}$ \\
362 | $X = \{1\}, Y = \{2,3\}$ \\
363 | $(Y,X) \notin S$
364 |
365 | \subsection{Solution (c)}
366 |
367 | Suppose $R$ is transitive. Let $X,Y$ and $Z$ be arbitrary element in
368 | $P(A)$ such that $(X,Y) \in R$ and $(Y,Z) \in R$. Let $x$ be an
369 | arbitrary element in $X$ and $z$ be some element in $Z$. We have to
370 | prove that $xRz$. From $(X,Y) \in R$ and $x \in X$, it follows that
371 | $\exists y \in Y (xRy)$. From $\exists y \in Y (xRy)$ and
372 | $(Y,Z) \in R$ and applying the transitive property, we get $xRz$.
373 |
374 | \section{Problem 22}
375 |
376 | Flaw is in the assumption of $xRy$. No where it is given to be
377 | assumed. \\
378 |
379 | Theorem is false.
380 |
381 | %Phew! this took some time. Finally worked out this solution.
382 | %with help from lots of nice people.%
383 | \section{Problem 23}
384 | Suppose $F \subseteq P(A)$. Let $a,b,c$ be arbitrary elements of A
385 | such that $(a,b) \in R$ and $(b,c) \in R$. We have to prove that
386 | $(a,c) \in R$. Expanding $(a,c) \in R$, we get $\forall X(X \subseteq
387 | A \setminus \{a,c\}) \implies X \cup \{a\} \in F \implies X \cup \{C\}
388 | \in F)$. Let $X$ be arbitrary and suppose $X \subseteq A \setminus
389 | \{a,c\}$ and $X \cup \{a\} \in F$. We have to prove that $X \cup \{c\}
390 | \in F$. We can consider two cases:
391 |
392 | Case 1. $b \notin X$. From $X \subseteq A \setminus \{a,c\}$, it
393 | follows that $a \notin X$ and $c \notin X$. From $(a,b) \in R$ and
394 | $(b,c) \in R$, it follows that $X \cup \{c\} \in F$.
395 |
396 | Case 2. $b \in X$. Let $X_o = X \setminus \{b\}, X' = X_o \cup \{a\},
397 | X'' = X_o \cup \{c\}$. We know that $c \notin X$, so $c \notin X'$. So
398 | $b,c \notin X'$. Also, $X' \cup \{b\} = X_o \cup \{a\} \cup \{b\} = X
399 | \setminus \{b\} \cup \{a\} \cup \{b\}$. Now $X \setminus \{b\} \cup
400 | \{a\} \cup \{b\} = X \cup \{a\} $ since $b \in X$. From $X \cup \{a\}
401 | \in F$, it follows that $X' \cup \{b\} \in F$. It follows that $X'
402 | \cup \{c\} \in F$ from $(b,c) \in R$. $X' \cup \{c\} = X_o \cup \{a\}
403 | \cup \{c\} = X'' \cup \{a\}$, so $X'' \cup \{a\} \in F$. We know that
404 | $a,b \notin X$, so $a,b \notin X_o$. So $a,b \notin X''$. We also know
405 | that $X'' \cup \{a\} \in F$. So from $(a,b) \in R$, it follows that
406 | $X'' \cup \{b\} \in F$. $X'' \cup \{b\} = X_o \cup \{b\} \cup \{c\} =
407 | X \setminus \{b\} \cup \{b\} \cup \{c\}$. $X \setminus \{b\} \cup
408 | \{b\} \cup \{c\} = X \cup \{c\}$ since $b \in X$. Therefore $X \cup
409 | \{c\} \in X$.
410 |
411 | \end{document}
412 |
413 |
414 |
415 |
--------------------------------------------------------------------------------
/chapter 4/section4-5.tex:
--------------------------------------------------------------------------------
1 | %%%%%%%%%%%%%%%%%%%%%%%%%%%%%%%%%%%%%%%%%
2 | % Author: Sibi
3 | %%%%%%%%%%%%%%%%%%%%%%%%%%%%%%%%%%%%%%%%%
4 | \documentclass{article}
5 | \usepackage{graphicx}
6 | \usepackage{verbatim}
7 | \usepackage{amsmath}
8 | \usepackage{amsfonts}
9 | \usepackage{amssymb}
10 | \usepackage{tabularx}
11 | \setlength\parskip{\baselineskip}
12 | \begin{document}
13 | \title{Chapter 4 (Section 4.3)}
14 | \author{Sibi}
15 | \date{\today}
16 | \maketitle
17 | \newpage
18 |
19 | \section{Problem 1}
20 | \subsection{Solution (a)}
21 |
22 | Reflexive closure of $R = \{(b,b), (c,c), (a,a), (a,b), (b,c),
23 | (c,b)\}$ \\
24 | Symmetric closure of $R = (a,a), (a,b), (b,a), (b,c), (c,b)$ \\
25 | Transitive closure of $R = (a,a), (a,b), (a,c), (a,b), (b,c), (c,b),
26 | (b,b)$
27 |
28 | \subsection{Solution (b)}
29 |
30 | Reflexive closure of $R = \{(x,y \in R \times R \mid x < y \lor x =
31 | y)\}$ \\
32 | $= \{(x,y) \in R \times R \mid x \leq y\}$ \\ \\
33 | Symmetric closure of $R = \{(x,y) \in R \times R \mid x < y \lor y <
34 | x\}$ \\ \\
35 | Transitive closure of $R = R$
36 |
37 | \subsection{Solution (c)}
38 |
39 | Reflexive and symmetric closure of $D_r = D_r$
40 | Transitive closure of $D_r = \mathbb{R} \times \mathbb{R}$
41 |
42 | \section{Problem 3}
43 | \subsection{Solution (a)}
44 |
45 | Suppose $R$ is asymmetric. Let $x$ and $y$ be an arbitrary element in
46 | $A$ such that $xRy$ and $yRx$. Since $R$ is asymmetric, from $xRy$ it
47 | follows that $(y,x) \notin R$. But this contradicts with out
48 | assumption that $yRx$. So, it is vacuously true.
49 |
50 | \subsection{Solution (b)}
51 | Suppose $R$ is a strict partial order on $A$. Let $x$ and $y$ be
52 | arbitrary element in $A$ such that $(x,y) \in R$. Let us try to prove
53 | by contradiction. Suppose $(y,x) \in R$. Then from transitive property
54 | of $R$ it follows that $(x,x) \in R$. But this contradicts the fact
55 | that $R$ is irreflexive. So $(y,x) \notin R$.
56 |
57 | \section{Problem 4}
58 | \subsection{Solution (a)}
59 | Suppose $R$ is a strict partial order on $A$. Let $S$ be the reflexive
60 | closure on $R$. That means $S = R \cup i_A$.
61 | \subsubsection{Proof of reflexive}
62 | By the definition of reflexive closure, $S$ is reflexive.
63 |
64 | \subsubsection{Proof of transitive}
65 | Let $x,y$ and $z$ be arbitrary elements in $A$ such that $(x,y) \in S$
66 | and $(y,z) \in S$. We have to prove that $(x,z) \in S$. Now we know
67 | that $S = R \cup i_A$. Let us consider the cases separately:
68 | Case 1. $(x,y) \in R$ Now this itself has two sub cases within it.
69 | \begin{itemize}
70 | \item $y,z \in R$ We know that $R$ is transitive, so $x,z \in R$.
71 | So $(x,z) \in R \cup i_A$.
72 | \item $y,z \in i_A$ From the property of identity set it follows that
73 | $y = z$. Substituting it in $(x,y) \in R$, we get $(x,z) \in R$. So
74 | $(x,z) \in R \cup i_A$.
75 | \end{itemize}
76 | Case 2. $(x,y) \in i_A$ From the property of identity set it follows
77 | that $x = y$. Substituting that in $(y,z) \in S$ gives $(x,z) \in S$.
78 |
79 | \subsubsection{Proof of Anti-symmetric}
80 | Let $x$ and $y$ be arbitrary elements in $A$ such that $(x,y) \in S$
81 | and $(y,x) \in S$. We have to prove that $x = y$. Now we know
82 | that $S = R \cup i_A$. Let us consider the cases separately:
83 | Case 1. $(x,y) \in R$ Now this itself has two sub cases within it.
84 | \begin{itemize}
85 | \item $y,x in R$. Since $R$ is transitive, it follows that $(x,x) \in
86 | R$. But we know that $R$ is irreflexive, so this contradicts the
87 | fact that $(x,x) \notin R$. So $(y,x) \notin R$.
88 | \item $y,x \in i_A$ From the property of identity set it follows that
89 | $y = x$.
90 | \end{itemize}
91 | Case 2. $(x,y) \in i_A$. From the property of identity set, it follows
92 | that $x = y$
93 |
94 | \subsection{Solution (b)}
95 | Suppose $R$ is total strict order. Let $x$ and $y$ be arbitrary
96 | elements in $A$. We have to prove that $xSy \lor ySx$. We know that R
97 | satisfies trichotomy, so $xRy \lor yRx \lor x = y$. Let us consider
98 | the cases separately:
99 | \begin{itemize}
100 | \item Case 1. $xRy$ Since $S = R \cup i_A$, it follows that $(x,y) \in
101 | S$. So $xSy \lor ySx$.
102 | \item Case 2. $yRx$ Similar to the case 1 proof.
103 | \item Case 3. $x = y$ So, $(x,y) \in i_A$. It follows that $(x,y) \in
104 | R \cup i_A$. So $xSy \lor ySx$.
105 | \end{itemize}
106 |
107 | \section{Problem 5}
108 | Suppose $R$ is a relation on $A$. Let $S = R \setminus i_A$.
109 | \subsection{Solution (a)}
110 | Let
111 | $F = \{T \subset A \times A \mid T \subseteq R and T is
112 | irreflexive\}$. We have to prove things here.
113 | \subsubsection{Proof $S \in F$}
114 | Let $x,y$ be arbitrary elements in $A$ such that $(x,y) \in S$. From
115 | $S = R \setminus i_A$, it follows that $(x,y) \in R$. So, $S \subseteq
116 | R$. It also follows that $(x,y) \notin i_A$. So $\forall x \forall y
117 | (x,y) \notin R$. That can be re-stated as $\forall x (x,x) \notin R$.
118 | So $R$ is irreflexive. So, we can conclude that $S \in F$.
119 | \subsubsection{S is the largest element in F}
120 | Let $T$ be an arbitrary element in $F$. We have to prove that $T
121 | \subseteq S$. Let $(x,y)$ be arbitrary element in $T$. Since $T
122 | \subseteq R$, it follows that $(x,y) \in R$. Also since $T$ is
123 | irreflexive, it follows that $x \neq y$. So $(x,y) \notin i_A$. Hence
124 | $(x,y) \in R \setminus i_A$. So, $T \subseteq S$.
125 |
126 | \subsection{Solution (b)}
127 | Suppose $R$ is a partial order on $A$.
128 | \subsubsection{Proof that S is irreflexive}
129 | Let $x$ be an arbitrary element in $A$. Since $R$ is reflexive it
130 | follows that $(x,x) \in R$. From the identity property, it follows
131 | that $(x,x) \in i_A$. So, it can be concluded that $(x,x) \notin R
132 | \setminus i_A$. Therefore $S$ is irreflexive.
133 | \subsubsection{Proof that S is transitive}
134 | Let $x,y$ and $z$ be arbitrary elements in $A$ such that $(x,y) \in S$
135 | and $(y,z) \in S$. We have to prove that $xSz$. We know that
136 | $S = R \setminus i_A$. Then we have $(x,y) \in R \land (x,y) \in i_A$
137 | and $(y,z) \in R \land (y,z) \in i_A$. From
138 | $(x,y) \in R \land (y,z) \in R$, it follows that $(x,z) \in R$ since
139 | $R$ is transitive. Now there can be two cases here. If
140 | $(x,z) \notin i_A$ then $xSz$. But if $(x,z) \in i_A$, then $x = z$.
141 | From $xRy$ and $yRx$, it follows that $x = y$ since
142 | $R is antisymmetric$. But we know that $(x,y) \in S$ and so this
143 | contradicts with the fact that $(x,y) \notin i_A$. Therefore $x \neq
144 | z$. Hence $xSz$.
145 |
146 | \section{Problem 6}
147 | \subsection{Solution (a)}
148 | $S = \{(p,q) \in P \times P \mid q is the descendant of P \}$
149 | \subsection{Solution (b)}
150 | $S^{-1} = \{(a,b) \in P \times P \mid a is the descendant of b\}$
151 | $S \circ S^{-1} = \{(p,q) \in P \times P \mid \exists z \in P((p,z)
152 | \in S^{-1} \land (z,q) \in S\}$ \\
153 | ${(p,q) \in P \times P \mid p is the descendant of z and q is the
154 | descendant of z}$ \\
155 | ${(p,q) \in P \times P \mid p and q are descendant of some same man}$
156 |
157 | \section{Problem 7}
158 | \subsection{Solution (a)}
159 | Let $S$ be the reflexive closure of $R$.
160 | $\Rightarrow$ Suppose $R$ is reflexive. From clause $1$ of reflexive
161 | closure it follows that $R \subseteq S$. From clause 3, it follows
162 | that $S \subseteq R$. So, $S = R$.
163 | $\Leftarrow$ Suppose $R = S$. By clause 2, $R$ is reflexive.
164 |
165 | \subsection{Solution (b)}
166 | Yes, it holds.
167 |
168 | \section{Problem 8}
169 | \begin{align*}
170 | a \in Dom(S) iff \exists b \in B((a,b) \in S) \\
171 | iff \exists b \in B((a,b) \in R \lor (a,b) \in R^{-1}) \\
172 | iff \exists b \in B((b,a) \in R^{-1} \lor (b,a) \in R) \\
173 | iff \exists b \in B((b,a) \in R^{-1} \lor R) \\
174 | iff \exists b \in B((b,a) \in S) \\
175 | iff a \in Ran(S)
176 | \end{align*}
177 |
178 | \begin{align*}
179 | a \in Dom(S) iff \exists b \in B((a,b) \in S) \\
180 | iff \exists b \in B((a,b) \in R \lor R^{-1}) \\
181 | iff \exists b \in B((a,b) \in R \lor (a,b) \in R^{-1}) \\
182 | iff \exists b \in B((a,b) \in R \lor (b,a) \in R) \\
183 | iff \exists b \in B((a,b) \in R) \lor \exists b \in B((b,a) \in R) \\
184 | iff a \in Ran(R) \cup Dom(R)
185 | \end{align*}
186 |
187 | \section{Problem 9}
188 | Let $T = \{(x,y) \in S \mid x \in Dom(R) \land y \in Ran(R)\}$ \\
189 |
190 | \subsection{Proof of $R \subseteq T$}
191 | Let $a,b$ be arbitrary element in $A$ such that $(a,b) \in R$. Since
192 | $a \in Dom(R)$ and $b \in Ran(R)$ it follows that $(a,b) \in T$. Since
193 | $a$ and $b$ are arbitrary, it follows that $R \subseteq T$.
194 |
195 | \subsection{Proof of transitivity of T}
196 | Let $a,b$ and $c$ be arbitrary element in $A$ such that $(a,b) \in T$
197 | and $(b,c) \in T$. So, it follows that $(a,b) \in S$ and $(b,c) \in
198 | S$. Since $S$ is transitive closure, it follows that $(a,c) \in S$.
199 | So, $(a,c) \in T$. Hence T is transitive.
200 |
201 | Proof. Since $T$ is transitive and $R \subseteq T$ it follows that $S
202 | \subseteq T$. Since $S$ is transitive closure of $R$, we know that $R
203 | \subseteq S$. So $Dom(R) \subseteq Dom(S)$.
204 | Let $a,b$ be arbitrary elements in $A$ such that $(a,b) \in S$ . Since
205 | $S \subseteq T$, it follows that $a \in Dom(R)$ and $b \in Dom(R)$.
206 | Since $a$ and $b$ were arbitrary it follows that $S \subseteq R$. So,
207 | $Dom(S) \subseteq Dom(R)$. Therefore $Dom(S) = Dom(R)$.
208 |
209 | \section{Problem 10}
210 | \subsection{Solution (a)}
211 | We know that $A \times A$ is symmetric. Also $R \subseteq A \times A$.
212 | So from the definition of $F$, it follows that $A \times A \in F$.
213 | Therefore $F \neq \emptyset$.
214 |
215 | \subsection{Solution (b)}
216 | \subsubsection{Proof of $R \subseteq \cap F$}
217 | Let $a,b$ be arbitrary elements in $A$ such that $(a,b) \in R$. Let
218 | $T$ be arbitrary elements of $F$. Then by the definition of $F$, $R
219 | \subseteq T$, so $(a,b) \in T$. Since $T$ was arbitrary this shows
220 | that $\forall T \in F((a,b) \in T)$. So $(a,b) \in \cap F$. Thus $R
221 | \subseteq \cap F$.
222 |
223 | \subsubsection{Proof of $\cap F$ is symmetric}
224 | Let $a,b$ be arbitrary element in $A$ such that $(a,b) \in S$. Let $T$
225 | be arbitrary element in $F$. Then by the definition of $F$, $T$ is
226 | symmetric. So $(a,b) \in S = (a,b) \in \cap F$. So $(a,b) \in T$.
227 | Since $T$ is symmetric, $(b,a) \in T$. Now since $F$ is arbitrary,
228 | $(b,a) \in \cap F$. Therefore $\cap F$ is symmetric.
229 |
230 | \subsection{$\cap F$ is smallest element in F}
231 | Let $T$ be arbitrary element in $F$. We have to prove that $\cap F
232 | \subseteq T$. Let $a,b$ be arbitrary element in $A$ such that $(a,b)
233 | \in \cap F$. Since $T$ is arbitrary it follows that $\forall T \in
234 | F((a,b) \in T)$. So, $(a,b) \in T$. Q.E.D.
235 |
236 | \section{Problem 11}
237 | Suppose $R_1$ and $R_2$ are relations on $A$ and $R_1 \subseteq R_2$.
238 | \subsection{Solution (a)}
239 | Let $S_1$ and $S_2$ be reflexive closure of $R_1$ and $R_2$. Then it
240 | follows that $R_1 \subseteq S_1$ and $R_2 \subseteq S_2$. Let $a,b$ be
241 | arbitrary element in $A$ such that $(a,b) \in S_1$. There can be two
242 | cases now:
243 |
244 | Case 1. $(a,b) \in R_1$. Since $R_1 \subseteq R_2$, it follows that
245 | $(a,b) \in R_2$. From $R_2 \subseteq S_2$, it follows that $S_1
246 | \subseteq S_2$.
247 | Case 2. $(a,b) \notin R_1$ Then it follows that $(a,b)$ was built as
248 | part of reflexive closure, so $a = b$. Now $R_2$ is a relation on $A$.
249 | So, reflexive closure of $R_2$ will have $(a,b)$. So $(a,b) \in S_2$.
250 |
251 | \subsection{Transitive closure}
252 | Let $S_1$ and $S_2$ be transitive closure of $R_1$ and $R_2$. Since
253 | the transitive closure of a relation is the smallest transitive
254 | relation containing the original relation, we know $R_1 \subseteq R_2$
255 | and $R_2 \subseteq S_2$. So, $R_1 \subseteq S_2$. $S_2$ is transitive
256 | and contains $R_1$, so it must contain the transitive closure of $R_1$
257 | ie $S_1$. Since the transitive closure is the smallest transitive
258 | relation containing $R_1$. So $S_1 \subseteq S_2$.
259 |
260 | \subsubsection{Symmetric closure}
261 | Let $S_1$ and $S_2$ be symmetric closure of $R_1$ and $R_2$. Let $a,b$
262 | be arbitrary element in $A$ such that $(a,b) \in S_1$. We know that
263 | $R_1 \subseteq S_1$. Let us consider the cases:
264 | Case 1. $(a,b) \in R_1$ Since $R_1 \subseteq R_2$, it follows that
265 | $(a,b) \in R_2$. From $R_2 \subseteq S_2$, it follows that $(a,b) \in
266 | S_2$.
267 | Case 2. $(a,b) \notin R_1$ This means $(b,a) \in R_1$. Since $R_1
268 | \subseteq R_2$, it follows that $(b,a) \in R_2$. Now symmetric closure
269 | of $R_2$ will have $(a,b)$. So $(a,b) \in S_2$.
270 |
271 | \section{Problem 12}
272 | \subsection{Solution (a)}
273 | $S_1 = R_1 \cup i_A$
274 | $S_2 = R_2 \cup i_A$
275 | $S_1 \cup S_2 = R_1 \cup i_A \cup R_2 \cup i_A$
276 | $R_1 \cup R_2 \cup i_A$
277 | $R \cup i_A$
278 | $S$
279 |
280 | \subsection{Solution (b)}
281 | $S_1 = R_1 \cup (R_1)^{-1}$
282 | $S_2 = R_2 \cup (R_2)^{-1}$
283 | \begin{align*}
284 | S_1 \cup S_2 = R_1 \cup R_2 \cup (R_1)^{-1} \cup (R_2)^{-1} \\
285 | = R \cup R^{-1} \\
286 | = S
287 | \end{align*}
288 |
289 | \section{Problem 13}
290 | Suppose $R_1$ and $R_2$ are relations on $A$. Let $R = R_1 \cap R_2$.
291 |
292 | \subsection{Solution (a)}
293 | Let $S_1, S_2$ and $S$ be the reflexive clsoure of $R_1, R_2$ and $R$.
294 |
295 | We know that $R \subseteq R_1$and $R \subseteq R_2$. From exercise 11,
296 | it follows that $S \subseteq S_1 $ and $S \subseteq S_2$. Let $(a,b)$
297 | be arbitrary element in $S$. Then it follows that $(a,b) \in S_1 \land
298 | S_2$. So, $S \subseteq S_1 \cap S_2$.
299 |
300 | \subsection{Solution (b)}
301 | We know that $R \subseteq R_1$ and $R \subseteq R_2$. From exercise
302 | 11, it follows that $S \subseteq S_1$ and $S \subseteq S_2$. So, $S
303 | \subseteq S_1 \cap S_2$.
304 |
305 | \subsection{Solution (c)}
306 | Similar proof.
307 |
308 | \section{Solution 14}
309 | \begin{align*}
310 | R_1 = \{(1,2),(2,5),(2,9)\} \\
311 | R_2 = \{(1,2)\} \\
312 | S_1 = \{(1,2), (2,5), (1,5), (1,9)\} \\
313 | S_2 = \{(1,2)\} \\
314 | R = R_1 \setminus R_2 = \{(2,5),(2,9)\} \\
315 | S_1 \setminus S_2 = \{(2,5), (1,5), (1,9)\} \\
316 | S = \{(2,5), (2,9)\} \\
317 | S_1 \setminus S_2 \nsubseteq S \land S \nsubseteq S_1 \setminus S_2 \\
318 | \end{align*}
319 |
320 | \section{Solution 15}
321 | Suppose $R$ is a relation on $A$. Let $S = R \cup i_A \cup R^{-1}$. We
322 | know that $S$ is symmetric and reflexive. Let $F = \{ T \subseteq A
323 | \times A \mid T \text{is reflexive and symmetric and} R \subseteq
324 | T\}$. Let $T$ be an arbitrary element in $F$. We have to prove that $S
325 | \subseteq T$. Let $(a,b)$ be arbitrary element in $S$. Now let us
326 | consider the cases:
327 |
328 | Case 1. $(a,b) \in R$. Since $R \subseteq T$, it follows that
329 | $(a,b)\in T$. So $S \subseteq T$.
330 | Case 2. $(a,b) \notin R$. Then it means either $(a,b) \in i_A$ ir
331 | $(a,b) \in R^{-1}$. If $(a,b) \in i_A$ then $a = b$. Since $T$ is
332 | reflexive closure, $(a,b) \in T$. If $(a,b) \in R^{-1}$, then $(b,a)
333 | \in R$. Since $R \subseteq T$, it follows that $(b,a) \in T$. Now $T$
334 | is symmetric closure, so $(a,b) \in T$. So, we can conclude that $S
335 | \subseteq T$.
336 |
337 | \section{Solution 16}
338 | Suppose $R$ is a relation on $A$ and $S$ be the reflexive closure of
339 | $R$.
340 | \subsection{Solution (a)}
341 | Suppose $R$ is symmetric. Let $a,b$ be arbitrary elements in $A$ such
342 | that $(a,b) \in S$. We know that $R \subseteq S$. Let us consider the
343 | cases:
344 | Case 1. $(a,b) \in S$. Since $R$ is symmetric, it follows that $(b,a)
345 | \in R \land (b,a) \in S$.
346 | Case 2. $(a,b) \notin R$ Then it means $a = b$. And $(b,a) \in S$.j
347 |
348 | \subsection{Solution (b)}
349 | Suppose $R$ is transitive. Let $a,b,c$ be arbitrary element in $A$
350 | such that $(a,b) \in S$ and $(b,c) \in S$. We know that $R \subseteq
351 | S$. Let us consider the cases:
352 |
353 | Case 1. $(a,b) \in R \land (b,c) \in R$. Since $R$ is transitive it
354 | follows that $(a,c) \in R$. From $R \subseteq S$, it follows that
355 | $(a,c) \in S$.
356 | Case 2. $(a,b) \in R \land (b,c) \notin R$. Then it follows that $b =
357 | c$. So, $(a,c) \in R$. From $R \subseteq S$, it follows that $(a,c)
358 | \in S$.
359 | Case 3. $(a,b) \notin R \land (b,c) \in R$. Then it follows that $a =
360 | b$. So, $(a,c) \in R$. From $R \subseteq S$, it follows that $(a,c)
361 | \in S$.
362 | Case 4. $(a,b) \notin R \land (b,c) \notin R$. Then it follows that $a
363 | = b$ and $b = c$. Since $S$ is reflexive closure of $R$, it follows
364 | that $(a,c) \in S$.
365 |
366 | \section{Solution 17}
367 |
368 | Lemma. Suppose $a S b$. Then there must be an $R$ chain $a R ... R b$.
369 |
370 | Suppose $aSb$. Then there's an R-chain a R x1 R ... R xn b (n >=0 ).
371 | Since $R$ is symmetric, we can reverse every link in the chain: b xn R
372 | ... R x1 R a. Since $S$ includes $R$, this also an $S$ chain b xn S
373 | ... S x1 S a. Since $S$ is transitive $b S a$. QED.
374 | \end{document}
375 |
376 |
377 |
--------------------------------------------------------------------------------
/chapter 3/section3.4.tex:
--------------------------------------------------------------------------------
1 | %%%%%%%%%%%%%%%%%%%%%%%%%%%%%%%%%%%%%%%%%
2 | % Author: Sibi
3 | %%%%%%%%%%%%%%%%%%%%%%%%%%%%%%%%%%%%%%%%%
4 | \documentclass{article}
5 | \usepackage{graphicx}
6 | \usepackage{verbatim}
7 | \usepackage{amsmath}
8 | \usepackage{amsfonts}
9 | \usepackage{amssymb}
10 | \usepackage{tabularx}
11 | \setlength\parskip{\baselineskip}
12 | \begin{document}
13 | \title{Chapter 3 (Section 3.4)}
14 | \author{Sibi}
15 | \date{\today}
16 | \maketitle
17 | \newpage
18 |
19 | \section{Problem 1}
20 |
21 | ( $\Rightarrow$ ) Suppose $\forall x ( P(x) \land Q(x))$. Let y be
22 | arbitrary. Then since $\forall x (P(x) \land Q(x))$, $P(y) \land Q(y)$
23 | and so in particular $P(y)$. Since y is arbitrary, this shows that
24 | $\forall x P(x)$. Similarly, $\forall x Q(x)$ for arbitrary y. Thus,
25 | $\forall x P(x) \land \forall x Q(x)$.
26 |
27 | ( $\Leftarrow$ ) Suppose $\forall x P(x) \land \forall x Q(x)$. Let y
28 | be arbitrary. Then since $\forall x P(x)$, $P(y)$ and similarly since
29 | $\forall x Q(x)$, $Q(y)$. Thus $P(y) \land Q(y)$ and since y is
30 | arbitrary, it follows that $\forall x(P(x) \land Q(x))$.
31 |
32 | \section{Problem 2}
33 |
34 | Suppose $A \subseteq B \land A \subseteq C$. Let x be an arbitrary
35 | element in $A$. From $x \in A$ and $A \subseteq B$, it follows that $x
36 | \in B$. Similarly, from $x \in A$ and $A \subseteq C$, it follows that
37 | $x \in C$. Therefore, $x \in (B \cap C)$. Since x is arbitrary, we can
38 | conclude that $A \subseteq B \cap C$.
39 |
40 | \section{Problem 3}
41 |
42 | Suppose $A \subseteq B$. Let $C$ be an arbitrary set and x be an
43 | arbitrary element in $C \setminus B$. It follows that $x \in C$ and $x
44 | \notin B$. From $x \notin B$ and $A \subseteq B$, it follows that $x
45 | \notin A$. Therefore $x \in C$ and $x \notin A$. Since x is arbitrary,
46 | $C/B \subseteq C/A$.
47 |
48 | \section{Problem 4}
49 |
50 | Suppose $A \subseteq B \cap A \nsubseteq C$. Let x be an arbitrary
51 | element in A. From $x \in A$ and $A \subseteq B$, it follows that $x
52 | \in B$. Let there be some $y \in A$ and from $A \nsubseteq C$, it
53 | follows that $y \notin C$. Therefore, $x \in B$ and $y \notin C$.
54 | Since x is arbitrary, $y \in B$. Therefore, $y \nsubseteq C$.
55 |
56 | \section{Problem 5}
57 |
58 | Suppose $A \subseteq B \setminus C$ and $A \neq \emptyset$. Let x be
59 | some element in A. From $x \in A$ and $A \subseteq B \setminus C$, it
60 | follows that $x \in B$ and $x \notin C$. Therefore $x \in B \land x
61 | \notin C$, so $B \nsubseteq C$.
62 |
63 | \section{Problem 6}
64 |
65 | Let x be arbitrary, then
66 | \begin{align*}
67 | x \in A \setminus (B \cap C) \iff x \in A \land x \notin (B \cap C) \\
68 | \iff x \in A \land \neg (x \in B \land
69 | C) \\
70 | \iff x \in A \land \neg(x \in B \land x
71 | \in C) \\
72 | \iff x \in A \land (x \notin B \lor x
73 | \notin C) \\
74 | \iff (x \in A \land x \notin B) \lor (x
75 | \in A \land x \notin C) \\
76 | \iff x \in (A \setminus B) \lor x \in
77 | (A \setminus C) \\
78 | \iff x \in (A \setminus B) \cup (A
79 | \setminus C)
80 | \end{align*}
81 |
82 | Thus, $\forall x (x \in A \setminus (B \cap C)) \iff x \in (A
83 | \setminus B) \cup (A \setminus C)$, so $A \setminus (B \cap C) = (A
84 | \setminus B) \cup (A \setminus C)$.
85 |
86 | \section{Problem 7}
87 |
88 | ( $\Rightarrow$ ) Let A and B be arbitrary set. Suppose $x \in P(A
89 | \cap B)$. Then by definition, $x \subseteq A \cap B$. Let y be an
90 | arbitrary element such that $y \in x$. From $y \in x$ and $x \subseteq
91 | A \cap B$, it follows that $y \in A \cap B$. Similarly, $x \in P(B)$.
92 | Therefore $x \in (P(A) \cap P(B))$.
93 |
94 | ( $\Leftarrow$ ) Suppose $x \in P(A) \cap P(B)$. Then $x \in P(A)$ and
95 | $x \in P(B)$, so $x \subseteq A$ and $x \subseteq B$. Let y be an
96 | arbitrary element in x, then $y \in A$ and $y \in B$. Therefore, $y
97 | \in A \cap B$. Since $y \in x$ and $y \in A \cap B$, it follows that
98 | $x \subseteq A \cap B$. Thus, $x \in P(A \cap B)$.
99 |
100 | \section{Problem 8}
101 |
102 | ( $\Rightarrow$ ) Suppose $A \subseteq B$. Let x be an arbitrary
103 | element in $A$. It follows that $x \in A$ and $x \in B$, therefore $x
104 | \subseteq P(A)$ and $x \subseteq P(B)$. Let y be an arbitrary element
105 | in x. From, $y \in x$ and $x \subseteq P(A)$, it follows that $y \in
106 | P(A)$. Similarly $y \in P(B)$. Since y is arbitrary, $P(A) \subseteq
107 | P(B)$.
108 |
109 | ( $\Leftarrow$ ) Let x be an arbitrary element in $P(A)$. Suppose
110 | $P(A) \subseteq P(B)$. It follows that $x \in P(A)$ and $x \in P(B)$,
111 | therefore $x \subseteq A$ and $x \subseteq B$. From here, we can
112 | conclude that $A \subseteq B$.
113 |
114 | \section{Problem 9}
115 |
116 | Suppose x and y are odd integers. Let k be some number such that $x =
117 | 2k + 1$. Similarly, let j be some number such that $y = 2j + 1$.
118 | Multiplying x and y, we get $2(2kj + k + j) + 1$. Since $2kj + k + j$
119 | is an integer, we can conclude that $xy$ is an odd number.
120 |
121 | \section{Problem 10}
122 |
123 | ( $\Rightarrow$ ) Let n be an arbitrary number in $ \mathbb{Z}$. We will prove
124 | the contrapositive. Suppose n is a odd number. Then there exists some
125 | k such that $n = 2k + 1$. Multiplying n with $n^2$ we get $2(2k^3 +
126 | 6k^2 + 3k) + 1$. Since $2k^3 + 6k^2 + 3k + 1$ is an integer, it
127 | follows that $n^3$ is an odd number.
128 |
129 | ( $\Leftarrow$ ) Let n be an arbitrary number in $ \mathbb{Z} $. Suppose n is
130 | even. Then there is an some number k such that $n = 2k$. Multiplying n
131 | by $n^2$, we get $n(n^2) = 2k(2k)^2 = 2k(4k^2) = 2(4k^3)$. Since
132 | $4k^3$ is an integer, it follows that $n^3$ is an even number.
133 |
134 | \section{Problem 11}
135 |
136 | Solution(a)
137 |
138 | You cannot introduce the same integer $k$ in both cases.
139 |
140 | Solution(b)
141 |
142 | \begin{align*}
143 | n = 0 \land m = 1 \\
144 | n^2 - m^2 = 0 - 1 = -1 \\
145 | n + m = 0 + 1 \\
146 | \end{align*}
147 |
148 | \section{Problem 12}
149 |
150 | ( $\Rightarrow$ ) Let x be an arbitrary element in R. Let there be
151 | some $y$ in $\mathbb{R}$. We will prove the contrapositive. Suppose $x
152 | = 1$, then $x + y = 1 + y$ and $xy = y$. So, $x + y \neq xy$.
153 | Therefore, if $x + y = xy$, then $x \neq 1$.
154 |
155 | ( $\Leftarrow$ ) Let x be an arbitrary element in $R$. Suppose $x \neq
156 | 1$. Let $y = x / x - 1$. Summing it with $x$, $x+y = xy$. Therefore if
157 | $ x \neq 1$, then $x + y = xy$.
158 |
159 | \section{Problem 13}
160 |
161 | ( $\Rightarrow$ ) Let $z = 1$. Let x be an arbitrary element in
162 | $\mathbb{R^+}$. Let $y$ be an element in $\mathbb{R}$. Suppose $y - x
163 | = y / x$. We will prove the contradiction. Let $x = z$, it follows
164 | that $x = 1$. Putting it in $y - x$, we get $y -x = y - 1$. Similarly,
165 | putting it in $y - x$, we get $y - x = y - 1$. Similarly, putting it
166 | in $y / x$, we get $y / x = y$. Therefore $y - x \neq y/x$. But it
167 | contradicts the fact that $y - x = y / x$, so $x \neq z$. Therefore,
168 | if $y -x = y/x$, then $x \neq z$.
169 |
170 | ( $\Leftarrow$ ) Let $z = 1$. Let $x$ be an arbitrary element in
171 | $\mathbb{R^+}$. Let $y = x^2 / x - 1$. Suppose $x \neq z$. Solving $ y
172 | - x = \frac{x^2}{x - 1} = \frac{x}{x - 1} * \frac{x}{x} =
173 | \frac{y}{x}$. So, $y - x = \frac{y}{a}$. Therefore if $x \neq z$, then
174 | $y - x = \frac{y}{x}$.
175 |
176 | \section{Problem 14}
177 |
178 | Let x be an arbitrary element in $\cup \{ A \setminus B | A \in F \}$.
179 | It follows that there exists some $A$ in $F$ such that $x \in A
180 | \setminus B$. So, $x \in A$ and $x \notin B$. From $x \in A$ and $x
181 | \notin B$, it follows that $A \nsubseteq B$. Since $A \nsubseteq B$,
182 | it follows that $A \notin P(B)$. So $A \in F$ and $A \notin P(B)$, $A
183 | \in F \setminus P(B)$. Since $x \in A$, it follows that $x \in \cup(F
184 | \setminus P(B))$. Since x is arbitrary, we can conclude that $\cup \{
185 | A \setminus B | A \in F \} \subseteq \cup (F \setminus P(B))$.
186 |
187 | \section{Problem 15}
188 |
189 | Let $A$ be an arbitrary element in $F$ and $B$ be some element in $G$.
190 | Suppose $\forall A \in F \exists B \in G (A \cap B = \emptyset)$. Let
191 | $x$ be an arbitrary element in $A$. From $x \in A$ and $A \in F$, it
192 | follows that $x \in \cup F$. Let us assume that $x \in \cap G$. Then
193 | for all element in $G$, x is present in it. But this contradicts the
194 | fact that there is some element $B in G$ such that $x \notin B$.
195 | Therefore $x \notin \cap G$, So $\cup F$ and $\cap G$ are disjoint
196 | sets.
197 |
198 | \section{Problem 16}
199 |
200 | ( $\Leftarrow$ ) Let $x$ be an arbitrary element in $A$. Since $A \in
201 | P(A)$, then by definition of $\cup P(A)$, $x \in \cup P(A)$.
202 |
203 | ( $\Rightarrow$ ) Let $x$ be an arbitrary element in $\cup P(A)$. Then
204 | by definition, there exists some element $B$ in $P(A)$ such that $x
205 | \in B$. Since $B \in P(A)$, it follows that $B \subseteq A$. From $x
206 | \in B$ and $B \subseteq A$, it follows that $x \in A$. Since x is
207 | arbitrary, $\cup P(A) \subseteq A$.
208 |
209 | \section{Problem 17}
210 |
211 | Solution (a)
212 |
213 | ( $\Leftarrow$ ) Let $x$ be an arbitrary element in $\cup(F \cap G)$.
214 | Then there exists some element $A$ in $F \cap G$ such that $x \in A$.
215 | Since $A \in F$ and $A \in G$, from $x \in A$, we can conclude that $x
216 | \in \cup F$ and $x \in \cup G$. Therefore, $x \in (\cup F) \cap (\cup
217 | G)$. Since $x$ is arbitrary, we can conclude that $(F \cap G)
218 | \subseteq (\cup F) \cap (\cup G)$.
219 |
220 | Solution (b)
221 |
222 | Both $A$ cannot be same.
223 |
224 | Solution (c)
225 |
226 | \begin{align*}
227 | F = \{\{1\}\} \\
228 | G = \{\{1\}, \{2\}\} \\
229 | \cup (F \cap G) = \emptyset \\
230 | \cup (F) \cap \cup (G) = \{1\} \\
231 | \end{align*}
232 |
233 | \section{Problem 18}
234 |
235 | ( $\Rightarrow$ ) Suppose $(\cup F) \cap (\cup G) \subseteq \cup (F
236 | \cap G)$. Let $x$ be an arbitrary element in $\forall A \in F \forall
237 | B \in G (A \cap B)$, so $x \in A$ and $x \in B$. Suppose $A \in F$ and
238 | $x \in A$, it follows that $x \in \cup F$. Similarly, $x \in \cup G$,
239 | therefore $x \in (\cup F) \cap (\cup G)$. From, $x \in (\cup F) \cap
240 | (\cup G)$ and $(\cup F) \cap (\cup G) \subseteq \cup (F \cap G)$ it
241 | follows that $x \in \cup (F \cap G).$ Since $x$ is arbitrary $\forall
242 | A \in F \forall B \in G (A \cap B) \subseteq \cup (F \cap G)$.
243 |
244 | ( $\Leftarrow$ ) Suppose $\forall A \in F \forall B \in G (A \cap B)
245 | \subseteq \cup (F \cap G)$. Let $x$ be an arbitrary element in $(\cup
246 | F) \cap (\cup G)$. Therefore $x \in \cup F$, it follows that there
247 | exists a set $f \in F$ such that $x \in f$. Similarly there exists a
248 | set $g \in G$ such that $x \in g$. Since $\forall A \in F \forall B \in G (A \cap B)
249 | \subseteq \cup (F \cap G)$, so $(f \cap g) \subseteq \cup (F \cap G)$,
250 | it follows $x \in (F \cap G)$. Since $x$ is arbitrary, we can conclude
251 | that $(\cup F) \cap (\cup G) \subseteq \cup(F \cap G)$.
252 |
253 | \section{Problem 19}
254 |
255 | ( $\Rightarrow$ ) Suppose $(\cup F) \cap (\cup G) = \emptyset$. Let
256 | $A$ be an arbitrary element in $F$ and $B$ be an arbitrary element in
257 | $G$. Let $x$ be an arbitrary element in $A$. From $x \in A$ and $A \in
258 | F$, it follows that $x \in \cup F$. Also, from $x \in \cup F$ and
259 | $(\cup F) \cap (\cup G)$, it follows that $x \notin \cup G$, so $x
260 | \notin B$. Since $x$ is arbitrary and $x \in A$ and $x \notin B$ it
261 | follows that $A \cap B = \emptyset$. Therefore if $(\cup F) \cap (\cup
262 | G) = \emptyset$, then $\forall A \in F \forall B \in G (A \cap B =
263 | \emptyset)$.
264 |
265 | ( $\Leftarrow$ ) Suppose $\forall A \in F \forall B \in G (A \cap B =
266 | \emptyset)$. Let $x$ be an arbitrary element in $\cup F$. Then there
267 | is some element $f$ in $F$ such that $x \in F$. From $\forall A \in F
268 | \forall B \in G (A \cap B = \emptyset)$, it follows $f \cap B =
269 | \emptyset$. Since $x \in f$, it follows that $x \notin B$. Since $x
270 | \notin B$, it follows that $x \notin \cup G$. Since $x$ is arbitrary
271 | and $x \in \cup F$ and $x \notin \cup G$, it follows that $(\cup F)
272 | \cap (\cup G) = \emptyset$. Therefore, if $\forall A \in F \forall B
273 | \in G (A \cap B = \emptyset)$ then $(\cup F) \cap (\cup G)$.
274 |
275 | \section{Problem 20}
276 |
277 | Solution (a)
278 |
279 | Let $x$ be an arbitrary element in $(\cup F) \setminus (\cup G)$, so
280 | $x \in \cup F$ and $x \notin \cup G$. From $x \in \cup F$, we can
281 | conclude that there is some $A$ in $F$ such that $x \in A$. From $x
282 | \notin \cup G$, we can conclude that for all elements $B$ in $G$, $x
283 | \notin B$. $A \notin G$ because if it was in $G$, it will lead to
284 | contradiction. Therefore, $A \in F \setminus G$. From $x \in F$ and $A
285 | \in F \setminus G$ it follows that $x \in \cup (F \setminus G)$. Since
286 | $x$ is arbitrary it follows that $(\cup F) \setminus (\cup G)
287 | \subseteq \cup (F \setminus G)$.
288 |
289 | Solution (b)
290 |
291 | Problem is with this line: Since $x \in A$ and $A \notin G$, $x \notin
292 | \cup G$.
293 |
294 | Example: $x = 1 \land A = \{1\} \land G = \{\{1,2\}\} \land x
295 | \in \cup G$.
296 |
297 | Solution (c)
298 |
299 | ( $\Rightarrow$ ) Suppose $\cup(F \setminus G) \subseteq (\cup F)
300 | \setminus (\cup G)$. Let $A$ be an arbitrary element in $F \setminus
301 | G$ and $B$ be an arbitrary element in $G$. Let $x$ be an arbitrary
302 | element in $A$. From $A \in F \setminus G$ and $x \in A$, it follows
303 | that $x \in \cup (F \setminus G)$. Since $x \in \cup(F \setminus G)$
304 | and from $\cup(F \setminus G) \subseteq (\cup F) \setminus (\cup G)$,
305 | it follows that $x \in (\cup F) \setminus (\cup G)$. So $x \in \cup F$
306 | and $x \notin \cup G$. Since $x \notin \cup G$ and $B \in G$, it
307 | follows that $x \notin B$. So, $x \in A$ and $x \notin B$. Since $x$
308 | is arbitrary $A \cap B = \emptyset$.
309 |
310 | ( $\Leftarrow$ ) Suppose $\forall A \in (F \setminus G) \forall B \in
311 | G(A \cap B = \emptyset)$. Let $x$ be an arbitrary element in $\cup(F
312 | \setminus G)$. Then there is some element $Y$ in $F \setminus G$ such
313 | that $x \in Y$. From $Y \in F \setminus G$, it follows that $Y \in F$
314 | and $Y \notin G$. Since $x \in Y$ and $Y \in F$ it follows that $x \in
315 | \cup F$. We will prove the contradiction. Let us assume $x \in \cup
316 | G$. Since $x \in \cup G$, it follows that there is some element $g$ in
317 | $G$ such that $x \in g$. But this contradicts the fact that for all
318 | element $B$ in $G$ if $x$ is in $A$ and $x \notin B$, so $x \notin \cup
319 | G$. Since $x$ is arbitrary, we can conclude $\cup (F \setminus G)
320 | \subseteq (\cup F) \setminus (\cup G)$.
321 |
322 | Solution (d)
323 | \begin{align*}
324 | F = \{\{1,2\}\} \\
325 | G = \{\{2\}\}
326 | \end{align*}
327 |
328 | \section{Problem 21}
329 |
330 | Suppose $\cup F \nsubseteq \cup G$. Let there be some element $A \in
331 | F$ and let $B$ be an arbitrary element in $G$. Let $x$ be an arbitrary
332 | element in $A$. From $x \in A$ and $A \in F$, it follows that $x \in
333 | \cup F$. From $x \in \cup F$ and $\cup F \nsubseteq \cup G$, it
334 | follows that $x \notin \cup G$. This means that for all element in
335 | $G$, $x$ is not present in element. Therefore $x \notin B$. Since x is
336 | an arbitrary element, $A \nsubseteq B$. So, if $\cup F \nsubseteq \cup
337 | G$, then $\exists A \in F \forall B \in G (A \nsubseteq B)$.
338 |
339 | \section{Problem 22}
340 |
341 | Solution(a)
342 |
343 | Proof strategy using Binconditional and Conjunction.
344 |
345 | Solution(b)
346 |
347 | ($\Rightarrow$) Let $x$ be an arbitrary element in $B \setminus
348 | (\cup_{i \in I} A_i)$, so $x \in B$ and $x \notin \cup_{i \in I}A_i$.
349 | From $x \notin \cup_{i \in I}A_i$, it follows that for all element $i$
350 | in $I$, $x \notin A_i$. So. $x \in B$ and $x \notin A_i$, therefore $x
351 | \in B \setminus A_i$. So, $ x \in \cap_{i \in I}(B \setminus A_i)$.
352 |
353 | ($\Leftarrow$) Let $x$ be an arbitrary element in $\cap_{i \in I}(B
354 | \setminus A_i)$. This means for all element $i \in I$, $x \in B
355 | \setminus A_i$. So $x \in B$ and $x \notin A_i$. Since i is arbitrary
356 | and $x \notin A_i$, it follows that $\forall i \in I(x \notin A_i)$.
357 | So, $x \notin \cup_{i \in I}A_i$. Therefore, $x \in B \setminus
358 | (\cup_{i \in I}A_i)$.
359 |
360 | Solution(c)
361 |
362 | $\cup_{i \in I}(B \setminus A_i)$
363 |
364 | \section{Problem 23}
365 |
366 | Solution(a)
367 |
368 | Let $x$ be an arbitrary element in $\cup_{i \in I}(A_i \setminus
369 | B_i)$. Then there exists an element $i$ in $I$ such that $x \in A_i
370 | \setminus B_i$. So $x \in A_i$ and $x \notin B_i$. From $x \in A_i$,
371 | it follows $x \in \cup_{i \in I}A_i$. From $x \notin B_i$, it follows
372 | that $x \notin \cap_{i \in I}B_i$. Therefore, $x \in (\cup_{i \in
373 | I}A_i) \setminus (\cap_{i \in I}B_i)$. Since $x$ is arbitrary,
374 | $\cup_{i \in I}(A_i \setminus B_i) \subseteq (\cup_{i \in I}A_i)
375 | \setminus (\cap_{i \in I}B_i)$.
376 |
377 | Solution (b)
378 |
379 | \begin{align*}
380 | I = \{1,2\} \\
381 | A_i = \{\{0,1\},\{2\}\} \\
382 | B_i = \{\{2\}\}
383 | \end{align*}
384 |
385 | \section{Problem 24}
386 |
387 | Solution(a)
388 |
389 | Let $x$ be an arbitrary element in $\cup_{i \in I}(A_i \cap B_i)$. Then
390 | there exists some element $i \in I$ such that $x \in A_i \cap B_i$, so
391 | $x \in A_i$ and $x \in B_i$. From $x \in A_i$, it follows that $x \in
392 | \cup_{i \in I}A_i$. Similarly from $x \in B_i$, it follows that $x \in
393 | \cup_{i \in I}B_i$. So, $x \in (\cup_{i \in I}A_i \cap \cup_{i \in
394 | I}B_i)$. Since x is arbitrary, we can conclude that $\cup_{i \in I}
395 | (A_i \cap B_i) \subseteq (\cup_{i \in I} A_i) \cap (\cup_{i \in
396 | I}B_i)$.
397 |
398 | Solution(b)
399 |
400 | \begin{align*}
401 | i = \{1,2\} \\
402 | A_i = \{\{1\},\{2\}\} \\
403 | B_i = \{\{1,2\}, \{3\}\}
404 | \end{align*}
405 |
406 | \section{Problem 25}
407 |
408 | Let $a$ and $b$ be any arbitrary element in $\mathbb{Z}$. Let $c$ be
409 | some element in $\mathbb{Z}$. Since $a$ and $c$ are integer, then
410 | there exists some integer $k$ such that $ak = c$, so $a | c$.
411 | Similarly we can prove that $ b | c$.
412 |
413 | \section{Problem 26}
414 |
415 | Solution(a)
416 |
417 | ($\Rightarrow$) Let $n$ be an arbitrary element in $\mathbb{Z}$.
418 | Suppose $15 | n$. Then there exists an integer $k$ such that $15k =
419 | n$. From $15k = n$, it follows that $3(5k) = n$, so $3 | n$.
420 | Similarly, from $15k = n$, it follows that $5(3k) = n$, so $5|n$.
421 | Therefore, if $15 | n$ then $3|n \land 5|n$.
422 |
423 | ($\Leftarrow$)(incorrect. why?) Suppose $3|n$ and $5|n$. We will prove the
424 | contradiction. Suppose $15 is not divisible by n$, then there exists
425 | some $k$ such that $15 k \neq n$. From $15k \neq n$, it follows $3(5k)
426 | \neq n$. But this contradicts the fact that $3|n$. Therefore, $15 |
427 | n$.
428 |
429 | ($\Leftarrow$)Suppose $3|n$ and $5|n$. Then we can choose
430 | integers $p$ and $q$ s.t. $3p = n$ and $5q = n$.
431 | now $n = 6n-5n = 2(3(5q)) - 5(3p) = 15(2q-p)$.
432 | It follows since $2q-p$ is an integer $15|n$.
433 |
434 | Solution(b)
435 |
436 | $n = 30$
437 |
438 | \end{document}
439 |
440 |
441 |
442 |
--------------------------------------------------------------------------------